a

Menu

M

Chiudi

Teoria sulle successioni

Teoria sulle Successioni

Home » Teoria sulle successioni

Benvenuti nella nostra guida teorica completa alla teoria delle successioni numeriche. L’idea intuitiva di una successione reale è quella di una “lista infinita” di numeri reali:

    \[a_0, \, a_1, \, a_2, \,\dots, \, a_n, \, \dots\]

Formalmente, in Analisi Matematica ciò viene vista come una funzione a \colon \mathbb{N} \to \mathbb{R}. Le successioni sono uno degli strumenti storicamente più antichi e, per questo motivo oltre che per la loro relativa semplicità, costituiscono il fondamento di numerose parti dell’Analisi Matematica, oltre che di altre branche della Matematica.
In questo articolo proponiamo uno studio completo della teoria riguardante questo affascinante argomento; la teoria è corredata da numerosi esempi ed illustrazioni; proponiamo infine una nutrita serie di esercizi, alcuni completamente risolti, altri di cui forniamo solo dei suggerimenti, per offrire al lettore il necessario spazio per applicare e familiarizzare con le tecniche apprese. Concludiamo fornendo infine delle utilissime tabelle riassuntive sui risultati principali, le forme indeterminate e i limiti notevoli incontrati.

Oltre agli articoli teorici reperibili nella cartella Successioni – teoria , consigliamo le ulteriori raccolte di esercizi su questo argomento:

Buona lettura!

 
 

Sommario

Leggi...

Questa dispensa tratta le successioni di numeri reali. Dopo le definizioni fondamentali, si introduce il concetto di limite di successione e le sue relazioni con le usuali operazioni algebriche. Si presentano poi i principali teoremi sui limiti, la nozione di successione di Cauchy, il numero di Nepero e, i principali criteri di convergenza e i limiti notevoli fondamentali. La dispensa si conclude proponendo una nutrita raccolta esercizi svolti e un’appendice sulla potenza a esponente reale.

 
 

Autori e revisori


 
 

Notazioni

Leggi...

\mathbb{N}    Insieme dei numeri naturali: \{1,2,\dots\};
\mathbb{N}_0=\mathbb{N}\cup\{0\}    Insieme dei numeri naturali non negativi: \{0,1,2,\dots\};
\mathbb{Z}    Insieme dei numeri interi relativi
\mathbb{Q}    Insieme dei numeri razionali;
\mathbb{R}    Insieme dei numeri reali;
\overline{\mathbb{R}}    Insieme dei numeri reali estesi, ovvero \overline{\mathbb{R}}\coloneqq\mathbb{R} \cup \{-\infty,+\infty\};
I_\varepsilon(x)    Intorno circolare di raggio \varepsilon di x, ossia (x-\varepsilon,x+\varepsilon);
A^c    \mathbb{R} \setminus A, ovvero il complementare dell’insieme A;
A^{\mathrm{o}}    Parte interna dell’insieme A;
\partial A    Frontiera dell’insieme A;
\{a_n\}_{n \in \mathbb{N}}    Successione di termine generale a_n, ossia la funzione a \colon \mathbb{N} \to \mathbb{R};
\{a_{\phi(k)}\}_{k \in \mathbb{N}}, \{a_{n_k}\}_{k \in \mathbb{N}}    Sottosuccessione della successione \{a_n\}_{n \in \mathbb{N}} ottenuta dalla funzione strettamente crescente \phi \colon \mathbb{N} \to \mathbb{N};
\lim_{n \to + \infty} a_n = \ell, a_n \to \ell    Limite della successione a_n;
a_n =o(b_n)    a_n è un o-piccolo di b_n, ovvero \lim_{n \rightarrow +\infty}\dfrac{a_n}{b_n}=0.;
a_n \sim b_n    a_n è asintotica a b_n, ovvero \lim_{n \rightarrow +\infty}\dfrac{a_n}{b_n}=1..


 
 

Introduzione

Leggi...

Il concetto di successione è storicamente uno dei primi ad essere entrato a far parte della Matematica: sin dai tempi dei babilonesi e degli antichi greci se ne sono comprese le potenzialità e l’utilità.

La nozione di successione formalizza l’idea di “lista” ordinata e infinita di oggetti. Una successione di numeri reali, l’oggetto principale di questa dispensa, si può quindi immaginare come

(1)   \begin{equation*} a_0, \, a_1, \dots,  a_n,\, \dots \end{equation*}

Ogni elemento a_n è un numero reale e tali numeri vengono ordinati mediante un indice costituito da un numero naturale n. Si può quindi formalizzare questa idea come una funzione a \colon \mathbb{N} \to \mathbb{R} in cui l’immagine a(n) del numero reale n tramite a viene indicata col simbolo a_n.

Uno dei concetti fondamentali relativi alle successioni è quello di limite: esso formalizza l’idea intuitiva del valore a cui si avvicinano i numeri a_n al crescere dell’indice n. Questa dispensa è principalmente dedicata allo studio dei limiti delle successioni reali. Dopo una presentazione dei concetti e le proprietà fondamentali relativi alle successioni, il focus viene infatti spostato sul concetto di limite, le sue proprietà algebriche e i risultati che ne descrivono le caratteristiche. Più precisamente, il lavoro è così organizzato.

    \[\quad\]

  • Nella sezione 1 studiamo la topologia dell’insieme \mathbb{R}, presentando la nozione di intorno che risulta fondamentale per definire formalmente il concetto di limite e permette di definire punti interni, di frontiera e di accumulazione di sottoinsiemi di \mathbb{R}.
  •  

  • Nella sezione 2 introduciamo le successioni, ne studiamo le proprietà principali quali limitatezza e monotonia e presentiamo il concetto di sottosuccessione.
  •  

  • Nella sezione 3 parliamo del concetto di limite di una successione e di come esso formalizzi l’idea dell’avvicinarsi dei termini della successione verso un certo valore. Trattiamo anche le proprietà di base dei limiti, come la loro unicità e alcune caratterizzazioni topologiche dei chiusi e dei punti di accumulazione.
  •  

  • Nella sezione 4 parliamo dell’algebra dei limiti, ossia di come il concetto di limite si relazioni con le usuali operazioni algebriche come somma, prodotto e quoziente. Introduciamo inoltre il concetto di forma indeterminata, che appare quando si tenta di estendere ai simboli +\infty e -\infty le usuali operazioni tra numeri reali.
  •  

  • Nella sezione 5 presentiamo i principali teoremi sui limiti: il fatto che le successioni monotone abbiano sempre limite, l’esistenza di sottosuccessioni monotone, i teoremi del confronto, dei carabinieri e della permanenza del segno, concludendo col teorema di Bolzano-Weierstrass sull’esistenza di estratte convergenti.
  •  

  • Nella sezione 6 introduciamo il concetto di successione di Cauchy, ossia successioni i cui termini diventano arbitrariamente vicini tra loro, al crescere dell’indice. Vedremo che tale nozione in \mathbb{R} è intimamente legata a quella di convergenza della successione, essendo infatti esse equivalenti.
  •  

  • Nella sezione 7 presentiamo il celebre numero di Nepero e=2,71\dots, obiquo in ogni campo della Matematica e delle scienze applicate. Lo introduciamo appunto come limite di una successione, per poi darne alcune definizioni equivalenti. Trattiamo inoltre l’esponenziale naturale.
  •  

  • Nella sezione 8 trattiamo i principali criteri di convergenza per successioni a termini positivi, ossia il criterio del rapporto e della radice, forniamo alcuni esempi di applicazione e li confrontiamo tra loro.
  •  

  • Nella sezione 9 presentiamo alcuni esempi di limiti di successione che vengono detti notevoli, in quanto ricorrono frequentemente nel calcolo dei limiti ed è quindi importante tenerli a mente. Presentiamo quindi la cosiddetta gerarchia degli infiniti e altre forme indeterminate di particolare interesse.
  •  

  • Nella sezione 10 introduciamo il concetto di successioni asintoticamente equivalenti e la nozione di o-piccolo, oltre al principio di sostituzione degli infinitesimi.
  •  

  • Nella sezione 11 trattiamo i teoremi di Stolz-Cesaro, che consentono di dedurre il limite di alcune medie di successioni a partire dai limiti delle successioni stesse.
  •  

  • Nella sezione 12 presentiamo il teorema ponte, così denominato in quanto costituisce il legame tra i concetti di limiti di successioni e di funzioni. Esso consente di caratterizzare il limite di una funzione mediante opportuni limiti di successioni.
  •  

  • Nella sezione 13 introduciamo il lettore allo studio delle cosiddette successioni per ricorrenza, ovvero successioni definite fornendo esclusivamente il primo termine e la relazione tra il termine successivo e i precedenti. Vedremo che il loro studio richiede un’attenzione particolare e tecniche interessanti.
  •  

  • Nella sezione 14 riportiamo degli esercizi, alcuni completamente risolti, altri proposti al lettore, in cui forniamo soltanto dei suggerimenti ove necessario.
  •  

  • Nella sezione 16 offriamo al lettore alcuni schemi riassuntivi sulle conclusioni ottenute nel corso della dispensa, in particolare sulle definizioni e concetti fondamentali, sulle forme indeterminate e sui limiti notevoli.
  •  

  • Nell’appendice A offriamo al lettore una trattazione autocontenuta della potenza a esponente reale, che applica il concetto di successione e di limite.

 

Topologia di \mathbb{R}

Introduzione.

Lo scopo fondamentale di questa sezione introduttiva è fornire le nozioni essenziali di topologia necessarie per affrontare lo studio delle successioni a valori reali. Iniziamo con la nozione di intorno di un punto, che formalizza l’idea di un insieme che lo “circonda”.

Definizione 1.1 (intorno, intorno circolare, punti interni ed esterni). Sia x_0 un numero reale. Un intorno circolare di x_0 è un intervallo aperto centrato in x_0, ovvero della forma

    \begin{equation*} 				I_\varepsilon(x_0)=(x_0-\varepsilon,\,x_0+\varepsilon) 			\end{equation*}

con \varepsilon>0. Un insieme J si dice intorno di x se contiene un intorno circolare di x.

Dato un insieme A\subseteq\,\mathbb{R}, diremo che un punto y\in\mathbb{R} è interno ad A se esiste \varepsilon>0 tale che

    \begin{equation*} 				I_\varepsilon(y)\subseteq A. 			\end{equation*}

Diremo inoltre che un punto z\in\mathbb{R} è esterno ad A se è interno al complementare A^c\coloneqq \mathbb{R} \setminus A di A.

Esempio 1.2. Consideriamo il punto x_0=2. Allora l’intorno circolare I_1(2) di centro 2 e raggio 1 è costituito dall’intervallo aperto

(2)   \begin{equation*} 			I_1(2) 			= 			(2-1,2+1) 			= 			(1,3), 		\end{equation*}

rappresentato in blu in figura 1. Considerando l’insieme A=[0,3] \cup [4,5), rappresentato in verde in figura 1, dalla definizione segue che x_0=2 è interno ad A, in quanto esso contiene l’intorno I_1(2) di x_0=2, ovvero I_1(2) \subseteq A.

Il punto x_1= \frac{7}{2}, rappresentato in rosso in figura 1, è invece esterno all’insieme A, in quanto il complementare A^c di A contiene l’intorno I_r(x_1) con r=\frac{1}{4}, ossia

(3)   \begin{equation*} 			I_{\frac{1}{4}}(x_1) \subset A^c. 		\end{equation*}

    \[\quad\]

    \[\quad\]

Teoria sulle successioni

Figura 1: gli insiemi dell’esempio 1.2; si nota che x_0 è interno all’insieme A (rappresentato in verde) in quanto esso contiene l’intorno circolare I_1(2) (rappresentato in blu). Il punto x_1=\frac{7}{2} è invece esterno ad A poiché l’intorno I_{\frac{1}{4}}(x_1) (rappresentato in rosso) è disgiunto da A.

    \[\quad\]

    \[\quad\]

Esempio 1.3. Consideriamo l’insieme A = (1, 2] \subset \mathbb{R}, rappresentato in verde in figura 2.

    \[\quad\]

    \[\quad\]

Teoria sulle successioni

Figura 2: l’insieme A (rappresentato in verde) dell’esempio 1.3; si nota che \frac{3}{2} è interno ad A poiché esso contiene l’intorno circolare I_{\frac{1}{4}}\left(\frac{3}{2} \right) (rappresentato in blu) di \frac{3}{2}. Il punto 3 è invece esterno ad A dato che l’intorno I_{\frac{1}{2}}(3) (rappresentato in rosso) è contenuto nel complementare di A. Si noti che gli estremi 1 e 2 di A non sono né interni ad A né esterni ad esso; essi sono quindi punti di frontiera per A.

    \[\quad\]

    \[\quad\]

Il punto x = \frac{3}{2} è chiaramente un punto interno di A. Infatti, possiamo identificare un \varepsilon = \frac{1}{4} (questo è solo un esempio, ma esistono infiniti altri valori possibili) tale che

    \begin{equation*} 		I_\frac{1}{4}\left(\frac{3}{2}\right)=\left(\frac{3}{2}-\frac{1}{4},\,\frac{3}{2}+\frac{1}{4}\right)=\left(\frac{5}{4},\,\frac{7}{4}\right)\subset (1,\,2].	 	\end{equation*}

Invece il punto x=3 è esterno ad A; infatti se consideriamo il complementare di A, A^c=\left(-\infty,\,1\right]\cup\left(2,+\infty\right) possiamo considerare ad esempio \varepsilon=\frac{1}{2} e concludere che

    \begin{equation*} 		I_\frac{1}{2}(3)=\left(3-\frac{1}{2},\,3+\frac{1}{2}\right)=\left(\frac{5}{2},\,\frac{7}{2}\right)\subset A^c,	 	\end{equation*}

ovvero x=3 è un punto interno del complementare di A.

Definizione 1.4 (punto di frontiera). Un punto x\in\mathbb{R} si dice punto di frontiera per un insieme A\subseteq\mathbb{R} se non è né interno né esterno ad esso.

    \[\quad\]

Esempio 1.5. Per l’insieme A=(1,\,2] i punti di frontiera sono x=1 e x=2.

Esempio 1.6. Consideriamo l’intervallo aperto A = (a, b). In questo caso, ogni punto x \in A è un punto interno. Per contro, ogni y \in \mathbb{R} che soddisfa y < a o y > b è un punto esterno. I punti di frontiera di A sono gli estremi a e b.

Esempio 1.7. Prendiamo in esame l’intervallo chiuso A = [a, b]. Qui, ogni punto x \in \mathbb{R} con a < x < b è interno ad A, mentre ogni y \in \mathbb{R} che verifica y < a o y > b è esterno. I punti di frontiera anche in questo caso sono gli estremi a e b.

Definizione 1.8 (parte interna e frontiera). Sia A\subseteq \mathbb{R}. La parte interna di A si indica con A^{\mathrm{o}} ed è l’insieme dei punti interni ad A. La frontiera di A è invece l’insieme dei punti di frontiera per A e si indica con \partial A.

Osservazione 1.9. La frontiera di un insieme coincide con la frontiera del suo complementare: \partial A=\partial A^c. Infatti, x è un punto di forntiera per l’insieme A se e solo se:

    \[\quad\]

  • x non è un punto interno di (A^c)^c=A;
  •  

  • x non è un punto esterno di A, ovvero x non è un punto interno di A^c.

Quindi per 1.1 il punto x è un punto di frontiera per A^c.

Esempio 1.10. Negli esempi precedenti abbiamo:

    \[\quad\]

  • se A=(1,\,2] allora A^{\mathrm{o}}=(1,\,2) e \partial A=\{1,\,2\};
  •  

  • se A=(a,b) allora A^{\mathrm{o}}=(a,\,b) e \partial A=\{1,\,2\};
  •  

  • se A=[a,b] allora A^{\mathrm{o}}=(a,\,b) e \partial A=\{a,\,b\}.

Definizione 1.11 (insiemi aperti e chiusi). Un insieme A\subseteq\mathbb{R} si dice aperto se ogni suo punto è a esso interno, ossia se per ogni x\in A esiste \varepsilon tale che I_\varepsilon(x)\subseteq A.

Un insieme A\subseteq\mathbb{R} si dice chiuso se contiene i suoi punti di frontiera, ovvero se \partial A\subseteq A.

    \[\quad\]

Un insieme è quindi aperto se non contiene alcun punto della sua frontiera o, in altre parole, se A^{\mathrm{o}}=A. Viceversa, un insieme è chiuso se li contiene tutti. Le nozioni di insieme aperto e chiuso sono quindi complementari, come precisa il prossimo risultato.

Teorema 1.12. Un insieme A\subseteq\mathbb{R} è aperto se e solo se il suo complementare A^c è chiuso.

    \[\quad\]

Dimostrazione. (\Rightarrow) Consideriamo un insieme aperto A \subseteq \mathbb{R} e un punto x_0 \in \partial (A^c) = \partial A, dove l’uguaglianza segue dall’osservazione 1.9. Poiché x_0 non appartiene all’interno di A, segue che x_0 non è in A. Pertanto, x_0 appartiene al complementare di A, ovvero x_0 \in A^c.

(\Leftarrow) Se A^c è chiuso e x \in A, allora x non è di frontiera per A in quanto \partial A=\partial (A^c) \subseteq A^c. Dunque x è interno ad A e quindi A è aperto.

Osservazione 1.13. Poiché ogni intorno di x contiene un intorno circolare di x e, d’altra parte, ogni intorno circolare è un particolare intorno, è chiaro che nelle definizioni e caratterizzazioni degli elementi topologici che ci interessano possiamo limitarci a utilizzare soltanto intorni circolari.

Definizione 1.14 (punto di accumulazione e isolato). Sia A\subseteq \mathbb{R}. Un punto x\in\mathbb{R} si dice punto di accumulazione per A se ogni intorno di x contiene almeno un elemento di A diverso da x, ovvero

    \[\forall  \varepsilon>0\,\,\,\,\exists\,\,\,y\neq x\,\,\text{t.c.}\,\, y\in I_\varepsilon(x).\]

In altre parole x\in\mathbb{R} è di accumulazione per A se e solo se I_\varepsilon (x) \cap A\setminus \{x\}\neq \emptyset per ogni \varepsilon>0. Un punto x\in A si dice punto isolato per A se esiste un intorno di x che non contiene alcun punto di A oltre a x, ovvero se esiste \varepsilon>0 tale che I_\varepsilon (x) \cap A\setminus \{x\}= \emptyset.

    \[\quad\]

Esempio 1.15. Nell’intervallo aperto A=(a,\,b) tutti i punti interni e di frontiera sono di accumulazione per A.

Esempio 1.16. Nell’intervallo chiuso A=[a,\,b] tutti i punti x\in A sono di accumulazione, sia quelli interni che quelli di frontiera.

Come il termine stesso suggerisce, intorno a un punto di accumulazione x_0 per un insieme A, i punti di A si addensano. Infatti, in ogni intorno di x_0 devono esserci infiniti punti dell’insieme A, come stabilito dalla prossima proposizione.

Proposizione 1.17. Sia A\subseteq \mathbb{R}. Un punto x è di accumulazione per A se e solo se ogni intorno I_\varepsilon(x) contiene infiniti punti di A.

    \[\quad\]

Osservazione 1.18. Da questa proposizione segue in particolare che un insieme finito non può possedere alcun punto di accumulazione.

Dimostraione. (\Leftarrow) Se ogni intorno I_\varepsilon(x_0) di x_0 contiene infiniti punti di A, ovviamente esso contiene punti di A distinti da x_0, per cui x_0 è di accumulazione.

(\Rightarrow) Per provare il viceversa, supponiamo che x_0 sia di accumulazione per A e, per assurdo, che esista \varepsilon>0 tale che I_\varepsilon(x_0) \cap A sia finito, ossia

(4)   \begin{equation*} 		I_\varepsilon(x_0) \cap A 		= 		\{x_0,x_1,\dots,x_n\} 		\qquad 		\text{con } n \in \mathbb{N}. 	\end{equation*}

Si veda la figura 3 per una rappresentazione grafica. Scegliamo \delta come metà della minima distanza tra x_0 e i punti x_i, ovvero

(5)   \begin{equation*} 		\delta \coloneqq 		\dfrac{1}{2}\min\{ |x_0-x_1|, \dots, |x_0-x_n|\}. 	\end{equation*}

Da questa scelta segue che \delta < \varepsilon e quindi I_{\delta}(x_0) \subset I_\varepsilon(x_0). D’altra parte, da (5) segue che nessun x_i appartiene a I_\delta(x_0), altrimenti si avrebbe

(6)   \begin{equation*} 		|x_i-x_0| < \delta = \dfrac{1}{2}\min\{ |x_0-x_1|, \dots, |x_0-x_n|\} < |x_0-x_i|, 	\end{equation*}

che è assurdo. Da ciò e da I_{\delta}(x_0) \subset I_\varepsilon(x_0) segue che

(7)   \begin{equation*} 		I_\delta(x_0) \cap A = \{x_0\}, 	\end{equation*}

ma ciò contraddice il fatto che x_0 sia un punto di accumulazione, in quanto l’intorno I_\delta(x_0) di x_0 contiene soltanto il punto x_0 di A.

    \[\quad\]

    \[\quad\]

Teoria sulle successioni

Figura 3: schematizzazione della dimostrazione della proposizione 1.17. Se l’intorno I_{\varepsilon}(x_0) (in verde) contenesse un numero finito di punti di A (rappresentati in verde e grigio), allora esisterebbe un intorno I_{\delta}(x_0) (in blu) la cui intersezione con A sarebbe costituita dal solo punto x_0, contro l’ipotesi che x_0 sia di accumulazione per A.


Topologia della retta reale estesa.

È importante dedicare un’attenzione specifica alla topologia della retta reale estesa. Questo concetto si riferisce all’insieme costituito dai numeri reali \mathbb{R},ampliato per includere i due estremi -\infty e +\infty.

Definizione 1.19 (numeri reali estesi). L’insieme dei numeri reali estesi è definito come

(8)   \begin{equation*} 			\overline{\mathbb{R}} 			\coloneqq 			\mathbb{R} \cup \{-\infty,+\infty\}. 		\end{equation*}

Il simbolo \infty viene detto infinito e valgono le relazioni d’ordine

(9)   \begin{equation*} 			-\infty 			< 			x 			< 			+\infty 			\qquad 			\forall x \in \mathbb{R}. 		\end{equation*}

    \[\quad\]

Introduciamo i simboli -\infty e +\infty in modo intuitivo, piuttosto che attraverso un approccio formale e rigoroso. L’idea principale è quella di estendere l’insieme dei numeri reali \mathbb{R} aggiungendo due elementi: -\infty, che rappresenta un numero infinitamente negativo, e +\infty, un numero infinitamente positivo. In altre parole, -\infty è più piccolo di qualsiasi numero reale, mentre +\infty è più grande.

Nei prossimi capitoli vedremo che è possibile eseguire operazioni aritmetiche ordinarie (con alcune restrizioni) anche nell’ambito di questi numeri reali estesi. Questo argomento è strettamente collegato alla teoria dei limiti, ma approfondiremo i dettagli in seguito.

Per ora, ci concentriamo sul modo in cui i concetti di intorno e punto di accumulazione vengono estesi a \overline{\mathbb{R}}. Considerando +\infty come un numero infinitamente grande all’estrema destra della retta reale, possiamo pensare a un intorno come una semiretta a destra. Questo ragionamento ci porta alla seguente definizione.

Definizione 1.20 (intorni dell’infinito). Un insieme A \subseteq \mathbb{R} si dice intorno di +\infty se esso contiene una semiretta destra aperta, ossia se esiste a \in \mathbb{R} tale che

(10)   \begin{equation*} 			(a,+\infty) \subseteq A. 		\end{equation*}

Analogamente, A si dice intorno di -\infty se esso contiene una semiretta sinistra aperta, cioè se esiste a \in \mathbb{R} tale che

(11)   \begin{equation*} 			(-\infty,a) \subseteq A. 		\end{equation*}

    \[\quad\]

Una volta definiti gli intorni di +\infty e -\infty, è naturale estendere la definizione di punto di accumulazione anche al caso +\infty e -\infty. Ricordiamo che, se A \subseteq \mathbb{R} e x_0 \in \mathbb{R}, allora x_0 è detto di accumulazione per A se ogni suo intorno contiene dei punti di A diversi da x_0. Questa definizione può essere “tradotta” anche nel caso x_0=\pm \infty, ovviamente tenendo conto di quali siano gli intorni di questi punti.

Definizione 1.21 (punti di accumulazione in \overline{\mathbb{R}}). +\infty si dice di accumulazione per A \subseteq \mathbb{R} se A è illimitato superiormente, ossia se

(12)   \begin{equation*} 			A \cap (a,+\infty) \neq \emptyset 			\qquad 			\forall a \in \mathbb{R}. 		\end{equation*}

Analogamente, -\infty si dice di accumulazione per A \subseteq \mathbb{R} se A è illimitato inferiormente, ovvero se

(13)   \begin{equation*} 			A \cap (-\infty,a) \neq \emptyset 			\qquad 			\forall a \in \mathbb{R}. \end{equation*}

    \[\quad\]

Questa definizione deriva dal fatto che le semirette destre (a,+\infty) sono gli intorni di +\infty, e le semirette sinistre (-\infty, a) quelli di -\infty. Pertanto, +\infty è un punto di accumulazione per un insieme A se ogni suo intorno include punti appartenenti ad A, analogamente a quanto avviene per i punti di accumulazione in \mathbb{R}. Esaminiamo ora un esempio pratico di questi concetti.

Esempio 1.22. L’insieme A=\{-1\} \cup (1,2) \cup \mathbb{N}, che nella figura 4 è mostrato in blu, ha +\infty come punto di accumulazione. Questo si deve al fatto che A è illimitato superiormente, includendo \mathbb{N}. Come illustrato nella figura (dove le semirette destre sono in verde), ogni semiretta destra contiene infiniti punti di A.

In contrasto, -\infty non è un punto di accumulazione per A. Ciò è evidente osservando che la semiretta sinistra (-\infty,-2), rappresentata in rosso nella figura, non include alcun punto di A.

    \[\quad\]

    \[\quad\]

Teoria sulle successioni

Figura 4: l’insieme A dell’esempio 1.22. Esso è illimitato superiormente e quindi +\infty è un suo punto di accumulazione. Invece, poiché A è limitato inferiormente, -\infty non è un suo punto di accumulazione.


 

Successioni a valori in \mathbb{R}

Introduzione.

Definizione 2.1 (successione). Una successione \{a_n\}_{n \in \mathbb{N}} a valori reali è una funzione

    \[\begin{aligned} 			a \colon \mathbb{N} 	& 	\to \mathbb{R}\\ 			n 					& 	\mapsto a_n=a(n) 		\end{aligned}\]

Solitamente, l’immagine a(n) viene indicata col simbolo a_n ed è detta termine n-esimo della successione. Quando non vi sia possibilità di equivoco, una successione si denoterà semplicemente indicando il suo termine generale a_n.

    \[\quad\]

Una successione è quindi, come il nome stesso suggerisce, una sorta di “lista” di numeri reali. Infatti, possiamo immaginare di ordinare i suoi elementi come

(14)   \begin{equation*} 	a_1,\,a_2,\,a_3,\,\dots, \, a_n,\dots \end{equation*}

Possiamo rappresentare la successione nel piano cartesiano, come qualunque funzione. Facciamo qualche esempio per chiarire il concetto.

Esempio 2.2. La funzione rappresentata in figura 5

    \begin{equation*} 		\begin{split} 			f\colon&\mathbb{N}\rightarrow \mathbb{N}\subset\mathbb{R}\\&n\mapsto n+1 		\end{split} 	\end{equation*}

è una successione il cui termine n-esimo è a_n=n+1.

    \[\quad\]

    \[\quad\]

Teoria sulle successioni

Figura 5: la successione a dell’esempio 2.2.

    \[\quad\]

Esempio 2.3. La funzione rappresentata in figura 6

    \begin{equation*} 		\begin{split} 			f\colon\mathbb{N}&\setminus\{0\}\longrightarrow \mathbb{Q}\subset\mathbb{R}\\&n\longmapsto \dfrac{1}{n} 		\end{split} 	\end{equation*}

è una successione il cui termine n-esimo è a_n=\frac{1}{n}.

    \[\quad\]

    \[\quad\]

Teoria sulle successioni

Figura 6: la successione a dell’esempio 2.3.

    \[\quad\]

Esempio 2.4. La funzione

    \begin{equation*} 		\begin{split} 			f:&\mathbb{N}\longrightarrow \mathbb{N}\subset\mathbb{R}\\&n\longrightarrow n! 		\end{split} 	\end{equation*}

è una successione il cui termine n-esimo è a_n=n!=n\cdot (n-1)\cdot\ldots\cdot 2\cdot1.

Esempio 2.5. La funzione S\colon \mathbb{N}\to\mathbb{R} che associa al numero naturale n la somma S_n dei primi n quadrati perfetti, definita cioè da

    \begin{equation*} 		S_n=0+1+4+9+\dots+n^2=\sum_{k=0}^{n}k^2=\frac{n(n+1)(2n+1)}{6}\qquad\forall n\in\mathbb{N} 	\end{equation*}

è una successione a valori in \mathbb{R}. L’uguaglianza \sum_{k=0}^{n}k^2=\frac{n(n+1)(2n+1)}{6} si può mostrare per induzione.


Limitatezza e monotonia.

Come qualunque funzione, anche per le successioni hanno senso i concetti di limitatezza e di monotonia. Rimandiamo a teoria sulle funzioni, sezioni 2.5 e 2.6 per una trattazione generale di queste nozioni. Qui ci limitiamo a osservare che è naturale studiare il comportamento dei termini a_n di una successione al variare dell’indice n \in \mathbb{N}, ponendosi le seguenti domande:

    \[\quad\]

  • Il termine generale a_n è crescente o decrescente al crescere di n?
  •  

  • Il termine generale a_n rimane limitato (superiormente o inferiormente) al variare di n?

Definizione 2.6 (successioni limitate). Sia \{a_n\}_{n\in\mathbb{N}} una successione a valori in \mathbb{R}. Diremo che

    \[\quad\]

  • \{a_n\}_{n\in\mathbb{N}} è limitata superiormente se

        \begin{equation*} 				\exists M\in\mathbb{R}\,\colon\, a_n\leq M\qquad\forall n\in\mathbb{N}, 			\end{equation*}

  •  

  • \{a_n\}_{n\in\mathbb{N}} è limitata inferiormente se

        \begin{equation*} 				\exists m\in\mathbb{R}\,\colon\, a_n\geq m\qquad\forall n\in\mathbb{N}, 			\end{equation*}

  •  

  • \{a_n\}_{n\in\mathbb{N}} è limitata se è limitata inferiormente e superiormente, ovvero se

        \begin{equation*} 				\exists M\in\mathbb{R}\,\colon\,|a_n|\leq M\qquad\forall n\in\mathbb{N}. 			\end{equation*}

Osservazione 2.7 (limitatezza e grafici). Per illustrare graficamente il concetto di una successione limitata o illimitata, possiamo considerare che una successione è rappresentabile come una funzione. In questa rappresentazione, l’indice n \in \mathbb{N} è posizionato sull’asse delle ascisse, mentre il valore corrispondente del termine generale a_n è sull’asse delle ordinate. Una successione è limitata se il suo grafico è contenuto all’interno di una striscia orizzontale di larghezza finita, come mostrato nella figura 7.

    \[\quad\]

    \[\quad\]

Teoria sulle successioni

Figura 7: esemplificazione dell’osservazione 2.7. La successione dell’esempio 2.2 (a sinistra) definita da a_n=n+1 è limitata inferiormente ma illimitata superiormente in quanto il suo grafico è contenuto nel semipiano superiore (ombreggiato in blu) delimitato dalla retta y=1 . Invece la successione dell’esempio 2.3 (a destra) definita da a_n=\frac{1}{n} è limitata in quanto il suo grafico è contenuto nella striscia (ombreggiata in blu) delimitata dalle rette di equazione y=0 e y=1.

    \[\quad\]

    \[\quad\]

Introduciamo il concetto di monotonia delle successioni, una nozione che determina se il termine generale a_n cresce o decresce all’aumentare dell’indice n.

Definizione 2.8 (successioni monotone). Sia \{a_n\}_{n\in\mathbb{N}} una successione a valori in \mathbb{R}. Diremo che

    \[\quad\]

  • \{a_n\}_{n\in\mathbb{N}} è crescente se a_{n+1}\geq a_n\qquad\forall n\in\mathbb{N}.
  •  

  • \{a_n\}_{n\in\mathbb{N}} è strettamente crescente se a_{n+1}> a_n\qquad\forall n\in\mathbb{N}.
  •  

  • \{a_n\}_{n\in\mathbb{N}} è decrescente se a_{n+1}\leq a_n\qquad\forall n\in\mathbb{N}.
  •  

  • \{a_n\}_{n\in\mathbb{N}} è strettamente decrescente se a_{n+1}< a_n\qquad\forall n\in\mathbb{N}.

La successione \{a_n\}_{n\in\mathbb{N}} è detta monotona se rientra in uno dei casi precedenti.

    \[\quad\]

Facciamo degli esempi per chiarire i concetti esposti.

Esempio 2.9. La successione dell’esempio 2.3 è limitata. Infatti essa soddisfa le seguenti proprietà.

    \[\quad\]

  • \{a_n\}_{n\in\mathbb{N}} è limitata inferiormente: esiste m=0 tale che

        \begin{equation*} 			a_n\geq 0\qquad \forall n\in\mathbb{N}. 		\end{equation*}

  •  

  • \{a_n\}_{n\in\mathbb{N}} è limitata superiormente: scegliendo M=1 si ha

        \begin{equation*} 			a_n\leq 1\qquad \forall  n\in\mathbb{N}. 		\end{equation*}

Inoltre la successione è decrescente, infatti

    \begin{equation*} 		a_{n+1}< a_n\Longleftrightarrow \frac{1}{n+1}< \frac{1}{n}\Longleftrightarrow n+1> n\Longleftrightarrow 1> 0. 	\end{equation*}

Osservazione 2.10. legami tra monotonia e limitatezza). È interessante studiare i legami tra i concetti di monotonia e limitatezza di successioni.

    \[\quad\]

  • Una successione crescente è limitata inferiormente. Infatti, se a_n è crescente, si ha

    (15)   \begin{equation*} 			a_n \geq a_0 			\qquad 			\forall n \in \mathbb{N}_0. 		\end{equation*}

    Analogamente si vede che una successione decrescente è limitata superiormente.

  •  

  • D’altra parte, esistono esempi di successioni crescenti limitate superiormente ed esempi di successioni crescenti illimitate superiormente. Infatti, la successione

    (16)   \begin{equation*} 			a_n=1-\frac{1}{n} 			\qquad 			\forall n \in \mathbb{N}_0 		\end{equation*}

    è crescente ed è limitata superiormente, poichè a_n \leq 1 per ogni n \in \mathbb{N}. Invece la successione dell’esempio 2.2 è crescente ed è illimitata superiormente. Analogamente si vede che una successione decrescente può essere limitata o illimitata inferiormente.

Esempio 2.10. La successione dell’esempio 2.4 è crescente, infatti

    \begin{equation*} 		a_{n+1}=(n+1)!=(n+1)\cdot n!> n!=a_n\,\qquad\forall n\in\mathbb{N} 	\end{equation*}

e questo ci permette di concludere che è limitata inferiormente per l’osservazione 2.10. Infatti per m=1 si ha che

    \begin{equation*} 		a_n\geq 1=0!=a_0\qquad\forall n\in\mathbb{N}. 	\end{equation*}

Possiamo ulteriormente osservare che la successione non è limitata superiormente, infatti

    \begin{equation*} 		n!\geq n\qquad\forall n\in\mathbb{N} 	\end{equation*}

e, dato che \mathbb{N} è illimitato superiormente, tale è anche la successione a_n.

Esempio 2.12. Consideriamo la successione a_n=(-1)^n, rappresentata in figura 8. Essa è limitata in quanto

(17)   \begin{equation*} 		-1 \leq a_n \leq 1 		\qquad 		\forall n \in \mathbb{N}. 	\end{equation*}

La successione a_n non è monotona, in quanto ad esempio per ogni n pari si ha a_n > a_{n-1} e a_{n+1} < a_n.

    \[\quad\]

    \[\quad\]

Teoria sulle successioni

Figura 8: la successione a_n=(-1)^n dell’esempio 2.12. Si vede che essa è limitata, in quanto il suo grafico è contenuto nella striscia delimitata dalle rette y=-1 e y=1. Essa non è però monotona.

    \[\quad\]

    \[\quad\]

Osservazione 2.13. La limitatezza o l’illimitatezza di una successione in generale non forniscono alcuna informazione sulla sua monotonia. Infatti, esistono successioni limitate che non sono monotone, come quella dell’esempio 2.12.


Proprietà verificate definitivamente e frequentemente.

Alcune proprietà di una successione possono non essere verificate per tutti i termini della successione. Ciononostante, risulta molto utile stabilire dei termini per indicare i seguenti casi:

    \[\quad\]

  • La proprietà è vera per tutti i termini della successione, a partire da un certo indice N in poi.
  •  

  • La proprietà è vera per indici n arbitrariamente grandi.

Facciamo qualche esempio per chiarire i concetti.

Esempio 2.14. Data la successione a_n=n+1 dell’esempio 2.2, non tutti i termini della successione sono maggiori di 100, ma lo diventano tutti per ogni n \geq 100. Infatti, se n \geq 100 allora a_n=n+1 \geq 101. Diremo che il termine generale a_n è definitivamente maggiore di 100.

Riguardo il secondo caso, si vede che non tutti i termini della successione a_n sono dispari, ma che vi sono infiniti indici n per cui questo succede: quelli della forma n=2k con k \in \mathbb{N}. Infatti, se n è pari, allora a_n=n+1 è dispari. Gli indici n per cui ciò accade sono arbitrariamente grandi. Diremo quindi che il termine generale a_n è frequentemente dispari. In altre parole, la proprietà di essere dispari non è definitivamente falsa.

Diamo ora la definizione formale di queste nozioni.

Definizione 2.15 (proprietà definitivamente e frequentemente vere). Sia a_n un successione reale e sia \mathcal{P} una proprietà.

    \[\quad\]

  • Diremo che \mathcal{P} è definitivamente vera se esiste N \in \mathbb{N} tale che a_n soddisfa \mathcal{P} per ogni n \geq N.
  •  

  • Diremo che \mathcal{P} è frequentemente vera se non è definitivamente falsa, ossia se per ogni N \in \mathbb{N} esiste n \geq N tale che a_n è vera.

    \[\quad\]

Esempio 2.16. Il termine generale a_n della successione dell’esempio 2.12 è frequentemente positivo. Infatti, poiché a_n=(-1)^n esso è positivo per ogni n pari. Poiché per ogni N \in \mathbb{N} esiste un numero n pari tale che n \geq N, a_n è frequentemente positivo.

a_n è anche definitivamente minore di 2, in quanto a_n < 2 per ogni n \in \mathbb{N}.

Esempio 2.17. Il termine generale a_n=\frac{1}{n} della successione dell’esempio 2.3 appartiene definitivamente all’intervallo I \coloneqq \left(-\dfrac{1}{10},\dfrac{1}{10}\right). Infatti

(18)   \begin{equation*} 		0 < a_n= \dfrac{1}{n} \leq \dfrac{1}{11} < \frac{1}{10} 		\qquad 		\forall n \geq 11. 	\end{equation*}

In altre parole, non tutti gli a_n appartengono a I, ma vi appartengono se n \geq 11.


Sottosuccessioni.

Data una successione a_n, l’idea di sottosuccessione corrisponde intuitivamente a quella di selezionarne solo alcuni termini in modo da ottenere una nuova successione, che viene detta appunto estratta da a_n.

Definizione 2.18. Sia \{a_n\}_{n\in\mathbb{N}} una successione a valori in \mathbb{R} e \phi\colon\mathbb{N}\to\mathbb{N} una funzione strettamente crescente. La sottosuccessione estratta \{a_{\phi(k)}\}_{k\in\mathbb{N}} da \{a_n\}_{n\in\mathbb{N}} è la funzione composta a\circ\phi\colon\mathbb{N}\to\mathbb{R}, dove a\colon\mathbb{N}\longrightarrow\mathbb{R} è la funzione che definisce la successione \{a_n\}_{n\in\mathbb{N}}.1

    \[\quad\]

Quando non ci sono possibilità di ambiguità, useremo la notazione n_k=\phi(k) e quindi la sottosuccessione sarà indicata come {a_{n_k}}.

Esempio 2.19. Consideriamo la successione di termine n-esimo

    \begin{equation*} 		a_n=\frac{(-1)^n}{n}\qquad\forall n\in\mathbb{N} 	\end{equation*}

e le due funzioni strettamente crescenti

(19)   \begin{equation*} \begin{array}{lll} 	\phi\colon\mathbb{N}\longrightarrow \mathbb{N} & & \xi\colon \mathbb{N}\longrightarrow \mathbb{N}$\\ 		k \longmapsto \phi(k)=2k 			& & 			i\longmapsto \xi(i)=2i+1.	 		\end{array} 	\end{equation*}

Allora le successioni in figura 9

(20)   \begin{equation*} \begin{split} 			$\displaystyle b_k:=a_{\phi(k)}=\frac{(-1)^{2k}}{2k}=\frac{1}{2k}$  			& & 			$\displaystyle c_i:=a_{\xi(k)}=\frac{(-1)^{2k+1}}{2k+1}=-\frac{1}{2k+1}$\\ 		\end{split} 	\end{equation*}

sono estratte dalla successione \{a_n\}_{n\in\mathbb{N}} ottenute considerando rispettivamente i termini di indice pari e i termini di indice dispari della successione stessa come in figura 9.

    \[\quad\]

    \[\quad\]

Teoria sulle successioni

Figura 9: dalla successione a_n dell’esempio 2.19 vengono estratte due sottosuccessioni. La sottosuccessione a_{\phi(n)} è costituita dai termini di indice pari (rappresentati in blu), mentre la sottosuccessione a_{\xi(n)} è costituita dai termini di indice dispari (rappresentati in verde).

    \[\quad\]

    \[\quad\]

Osservazione 2.20. Se una successione a_n verifica definitivamente una proprietà, essa è verificata definitivamente da ogni sottosuccessione a_{n_k}. Infatti, se esiste N \in \mathbb{N} tale che \mathcal{P}(a_n) è vera per ogni n \geq N, poiché per il lemma 3.15, che dimostreremo nel paragrafo successivo, vale

(21)   \begin{equation*} 		n_k 		\geq 		k 		\qquad 		\forall k \in \mathbb{N}, 	\end{equation*}

allora n_k \geq N per ogni k \geq N, quindi \mathcal{P}(a_{n_k}) è vera per ogni k \geq N.

Introduciamo la seguente proprietà, che sarà di grande utilità nelle prossime sezioni e che collega l’esistenza di sottosuccessioni a proprietà verificate frequentemente.

Proposizione 2.21. Data una successione \{a_n\}_n, una proprietà \mathcal{P} è frequentemente vera se e solo se esiste una sottosuccessione \{a_{n_k}\}_k per cui \mathcal{P} è vera per ogni k \in \mathbb{N}.

    \[\quad\]

Dimostrazione. Se \mathcal{P} è frequentemente vera, definiamo una funzione \phi \colon \mathbb{N} \to \mathbb{N} per ricorrenza, ovvero poniamo \phi(1)=n_1=\min\{n \in \mathbb{N} \colon a_n \text{ è vera}\}. Una volta definita \phi(k), definiamo

(22)   \begin{equation*} 			\phi(k+1) 			= 			n_{k+1} 			= 			\min \{n > n_k \colon a_n \text{ è vera} \}. 		\end{equation*}

Poiché \mathcal{P} è frequentemente vera, l’insieme in (22) non è vuoto, pertanto n_{k+1} è ben definito. Chiaramente \{a_{n_k}\}_{k \in \mathbb{N}} è una sottosuccessione di a_n per cui \mathcal{P} è sempre vera.

Viceversa se esiste una sottosuccessione \{a_{n_k}\}_{k \in \mathbb{N}} di a_n per cui \mathcal{P} è sempre vera, fissiamo N \in \mathbb{N}. Poiché la funzione k \mapsto n_k è a valori in \mathbb{N} ed è strettamente crescente (per definizione di sottosuccessione), la sua immagine è illimitata, quindi esiste k \in \mathbb{N} tale che n_k >N e inoltre il termine a_{n_k} soddisfa la proprietà \mathcal{P}. Per l’arbitrarietà di N, \mathcal{P} è frequentemente vera.

Studiamo ora come si comportano le nozioni studiate in precedenza rispetto al concetto di sottosuccessione. Valgono le seguenti osservazioni.

    \[\quad\]

  • Una successione \{a_n\}_{n \in \mathbb{N}} è limitata se e solo se tutte le sue sottosuccessioni sono limitate. D’altra parte, se un’estratta \{a_{n_k}\}_{k \in \mathbb{N}} della successione a_n è limitata, non è detto che a_n lo sia, in quanto potrebbero esserci dei termini non appartenenti a \{a_{n_k}\}_{k \in \mathbb{N}} non uniformemente limitati.
  •  

  • Una successione a_n è monotona se e solo se tutte le sue estratte possiedono la stessa monotonia. D’altra parte, una successione a_n non monotona può avere anche estratte monotone, come nell’esempio 2.19.

Esempio 2.22. Si consideri la successione \{a_n\}_{n \in \mathbb{N}} definita da

(23)   \begin{equation*} 		a_n= 		\begin{cases} 			k			& \text{se } n=3k-2 \text{ con } k \in \mathbb{N}\\ 			-k		& \text{se } n=3k-1\text{ con } k \in \mathbb{N}\\ 			0			& \text{se } n=3k \text{ con } k \in. \mathbb{N} 		\end{cases} 	\end{equation*}

Questa successione chiaramente non è limitata, né inferiormente né superiormente, e non è monotona. Tuttavia, consideriamo le sottosuccessioni \{a_{\phi(n)}\}_{n \in \mathbb{N}}, \{a_{\xi(n)}\}_{n \in \mathbb{N}} e \{a_{\zeta(n)}\}_{n \in \mathbb{N}} definite da

(24)   \begin{equation*} 		\phi(n)=3n-2, 		\quad 		\xi(n)=3n-1, 		\quad 		\zeta(n)=3n 		\qquad 		\forall n \in \mathbb{N}. 	\end{equation*}

Esse sono rappresentate rispettivamente in blu, in verde e in rosso in figura 10 e soddisfano le seguenti proprietà:

    \[\quad\]

  • \{a_{\phi(n)}\}_{n \in \mathbb{N}} è strettamente crescente, limitata inferiormente e illimitata superiormente. Osserviamo che, definendo la successione b_n=a_{\phi(n)}, si ha b_n=n.
  •  

  • \{a_{\xi(n)}\}_{n \in \mathbb{N}} è strettamente decrescente, limitata superiormente e illimitata inferiormente. Osserviamo che, definendo la successione c_n=a_{\xi(n)}, si ha b_n=-n.
  •  

  • \{a_{\zeta(n)}\}_{n \in \mathbb{N}} è costante e quindi ovviamente limitata.

    \[\quad\]

    \[\quad\]

Teoria sulle successioni

Figura 10: la successione a_n dell’esempio 2.22.

    \[\quad\]

    \[\quad\]

   


  1. La funzione composta è il risultato dell’operazione di composizione di due funzioni. Formalmente, date due funzioni

        \begin{equation*} 				f\colon A\longrightarrow B\qquad g\colon C\longrightarrow D 			\end{equation*}

    tale che B\subseteq C, definiamo la funzione composta come

        \begin{equation*} 				\begin{split} 					g\circ f\colon&A\longrightarrow D\\&x\mapsto (g\circ f)(x)=g(f(x)). 				\end{split} 		\end{equation*}

    Per approfondire l’argomento si rimanda a teoria sulle funzioni.


 

Limiti di successioni

Introduzione.

La nozione di limite è forse la più centrale e importante dell’Analisi Matematica. Essa formalizza l’idea intuitiva di “valore a cui si avvicina un certo oggetto”. Infatti, dagli esempi che abbiamo esaminato, risulta naturale porsi la seguente domanda.

Domanda 3.1. Verso quale (o quali) valore si avvicinano i termini a_n della successione al crescere di n?

Analizziamo gli esempi precedenti in questa ottica per cercare di rispondere alla domanda.

    \[\quad\]

  • Nell’esempio 2.2, il termine a_n=n+1 non sembra avvicinarsi ad alcun valore, anzi, il termine generale sembra diventare più grande di qualsiasi numero reale e avvicinarsi a +\infty. Infatti, considerando un qualunque intorno (a,+\infty) di +\infty, da un certo N in poi a_n appartiene a (a,+\infty). Vedremo che questa idea si formalizza dicendo che \displaystyle\lim_{n\to + \infty} a_n=+\infty oppure che “a_n diverge a +\infty”.
  •  

  • La successione a_n= \frac{1}{n} dell’esempio 2.3 è decrescente e il termine generale diventa via via più piccolo al crescere di n. Sembra proprio che a_n si avvicini al valore 0, quando n sia molto grande. Infatti, qualunque intorno I_\varepsilon(0) si consideri, da un certo N in poi, a_n appartiene a I_\varepsilon(0). Formalizzeremo questa idea scrivendo \displaystyle\lim_{n \to + \infty} a_n=0 oppure che “a_n è convergente a 0”.
  •  

  • La successione a_n=(-1)^n dell’esempio 2.12 oscilla continuamente tra i valori -1 e 1 e non sembra avvicinarsi definitivamente a nessun numero reale. Formalmente, diremo che la successione a_n non ha limite oppure che \displaystyle\lim_{n \to +\infty}a_n non esiste.

Motivati da queste considerazioni di carattere intuitivo, diamo la definizione di limite di una successione, illustrato in figura 11.

Definizione 3.2 (limiti di successioni). Sia a_n una successione reale e sia \ell \in \overline{\mathbb{R}}. Si dice che a_n ha limite \ell oppure tende a \ell se, per ogni intorno I di \ell, esiste N \in \mathbb{N} tale che

(25)   \begin{equation*} 			a_n \in I 			\qquad 			\forall n \geq N. 		\end{equation*}

In tal caso si scrive

(26)   \begin{equation*} 			\lim_{n \to + \infty} a_n=\ell 			\qquad 			\text{oppure} 			\qquad 			a_n \xrightarrow[n \to + \infty]{} \ell 			\qquad 			\text{oppure} 			\qquad 			a_n \to \ell. 		\end{equation*}

    \[\quad\]

  1. Se \ell \in \mathbb{R}, a_n si dice convergente a \ell. Se \ell=0, a_n si dice anche infinitesima.
  2.  

  3. Se \ell \in \{-\infty,+\infty\}, a_n si dice divergente a \ell.

Le successioni convergenti o divergenti si dicono regolari, mentre le successioni che non ammettono limite si dicono non regolari.

    \[\quad\]

    \[\quad\]

Teoria sulle successioni

Figura 11: illustrazione del concetto di limite di una successione nel caso in cui \ell \in \mathbb{R}. Fissando un qualunque intorno circolare I_\varepsilon(\ell) = (\ell-\varepsilon,\ell+\varepsilon) di \ell, i termini della successione a_n vi appartengono per ogni n \geq N, con N opportuno; in questo caso N=6.

    \[\quad\]

    \[\quad\]

Osservazione 3.3. La definizione di limite formalizza l’idea intuitiva dell’avvicinarsi di a_n a \ell: si consideri un intorno I arbitrariamente piccolo di \ell; allora, da un certo N in poi (che naturalmente dipende dalla dimensione di I), tutti i termini di a_n appartengono a I.

Prima di fornire alcuni esempi, osserviamo che la nozione di limite può essere formulata in varie maniere equivalenti, che saranno utilizzate indistintamente nel seguito come definizioni di limite.

Proposizione 3.4 (definizioni equivalenti di limite: \ell \in \mathbb{R}). Sia \{a_n\}_{n \in \mathbb{N}} una successione e sia \ell \in \mathbb{R}. Allora le seguenti affermazioni sono equivalenti:

  1. \displaystyle \lim_{n \to +\infty} a_n = \ell;
  2. Per ogni \varepsilon>0, esiste N \in \mathbb{N} tale che

    (27)   \begin{equation*} 				a_n \in I_\varepsilon(\ell) = (\ell - \varepsilon,\ell+\varepsilon) 				\qquad 				\forall n \geq N; 			\end{equation*}

  3. Per ogni \varepsilon>0, esiste N \in \mathbb{N} tale che

    (28)   \begin{equation*} 				|a_n - \ell| < \varepsilon 				\qquad 				\forall n \geq N; 			\end{equation*}

  4. Per ogni \varepsilon>0, definitivamente si ha |a_n - \ell|<\varepsilon.

    \[\quad\]

Dimostrazione. Mostriamo che ognuna delle condizioni 1, 2, 3, 4 implica le altre.

1 \Rightarrow 2 Sia \varepsilon>0; allora I_\varepsilon(\ell) è un intorno di \ell e, per la definizione 3.2, esiste N \in \mathbb{N} tale che a_n \in I_\varepsilon(\ell) per ogni n \geq N.

2 \Rightarrow 3 Fissato \varepsilon>0, per 2 esiste N \in \mathbb{N} tale che a_n \in I_\varepsilon(\ell) per ogni n \geq N. Si ha che a_n \in I_\varepsilon(\ell) se e solo se la distanza tra a_n e \ell è minore di \varepsilon, cioè se e solo se |a_n - \ell|< \varepsilon, che è quindi vero per ogni n \geq N.

3 \Rightarrow 4 per definizione di proprietà vera definitivamente, 3 implica 4.

4 \Rightarrow 1 Si fissi un intorno I di \ell. Per definizione di intorno, esiste \varepsilon>0 tale che I_{\varepsilon}(\ell) \subseteq I. Poiché |a_n-\ell|<\varepsilon definitivamente, esiste N \in \mathbb{N} tale che

(29)   \begin{equation*} 			|a_n-\ell|< \varepsilon 			\qquad 			\forall n \geq N. 		\end{equation*}

Per definizione di I_\varepsilon(\ell)=(\ell-\varepsilon,\ell+\varepsilon), si ha quindi

(30)   \begin{equation*} 			a_n \in I_\varepsilon(\ell) \subseteq I 			\qquad 			\forall n \geq N, 		\end{equation*}

ossia la 1.

Nei casi \ell\in \{-\infty, +\infty\}, vale un’analoga caratterizzazione della definizione di limite. La scriviamo esplicitamente solo per \ell=+\infty, ma il lettore può esercitarsi a scrivere le semplici controparti per \ell=-\infty. Non dimostriamo la proposizione, in quanto la dimostrazione è analoga a quella della proposizione 3.4.

Proposizione 3.5 (definizioni equivalenti di limite: \ell=+\infty). Sia \{a_n\}_{n \in \mathbb{N}} una successione. Le seguenti affermazioni sono equivalenti:

    \[\quad\]

  1. \displaystyle \lim_{n \to +\infty} a_n = +\infty;
  2.  

  3. Per ogni M>0, esiste N \in \mathbb{N} tale che

    (31)   \begin{equation*} 				a_n \in (M,+\infty) 				\qquad 				\forall n \geq N; 			\end{equation*}

  4.  

  5. Per ogni M>0, esiste N \in \mathbb{N} tale che

    (32)   \begin{equation*} 				a_n > M 				\qquad 				\forall n \geq N; 			\end{equation*}

  6.  

  7. Per ogni \varepsilon>0, definitivamente si ha a_n > M.

Esempio 3.6. Utilizzando la definizione equivalente di limite indicata nel punto 3 della proposizione 3.4, dimostriamo quanto segue:

    \begin{equation*} 	\lim_{n\to+\infty}\frac{1}{n}=0. \end{equation*}

Dato \varepsilon>0, si ha

    \begin{equation*} 	\Big\vert\frac{1}{n}-0\Big\vert<\varepsilon \quad \Longleftrightarrow \quad \frac{1}{n}<\varepsilon \quad \Longleftrightarrow \quad n>\frac{1}{\varepsilon}. \end{equation*}

Scegliendo ora un qualsiasi N tale che N>\frac{1}{\varepsilon}, possiamo concludere che per ogni n\geq N si ha |a_n-0|<\varepsilon, dimostrando così la tesi.

    \[\quad\]

    \[\quad\]

Figura 12: rappresentazione dell’esempio 3.6. Per ogni n \geq 6, a_n appartiene all’intervallo (-\varepsilon,+\varepsilon), ovvero i punti di coordinate (n,a_n) (rappresentati in blu) appartengono alla striscia orizzontale (ombreggiata in verde) delimitata dalle rette di equazione y=-\varepsilon e y=+\varepsilon.

    \[\quad\]

Esempio 3.7. Sfruttando la definizione dimostriamo che

    \begin{equation*} 		\lim_{n\to+\infty}\frac{n}{n+1}=1. 	\end{equation*}

Sia \varepsilon>0; allora

    \begin{equation*} 		\Big\vert\frac{n}{n+1}-1\Big\vert<\varepsilon\Longleftrightarrow\Big\vert-\frac{1}{n+1}\Big\vert<\varepsilon\Longleftrightarrow n+1>\frac{1}{\varepsilon}. 	\end{equation*}

Fissando un qualunque N>\frac{1}{\varepsilon}, per ogni n\geq N si ha che |a_n-1|<\varepsilon, ottenendo quindi la tesi.

Esempio 3.8. Verifichiamo, utilizzando la definizione, che

    \begin{equation*} 	\lim_{n\to+\infty} \frac{n^2+4}{2n^2+3} = \frac{1}{2}. \end{equation*}

Sia \varepsilon>0; allora

    \begin{equation*} 	\left|\frac{n^2+4}{2n^2+3}-\frac{1}{2}\right|=\left|\frac{2n^2+8-2n^2-3}{2(2n^2+3)}\right|=\left|\frac{5}{4n^2+6}\right|<\varepsilon\Longleftrightarrow\, 4n^2+6>\frac{5}{\varepsilon}\Longleftarrow\, n>\sqrt{\frac{5}{4\varepsilon}-\frac{3}{2}}. \end{equation*}

Fissando un qualunque N>\sqrt{\frac{5}{4\varepsilon}-\frac{3}{2}} (quantità reale solo nel caso in cui \varepsilon sia sufficientemente piccolo) per ogni n\geq N si ha che |a_n-\frac{1}{2}|<\varepsilon, ottenendo quindi la tesi.

Esempio 3.9. Sia a_n la successione definita da a_n=2^n per ogni n \in \mathbb{N}. Mostriamo che a_n \to + \infty. Infatti, si ha

(33)   \begin{equation*} 		2^n 		= 		(1 + 1)^n 		\geq 		1 + n 		\qquad 		\forall n \in \mathbb{N}, 	\end{equation*}

dove (33) segue dalla disuguaglianza di Bernoulli.2 Sia M \in \mathbb{R} e sia N \in \mathbb{N} tale che N+1 \geq M. Allora si ha

(34)   \begin{equation*} 		2^n 		\geq n+1 		\geq M 		\qquad 		\forall n \geq N. 	\end{equation*}

Per l’arbitrarietà di M, si ha a_n \to + \infty.

Esempio 3.10. Sfruttando la definizione dimostriamo che

    \begin{equation*} 		\lim_{n\to+\infty}\sqrt{n}=+\infty. 	\end{equation*}

Scelto un valore M>0, risolviamo la disequazione \sqrt{n}>M:

    \begin{equation*} 		\sqrt{n}>M\Longleftrightarrow n>M^2. 	\end{equation*}

Allora esiste N>M^2 tale che

    \begin{equation*} 		a_n>M\qquad\forall n>N. 	\end{equation*}

    \[\quad\]

    \[\quad\]

Figura 13: la successione dell’esempio 3.10. Se n\geq M^2, allora \sqrt{n}\geq M e quindi a_n\geq M. Quindi la successione diverge positivamente.

    \[\quad\]

Esempio 3.11. Sfruttando la definizione dimostriamo che

    \begin{equation*} 		\lim_{n \rightarrow +\infty} (3\sqrt{n}-4n)=-\infty. 	\end{equation*}

Osserviamo che

    \begin{equation*} 		3\sqrt{n}-4n=\sqrt{n}(3-4\sqrt{n})\qquad\forall n\in\mathbb{N}, 	\end{equation*}

e inoltre, limitandoci a considerare n>1, vale

    \[3-4\sqrt{n}<-1.\]

Quindi per M>0 vale

    \begin{equation*} 		3\sqrt{n}-4n=\sqrt{n}(3-4\sqrt{n})<-\sqrt{n}<-M\Longleftrightarrow\, n>M^2. 	\end{equation*}

Scegliendo N>M^2 si ha

    \begin{equation*} 		3\sqrt{n}-4n<-M\qquad\forall n>N. 	\end{equation*}

   


  1. Tale disuguaglianza afferma che

    (35)   \begin{equation*} 			(1+x)^n \geq 1+nx 			\qquad 			\forall x >-1,\,\,\forall n \in \mathbb{N}, 		\end{equation*}

    e può essere dimostrata per induzione.


Prime proprietà dei limiti.

Cerchiamo adesso di rispondere alle possibili domande che possono sorgere a partire da queste prime definizioni, esponendo le prime proprietà dei limiti.

Teorema 3.12 (unicità del limite). Sia \{a_n\}_{n\in\mathbb{N}} una successione tale che

    \begin{equation*} 			\lim_{n\to+\infty}a_n=\ell \in \overline{\mathbb{R}};  		\end{equation*}

allora tale limite è unico.

    \[\quad\]

Dimostrazione. Supponiamo che a_n \to \ell_1 e a_n \to \ell_2 con \ell_1,\ell_2 \in \mathbb{R} \cup \{-\infty,+\infty\}. Siano I_1 un qualsiasi intorno di \ell_1 e I_2 un qualsiasi intorno di \ell_2. Per la definizione di limite 3.2, a_n \in I_1 definitivamente e a_n \in I_2 definitivamente, quindi esiste N \in \mathbb{N} tale che

(36)   \begin{equation*} 		a_n \in I_1 \cap I_2 		\qquad 		\forall n \geq N. 	\end{equation*}

Il punto principale della dimostrazione è provare che, se \ell_1 \neq \ell_2, allora esistono due rispettivi intorni I_1 e I_2 disgiunti, rendendo quindi impossibile (36). Per mostrare l’esistenza di tali I_1,I_2, distinguiamo vari casi. Senza perdita di generalità supporremo sempre \ell_1 < \ell_2.

    \[\quad\]

  • \ell_1,\ell_2 \in \mathbb{R}. Se \ell_1 e \ell_2 sono numeri reali distinti, consideriamo \varepsilon \coloneqq 		\frac{|\ell_1 - \ell_2|}{4} 		= 		\frac{\ell_2-\ell_1}{4} e poniamo

    (37)   \begin{gather*} 			I_1 			= 			I_\varepsilon(\ell_1) 			= 			(\ell_1 - \varepsilon, \ell_1 + \varepsilon), 			%= 			%\left ( \ell_1 - \dfrac{\ell_2-\ell_1}{4} , \ell_1 + \dfrac{\ell_2-\ell_1}{4} \right ) 			\qquad 			I_2 			= 			I_\varepsilon(\ell_2) 			= 			(\ell_2 - \varepsilon, \ell_2 + \varepsilon), 		\end{gather*}

    rappresentati rispettivamente in blu e in verde in figura 14. Poiché

    (38)   \begin{equation*} 			\ell_1 + \varepsilon 			= 			\ell_1 + \dfrac{\ell_2-\ell_1}{4} 			= 			\dfrac{3 \ell_1 + \ell_2}{4} 			< 			\dfrac{3\ell_2 + \ell_1}{4} 			= 			\ell_2 - \dfrac{\ell_2-\ell_1}{4} 			= 			\ell_2 - \varepsilon, 		\end{equation*}

    dove la disuguaglianza segue da \ell_1 < \ell_2, l’estremo destro dell’intervallo I_1 è strettamente minore dell’estremo sinistro dell’intervallo I_2, quindi I_1 \cap I_2= \emptyset.

        \[\quad\]

        \[\quad\]

    Figura 14: rappresentazione del teorema 3.12 nel caso \ell_1,\ell_2 \in \mathbb{R}. Poiché \ell_1 < \ell_2, esistono intorni I_1 e I_2 (in blu e in verde) rispettivamente di \ell_1 e \ell_2 disgiunti. Dato che a_n non può appartenere definitivamente a entrambi gli intervalli, \ell_1 e \ell_2 non possono essere entrambi limiti della successione.

        \[\quad\]

  •  

  • \ell_1 \in \mathbb{R}, \ell_2=+\infty. Sia un qualsiasi \varepsilon>0 e si consideri l’intorno I_1=I_\varepsilon(\ell_1) = (\ell_1-\varepsilon,\ell_1+\varepsilon). Si scelga poi M>\ell_1+\varepsilon e si ponga I_2=(M,+\infty). Ovviamente I_1 e I_2 sono intorni disgiunti rispettivamente di \ell_1 e \ell_2.
  •  

  • \ell_1 =-\infty, \ell_2=+\infty. Si scelga M>0 e si ponga

    (39)   \begin{equation*} 			I_1=(-\infty,-M), 			\qquad 			I_2=(M,+\infty). 		\end{equation*}

    Chiaramente I_1 e I_2 sono intorni rispettivamente di \ell_1 e \ell_2 e I_1 \cap I_2=\emptyset.

  • La nozione di insieme chiuso può essere caratterizzata mediante successioni; vale infatti il seguente risultato.

    Proposizione 3.13 (caratterizzazione dei chiusi per successioni). Sia A \subseteq \mathbb{R}; le seguenti affermazioni sono equivalenti:

    1. A è chiuso;
    2. per ogni successione a_n a valori in A convergente a un numero reale \ell, si ha \ell \in A.

        \[\quad\]

    Osservazione 3.14. È importante sottolineare che la proprietà 2 non implica che ogni successione con valori in A converga. Piuttosto, stabilisce che, nel caso in cui una successione \{a_n\} converga ad un limite \ell, allora tale limite \ell deve appartenere all’insieme A.

    Dimostrazione. Analizziamo separatamente le due implicazioni:

        \[\quad\]

    • 1 \Rightarrow 2. Consideriamo A chiuso e una successione \{a_n\} in A che converge a \ell. Per dimostrare che \ell \notin A^c, supponiamo per assurdo che \ell \in A^c. Poiché A^c è aperto (come stabilito dalla proposizione 1.12), esiste un intorno J di \ell completamente contenuto in A^c. Tuttavia, per la definizione di limite, si avrebbe che:

      (40)   \begin{equation*} 		a_n \in J \subseteq A^c \quad \text{definitivamente}, 	\end{equation*}

      contraddicendo l’ipotesi iniziale che la successione \{a_n\} abbia valori in A.

    •  

    • 2 \Rightarrow 1. Supponendo ora che la proprietà 2 sia valida, vogliamo dimostrare che A^c è aperto, il che implica che A è chiuso (come affermato dalla proposizione 1.12). Se A^c non fosse aperto, esisterebbe un punto x \in A^c che non è interno, ovvero per cui vale:

      (41)   \begin{equation*} 		\exists a_n \in A \cap \left( x-\frac{1}{n},x+ \frac{1}{n} \right ) \quad \forall n \in \mathbb{N}. 	\end{equation*}

      Questa successione \{a_n\}, definita in A, converge a x. Infatti, per \varepsilon>0 arbitrario vale:

      (42)   \begin{equation*} 		\left( x-\frac{1}{n},x+ \frac{1}{n} \right ) \subseteq (x-\varepsilon,x+\varepsilon) \quad \forall n \geq \frac{1}{\varepsilon}, 	\end{equation*}

      e quindi, per (41), si ha che:

      (43)   \begin{equation*} 		a_n \in (x-\varepsilon,x+\varepsilon) \quad \forall n \geq \frac{1}{\varepsilon}. 	\end{equation*}

      L’arbitrarietà di \varepsilon implica che a_n \to x. Questo porta ad una contraddizione con l’ipotesi 2, poiché abbiamo identificato una successione in A che converge ad un punto in A^c. Di conseguenza, deve necessariamente valere la proprietà 1.

    Abbiamo bisogno di un lemma tecnico per dimostrare un teorema che mette in relazione la convergenza di una successione e il comportamento delle sottosuccessioni estratte.

    Lemma 3.15. Per ogni funzione

        \begin{equation*} 			\begin{split} 				&n\colon \mathbb{N}\to\mathbb{N}\\ 				&\,\quad k\mapsto n_k 			\end{split} 		\end{equation*}

    crescente di numeri naturali, si ha che

        \begin{equation*} 			n_k\geq k\qquad\qquad\forall k\in\mathbb{N}. 		\end{equation*}

        \[\quad\]

    Dimostrazione. Dimostriamo l’affermazione per induzione sui termini della successione n_k:

        \[\quad\]

    • Passo base (k=0): si ha n_k\geq 0 perché per ipotesi \{n_k\} è una successione di numeri naturali.
    •  

    • Passo induttivo: Supponiamo che l’affermazione sia vera per un certo k, ovvero n_k \geq k. Dobbiamo dimostrare che essa è valida anche per k+1. Dall’ipotesi induttiva e dal fatto che \{n_k\} è strettamente crescente, abbiamo che

          \begin{equation*} 			n_{k+1} > n_k \geq k \quad\Rightarrow\quad n_{k+1} > k \quad\Rightarrow\quad n_{k+1} \geq k + 1, 		\end{equation*}

      il che completa il passo induttivo e conferma la validità dell’asserzione per ogni k.

    Poiché una sottosuccessione di una successione \{a_n\}_{n\in\mathbb{N}} si ottiene selezionando alcuni termini della successione, se questa ammette limite è ragionevole aspettarsi che anche la sottosuccessione ammetta lo stesso limite.

    Teorema 3.16. ia \{a_n\}_{n \in \mathbb{N}} una successione avente limite \ell \in \overline{\mathbb{R}}. Allora ogni sua estratta ha lo stesso limite.

        \[\quad\]

    Dimostrazione. Consideriamo una sottosuccessione \{a_{n_k}\}_{k \in \mathbb{N}} della successione \{a_n\} e un intorno I del limite \ell.3 Dato che la successione \{a_n\} converge a \ell, possiamo affermare l’esistenza di un N \in \mathbb{N} tale che

    (44)   \begin{equation*} 		a_n \in I \quad \forall n \geq N. 	\end{equation*}

    Applicando il lemma 3.15, sappiamo che n_k \geq k per ogni k \in \mathbb{N}. Questo implica che per ogni k \geq) N, n_k sarà anch’esso maggiore o uguale a N. Di conseguenza, dalla (44), otteniamo

    (45)   \begin{equation*} 		a_{n_k} \in I \quad \forall k \geq N. 	\end{equation*}

    L’arbitrarietà nella scelta dell’intorno I e la proprietà appena dimostrata ci permettono di concludere che la sottosuccessione \{a_{n_k}\} converge effettivamente a \ell.

    Esempio 3.17. Questo teorema è estremamente utile per verificare la non regolarità di una particolare successione. Prendiamo ad esempio a_n=(-1)^n e consideriamo le due sottosuccessioni

        \begin{equation*} 		b_k=(-1)^{2k},\qquad\qquad\qquad c_k=(-1)^{2k+1}. 	\end{equation*}

    Si ha che

        \begin{equation*} 		\lim_{k  \rightarrow + \infty } b_k=1,\qquad\qquad\qquad \lim_{k  \rightarrow + \infty } c_k=-1, 	\end{equation*}

    in quanto b_k=1 e c_k=-1 per ogni k\in\mathbb{N}. Allora possiamo concludere che il limite della successione a_n non esiste, perchè se per assurdo esistesse allora ogni estratta dovrebbe convergere a tale limite.

    Il limite \ell di una successione a_n è quindi il valore verso cui “si accumulano” tutti i valori assunti da a_n. Risulta quindi naturale indagare la relazione tra limiti della successione a_n e punti di accumulazione dell’insieme \{a_n \colon n \in \mathbb{N}\} immagine della successione a_n, esplicitato dalla prossima proposizione.

    Proposizione 3.18 (caratterizzazione dei punti di accumulazione). Sia A \subseteq \mathbb{R} e sia x_0 \in \mathbb{R} \cup \{-\infty,+\infty\}. Allora le seguenti affermazioni sono equivalenti:

    1. x_0 è un punto di accumulazione per A;
    2. esiste una successione \{a_n\}_{n \in \mathbb{N}} a valori in A \setminus \{x_0\} tale che

      (46)   \begin{equation*} 			\lim_{n \to + \infty} a_n 			= 			x_0. 		\end{equation*}

        \[\quad\]

    La dimostrazione della presente proposizione richiede l’applicazione del teorema del confronto 5.8. Si rimanda quindi alla sezione 5.2.

    Teorema 3.19. Una successione \{a_n\}_{n\in\mathbb{N}} convergente è limitata.

        \[\quad\]

    Dimostrazione. Consideriamo una successione \{a_n\}_{n \in \mathbb{N}} che converge a \ell \in \mathbb{R} e sia I_\varepsilon(\ell) = (\ell - \varepsilon, \ell + \varepsilon) un intorno di \ell. In base alla definizione di limite 3.4, esiste N \in \mathbb{N} tale che

    (47)   \begin{equation*} 	|a_n - \ell| < \varepsilon \quad \forall n \geq N. \end{equation*}

    Da ciò, deduciamo che

    (48)   \begin{equation*} 	|a_n| = |a_n - \ell + \ell| \leq |a_n - \ell| + |\ell| < \varepsilon + |\ell| \quad \forall n \geq N. \end{equation*}

    Definiamo poi

    (49)   \begin{equation*} 	M_1 \coloneqq \max\{|a_1|, \ldots, |a_N|\}, \end{equation*}

    che esiste in quanto si tratta di un insieme finito di termini. Ponendo M \coloneqq \max\{|\ell| + \varepsilon, M_1\}, dalle equazioni (48) e (49) segue che

    (50)   \begin{equation*} 	|a_n| \leq M \quad \forall n \in \mathbb{N}, \end{equation*}

    dimostrando che la successione a_n è limitata.

        \[\quad\]

        \[\quad\]

    Figura 15: dimostrazione del teorema 3.19, che per semplicità abbiamo illustrato relativamente a una successione a termini positivi convergente a \ell. Poiché per ogni n \geq 6 si ha a_n \leq \ell+\varepsilon e i termini \{a_1,\dots,a_5\} sono limitati da M_1=a_4, la successione è limitata.

        \[\quad\]

    Esempio 3.20. (successione che non ammette limite). Possiamo dimostrare con un esempio che non vale il viceversa del teorema precedente, ovvero non è vero che una successione limitata è convergente. Consideriamo la successione di termine n-esimo

        \[a_n=(-1)^n\qquad\forall  n\in\mathbb{N},\]

    questa è ovviamente limitata; infatti

        \begin{equation*} 		|a_n|=\left|(-1)^n\right|\leq 1\qquad\forall n\in\mathbb{N}. 	\end{equation*}

    ma come abbiamo visto nell’esempio 3.17 non è convergente.

    Il teorema 3.19 stabilisce che ogni successione convergente è necessariamente limitata. Al contrario, se una successione diverge positivamente, ne consegue che è illimitata superiormente. Tuttavia, adottando un approccio analogo a quello utilizzato nella dimostrazione del teorema 3.19, possiamo dimostrare che tale successione divergente è limitata inferiormente.

    Proposizione 3.21. Sia \{a_n\}_{n \in \mathbb{N}} una successione tale che

    (51)   \begin{equation*} 			\lim_{n\to +\infty} a_n=+\infty, 		\end{equation*}

    allora essa è limitata inferiormente. Analogamente, se

    (52)   \begin{equation*} 			\lim_{n\to +\infty} a_n=-\infty, 		\end{equation*}

    allora essa è limitata superiormente.

        \[\quad\]

    Omettiamo la dimostrazione di questa proposizione, invitando il lettore a considerarla come un esercizio stimolante. La soluzione può essere trovata adattando il ragionamento impiegato nella dimostrazione del teorema 3.19.    


    1. Si ricorda che, se \ell \in \mathbb{R}, I include un intervallo del tipo (\ell - \varepsilon, \ell + \varepsilon) con \varepsilon > 0. Nel caso in cui \ell = +\infty, I comprende una semiretta del tipo (M, +\infty) con M \in \mathbb{R}, e per \ell = -\infty, I include una semiretta (-\infty, M).

 

Le operazioni con i limiti

Leggi...

Valgono le seguenti proprietà di calcolo.

Teorema 4.1 (operazioni con i limiti). Siano \{a_n\}_{n\in\mathbb{N}} e \{b_n\}_{n\in\mathbb{N}} due successioni convergenti

    \begin{equation*} 			\lim_{n\to+\infty}a_n=a,\qquad \lim_{n\to+\infty}b_n=b,\qquad\text{ con } a,\,b\in \mathbb{R}. 		\end{equation*}

Allora

  1. \displaystyle\lim_{n\to+\infty}|a_n|=|a|;
  2. \displaystyle\lim_{n\to+\infty}(a_n\pm b_n)=a\pm b;
  3. \displaystyle\lim_{n\to+\infty}(a_n\cdot b_n)=a\cdot b;
  4. Se \displaystyle b\neq 0 si ha \lim_{n\to+\infty}\frac{a_n}{b_n}=\frac{a}{b}.

    \[\quad\]

Dimostrazione.

  1. Per l’ipotesi di convergenza della successione si ha che per ogni \varepsilon>0

        \begin{equation*} 			\exists N>0\,\,:\,\, |a_n-a|<\varepsilon\qquad\forall n>N. 				\end{equation*}

    Allora, per la disuguaglianza triangolare,

        \begin{equation*} 			||a_n|-|a||\leq |a_n-a|<\varepsilon\qquad\forall n>N. 		\end{equation*}

    quindi possiamo concludere che \displaystyle\lim_{n\to+\infty}|a_n|=|a|.

  2.  

  3. Dimostriamo che \displaystyle\lim_{n\to+\infty}(a_n+b_n)=a+b. Per ipotesi di convergenza delle successioni si ha che per ogni \varepsilon>0

    (53)   \begin{equation*} 			\begin{aligned} 				&\exists n _1>0\,: \, \,\, \left \vert a_n-a\right \vert<\frac{\varepsilon}{2} &\forall n>n_1,\\ 				&\exists  n_2>0\,:\,\,\, \left \vert b_n-b\right \vert<\frac{\varepsilon}{2} &\forall n>n_2.\\ 			\end{aligned} 		\end{equation*}

    Ponendo N=\max\{n_1,n_2\} si ha che per ogni n>N

        \begin{equation*} 			|a_n+b_n-(a+b)|\leq |a_n-a|+|b_n-b|\stackrel{53}{<}\frac{\varepsilon}{2}+\frac{\varepsilon}{2}=\varepsilon, 		\end{equation*}

    dove la prima disuguaglianza segue dalla disuguaglianza triangolare. La stessa argomentazione si applica analogamente alla dimostrazione per la differenza.

  4.  

  5. Dato che la successione è convergente, essa è limitata per il teorema 3.19, ovvero esiste M>0 tale che

        \begin{equation*} 			|a_n|\leq M\qquad\forall n\in\mathbb{N}. 		\end{equation*}

    Fissiamo \varepsilon>0 e definiamo n_1 e n_2 che soddisfino (53). Ponendo N=\max\{n_1,n_2\} per ogni n>N si ha

        \begin{equation*} 			|a_n b_n-a b|=|a_nb_n-a_nb+a_n b -a b|\leq |a_n(b_n-b)|+|b(a_n-a)|\leq M|b_n-b|+|b||a_n-a|<\frac{\varepsilon}{2}(M+|b|). 		\end{equation*}

    Per l’arbitrarietà di \varepsilon la tesi è dimostrata.

  6.  

  7. Mostriamo che la successione \frac{1}{b_n} converge a \frac{1}{b}; ciò, insieme al punto 3 concluderà la dimostrazione.

        \[\quad\]

    • b_n^{-1} esiste definitivamente. Osserviamo innanzitutto che esiste N_1 \in \mathbb{N} tale che \dfrac{1}{b_n} esiste per ogni n \geq N. Infatti, poiché b \neq 0 fissiamo \varepsilon>0 tale che 0 \notin (b-\varepsilon,b+\varepsilon); per la definizione 3.2, esiste N_1 \in \mathbb{N} tale che b_n \in (b-\varepsilon,b+\varepsilon). Pertanto, per come abbiamo scelto \varepsilon,

      (54)   \begin{equation*} 				b_n \neq 0 				\qquad 				\forall n \geq N_1, 			\end{equation*}

      quindi \frac{1}{b_n} è ben definita per ogni n \geq N. Vedremo che tale argomento consiste nella dimostrazione del cosiddetto teorema di permanenza del segno (teorema 5.3).

    •  

    • b_n^{-1} \to {b}^{-1}. Si fissi \varepsilon>0. Poiché b_n \to b, per il punto 1 si ha |b_n| \to |b|. Pertanto, considerando gli intorni (b-\varepsilon,b+\varepsilon) e \left ( \dfrac{|b|}{2}, 2 |b| \right ) rispettivamente di b e |b|4, per la definizione 3.2 esiste N \geq N_1 tale che

      (55)   \begin{equation*} 				|b_n - b|< \varepsilon 				\quad 				\text{e} 				\quad 				|b_n| \geq \frac{|b|}{2} 				\qquad 				\forall n \geq N. 			\end{equation*}

      Si ha quindi

      (56)   \begin{equation*} 				\left | \frac{1}{b_n} - \frac{1}{b} \right | 				= 				\frac{|b-b_n|}{|b_n b|} 				\leq 				\frac{2}{|b|^2} \varepsilon 				\qquad 				\forall n \geq N. 			\end{equation*}

      Per l’arbitrarietà di \varepsilon, segue che \frac{1}{b_n} \to \frac{1}{b}.

Osservazione 4.2. Il viceversa del punto 1 del teorema 4.1 non è valido: se una successione a_n è tale che |a_n| \to |a|, la successione a_n può anche non essere convergente, come mostra l’esempio 3.17: si ha |a_n|= |(-1)^n|=1 per ogni n \in \mathbb{N}, quindi |a_n| \to 1 ma, come si è già osservato, la successione a_n non è convergente. Osserviamo però che tale viceversa è valido se a=0, come mostra la seguente proposizione.

Proposizione 4.3. Sia a_n una successione tale che |a_n| \to 0; allora si ha \displaystyle\lim_{n \to + \infty} a_n=0.

    \[\quad\]

Dimostrazione. Fissiamo \varepsilon>0. Poiché |a_n| \to 0, per il punto 2 della proposizione 3.4 esiste N \in \mathbb{N} tale che

(57)   \begin{equation*} 		|a_n| < \varepsilon 		\qquad 		\forall n \geq N. 	\end{equation*}

Dalle proprietà del valore assoluto deduciamo quindi -\varepsilon < a_n < \varepsilon per ogni n \geq N, dimostrando quindi che anche a_n è infinitesima.

Risultati analoghi si hanno quando una o entrambe le successioni sono divergenti.

Proposizione 4.4. (algebra dei limiti in \overline{\mathbb{R}}). Sono valide le seguenti proprietà.

  1. Se a_n\to +\infty e b_n è limitata inferiormente (in particolare se b_n \to \ell \neq -\infty)), allora

    (58)   \begin{equation*} 			\lim_{n \to +\infty} (a_n+b_n)=+\infty. 		\end{equation*}

    Un risultato analogo vale se a_n\to -\infty e b_n è limitata superiormente.

  2. Se \displaystyle\lim_{n \rightarrow +\infty}a_n=+\infty e \displaystyle\lim_{n \rightarrow +\infty}b_n=\ell \neq 0, allora

        \[\lim_{n \rightarrow +\infty} a_n\cdot b_n=\operatorname{sgn}(\ell) \cdot (+\infty).\]

    Un risultato analogo vale se a_n \to -\infty.

  3. Se \displaystyle\lim_{n \rightarrow +\infty}a_n=\pm\infty e b_n è una successione limitata, allora \displaystyle\lim_{n \rightarrow +\infty} \frac{b_n}{a_n}=0.
  4.  

  5. Se \displaystyle\lim_{n \rightarrow +\infty}a_n=0 con a_n\neq 0 definitivamente e b_n è una successione tale che esista \beta>0 con b_n>\beta definitivamente, allora

        \[\displaystyle\lim_{n \rightarrow +\infty} \frac{b_n}{a_n}=+\infty.\]

    Valgono risultati analoghi per le variazioni di segno di a_n e b_n.

    \[\quad\]

In generale non possiamo concludere niente nei casi che non rientrano nella tabella precedente e cioè per le cosiddette forme indeterminate che saranno oggetto di studio dei successivi paragrafi:

    \begin{equation*} 	\left[\infty - \infty\right], \quad \left[0 \cdot \infty\right], \quad \left[\frac{\infty}{\infty}\right], \quad \left[\frac{0}{0}\right]. \end{equation*}

Per esempio, quando diciamo che \left[+\infty - \infty\right] è una forma indeterminata, intendiamo che se a_n \to +\infty e b_n \to -\infty, non possiamo dedurre in modo immediato quale sarà il comportamento limite della successione a_n + b_n. Questa potrebbe convergere verso un qualsiasi elemento di \overline{\mathbb{R}} o potrebbe anche non avere limite. La determinazione dell’esistenza e del valore di un eventuale limite per a_n + b_n richiede un’analisi dettagliata del caso specifico, come illustrato nell’esempio seguente.

Esempio 4.5. (forma indeterminata +\infty - \infty). Ci accingiamo a mostrare che, se le successioni a_n e b_n divergono rispettivamente a +\infty e -\infty, la successione a_n+b_n può divergere positivamente, oppure divergere negativamente, oppure avere come limite un qualsiasi numero reale, oppure non avere limite.

    \[\quad\]

  1. Consideriamo le successioni definite da

    (59)   \begin{equation*} 			a_n = 2n, 			\quad 			b_n=-n 			\qquad 			\forall n \in \mathbb{N}. 		\end{equation*}

    Per quanto visto precedentemente, vale a_n \to + \infty e b_n \to - \infty. Si ha

    (60)   \begin{equation*} 			\lim_{n \to + \infty} (a_n + b_n) 			= 			\lim_{n \to + \infty} (2n-n) 			= 			\lim_{n \to + \infty} n 			= 			+\infty. 		\end{equation*}

  2.  

  3. Analogamente, se

    (61)   \begin{equation*} 			a_n = n, 			\quad 			b_n=-2n 			\qquad 			\forall n \in \mathbb{N}, 		\end{equation*}

    vale (a_n + b_n) \to - \infty.

  4.  

  5. Consideriamo ora

    (62)   \begin{equation*} 			a_n = n + 7, 			\quad 			b_n=-n 			\qquad 			\forall n \in \mathbb{N}. 		\end{equation*}

    Di nuovo abbiamo a_n \to + \infty e b_n \to - \infty, ma

    (63)   \begin{equation*} 			\lim_{n \to + \infty} (a_n + b_n) 			= 			\lim_{n \to + \infty} (n + 7 -n) 			= 			\lim_{n \to + \infty} 7 			= 			7, 		\end{equation*}

    quindi in questo caso a_n + b_n ha limite finito. Chiaramente il numero 7 è arbitrario e al suo posto può essere scelto qualsiasi numero reale.

  6.  

  7. Studiamo infine il caso

    (64)   \begin{equation*} 			a_n = n + (-1)^n, 			\quad 			b_n=-n 			\qquad 			\forall n \in \mathbb{N}. 		\end{equation*}

    Si ha a_n \geq n-1 per ogni n \in \mathbb{N}, quindi vale a_n \to + \infty e b_n \to - \infty, ma

    (65)   \begin{equation*} 			a_n + b_n 			= 			n + (-1)^n -n 			= 			(-1)^n 			\qquad 			\forall n \in \mathbb{N}, 		\end{equation*}

    che non ha limite, per l’esempio 3.17.

Teorema 4.6 (prodotto di successioni limitate per infinitesime). Siano \{a_n\}_{n\in\mathbb{N}} e \{b_n\}_{n\in\mathbb{N}} due successioni tali che

    \begin{equation*} 			\lim_{n \rightarrow +\infty} a_n=0 		\end{equation*}

e b_n limitata. Allora

    \begin{equation*} 			\lim_{n \rightarrow +\infty}a_n\cdot b_n=0. 		\end{equation*}

    \[\quad\]

Dimostrazione. Per ipotesi \{b_n\}_{n\in\mathbb{N}} è una successione limitata, quindi esiste M>0 tale che |b_n|<M per ogni n\in\mathbb{N}, mentre la successione \{a_n\}_{n\in\mathbb{N}} è infinitesima, quindi fissato un valore \varepsilon>0, esiste N>0 tale che

    \begin{equation*} 		|a_n|<\frac{\varepsilon}{M}\qquad\forall n>N. 	\end{equation*}

Quindi, per n>N

    \begin{equation*} 		|a_n\cdot b_n|=|a_n|\cdot |b_n|<\frac{\varepsilon}{M}\cdot M=\varepsilon, 	\end{equation*}

ovvero la successione \{a_n\cdot b_n\} è anch’essa infinitesima.

Esempio 4.7. Calcoliamo il limite

    \begin{equation*} 	\lim_{n \rightarrow +\infty} \frac{\sin n}{n}. \end{equation*}

Definendo le successioni a_n = \frac{1}{n} e b_n = \sin n, si ha che

    \begin{equation*} 	\lim_{n \rightarrow +\infty} a_n = 0 \quad \text{e} \quad |\sin n| \leq 1 \quad \forall n \in \mathbb{N}. \end{equation*}

In virtù del teorema 4.6, possiamo concludere che

    \begin{equation*} 	\lim_{n \rightarrow +\infty} \frac{\sin n}{n} = \lim_{n \rightarrow +\infty} a_n \cdot \sin n = 0. \end{equation*}

   


  1. l’intervallo \left ( \frac{|b|}{2}, 2 |b| \right ) è non vuoto dato che b \neq 0.

 

Teoremi sui limiti di successioni

Introduzione.

Esponiamo qui di seguito alcuni risultati chiave sulle successioni, elementi essenziali della teoria.

Successioni e monotonia.

In questa sezione esaminiamo le correlazioni tra la monotonia delle successioni e i loro limiti. Iniziamo con un risultato fondamentale che collega la regolarità di una successione alla sua monotonia, stabilendo che tutte le successioni monotone hanno limite, come illustrato in figura 16.

Teorema 5.1 (limiti di successioni monotone). Se a_n è una successione monotona, essa ha limite \ell. Se in aggiunta a_n è limitata, allora \ell \in \mathbb{R} e a_n converge a \ell. Precisamente, valgono le seguenti proprietà.

    \[\quad\]

  • Se a_n è crescente, allora \ell è il suo estremo superiore: \ell= \sup \{a_n \colon n \in \mathbb{N}\}.
  •  

  • Se a_n è decrescente, allora \ell è il suo estremo inferiore: \ell= \inf \{a_n \colon n \in \mathbb{N}\}.

    \[\quad\]

    \[\quad\]

    \[\quad\]

Figura 16: illustrazione del teorema 5.1 con a_n crescente e \ell \in \mathbb{R}. Poiché \ell=\sup_n a_n, chiaramente a_n \leq \ell per ogni n. D’altra parte, a_n si avvicina a \ell per la monotonia di a_n e proprio in quanto esso è l’estremo superiore della successione.

    \[\quad\]

    \[\quad\]

Dimostrazione. Ci concentriamo sul caso di una successione a_n crescente, poiché l’analisi per una successione decrescente è analoga o può essere derivata considerando la successione -a_n. Definiamo \ell = \sup\{a_n \mid n \in \mathbb{N}\}. Data la monotonia di a_n, è evidente che \ell \neq -\infty. Esaminiamo i due casi possibili: \ell = +\infty e \ell \in \mathbb{R}.

    \[\quad\]

  • \ell=+\infty In tal caso a_n è illimitata superiormente, ossia per ogni M \in \mathbb{R}, esiste N \in \mathbb{N} tale che a_N \geq M. Poichè la successione a_n è crescente, da tale disuguaglianza segue

    (66)   \begin{equation*} 			a_n \geq M 			\qquad 			\forall n \geq N. 		\end{equation*}

    Per l’arbitrarietà di M, si ha quindi a_n \to + \infty.

  •  

  • \ell\in \mathbb{R}. La successione è limitata e sia \ell=\sup \{a_n \colon n \in \mathbb{N}\}. Fissato \varepsilon>0, per le proprietà dell’estremo superiore esiste N>0 tale che

        \begin{equation*} 			a_N>\ell-\varepsilon. 		\end{equation*}

    Per la monotonia della successione a_n, per ogni n>N

        \begin{equation*} 			\ell-\varepsilon<a_N<a_n\leq\ell<l+\varepsilon, 		\end{equation*}

    ovvero \displaystyle \lim_{n\rightarrow +\infty}a_n=\ell.

Il seguente teorema garantisce che ogni successione possiede un’estratta monotona.

Teorema 5.2 (esistenza di estratte monotone). Ogni successione a_n possiede un’estratta monotona. Se inoltre a_n è illimitata, valgono le seguenti proprietà.

    \[\quad\]

  • Se a_n è illimitata superiormente, allora esiste una sua estratta a_{n_k} strettamente crescente e tale che a_{n_k} \to + \infty.
  •  

  • Se a_n è illimitata inferiormente, allora esiste una sua estratta a_{n_k} strettamente decrescente e tale che a_{n_k} \to - \infty.

    \[\quad\]

Dimostrazione. Sia a_n la successione in questione. Diciamo che un termine a_n è un picco se

(67)   \begin{equation*} 		a_n \geq a_m 		\qquad 		\forall m > n, 	\end{equation*}

ossia se a_n è maggiore o uguale a ognuno dei termini ad esso successivi.5 Distinguiamo quindi due casi.

    \[\quad\]

  • a_n ha infiniti picchi. In tal caso, scegliamo come sottosuccessione a_{n_k} quella dei picchi. Poiché per definizione di estratta la funzione k \mapsto n_k è strettamente crescente, si ha n_{k+1} > n_k per ogni k \in \mathbb{N}. Quindi, dato che ogni a_{n_k} è un picco, si ha

    (68)   \begin{equation*} 			a_{n_k} 			\geq 			a_{n_{k+1}} 			\qquad 			\forall k \in \mathbb{N}, 		\end{equation*}

    ovvero la sottosuccessione a_{n_k} è decrescente.

  •  

  • a_n ha un numero finito di picchi. Poiché il numero di picchi è finito, esiste N \in \mathbb{N} tale che, per ogni n \geq N, a_n non è un picco. Costruiamo quindi una sottosuccessione crescente per induzione.

    Fissiamo a_{n_1}= a_{N}. Supponiamo di aver scelto a_{n_k} e scegliamo a_{n_{k+1}} nel seguente modo: dato che n_k \geq N, a_{n_k} non è un picco, quindi esiste m > n_k tale che a_m > a_{n_k}. Poniamo quindi a_{n_{k+1}}=a_m. Per induzione l’estratta \{a_{n_k}\}_{k \in \mathbb{N}} è definita ed è strettamente crescente.

Ciò conclude la dimostrazione dell’esistenza di un’estratta monotona. Rimane da provare la seconda parte del teorema, ovvero che tale estratta può essere scelta divergente a +\infty se la successione a_n è illimitata superiormente e divergente a -\infty se a_n è illimitata inferiormente. Mostriamo solo il caso in cui essa sia illimitata superiormente in quanto l’altro caso si svolge in maniera del tutto analoga. Osserviamo che l’estratta monotona esibita nella prima parte della dimostrazione potrebbe non essere divergente, anche se a_n è illimitata. Pertanto occorre definire la sottosuccessione a_{n_k} in altro modo.

Costruiamo per induzione l’estratta a_{n_k} crescente. Scegliamo a_{n_1} >  1, che esiste per l’illimitatezza di a_n. Supponendo di aver scelto i termini fino a a_{n_k}, scegliamo a_{n_{k+1}} tale che

(69)   \begin{equation*} 		a_{n_{k+1}} > \max \{ a_{n_1}, \dots, a_{n_k}, k+1\}, 	\end{equation*}

e tale scelta è possibile di nuovo perché a_n è illimitata superiormente. Per costruzione si ha quindi

(70)   \begin{equation*} 		a_{n_k} < a_{n_{k+1}} 		\qquad 		\forall k \in \mathbb{N}, 	\end{equation*}

ossia a_{n_k} è strettamente crescente. A questo punto fissato M>0, sia N\in\mathbb{N} tale che N\geq M. Poiché per costruzione si ha che a_{n_N}>N, per la monotonia di a_n si ha a_{n_k}\geq M per ogni k\geq N.    


  1. Osserviamo che, se la successione è strettamente crescente, allora nessun termine è un picco; mentre, se essa è decrescente, allora tutti i termini sono picchi.

Teoremi di confronto e della permanenza del segno.

In questa sezione studiamo gli importantissimi teoremi di confronto. Essi ci permettono di dedurre il carattere e le proprietà dell’eventuale limite di una successione, basandoci sul confronto (ovvero su disuguaglianze) con altre successioni di cui conosciamo il comportamento. Questi strumenti si rivelano utilissimi in quanto molto spesso nella pratica risulta difficile provare la convergenza di una particolare successione a_n e stabilire il valore del suo eventuale limite. Può essere di notevole aiuto confrontare a_n con una successione nota e quindi dedurre tutte le informazioni necessarie da tale confronto.

Un primo importante risultato è il seguente teorema, detto della permanenza del segno, che afferma che una successione avente limite strettamente positivo è definitivamente a termini strettamente positivi.

Teorema 5.3 (permanenza del segno). Sia \{a_n\}_{n\in\mathbb{N}} una successione.

    \[\quad\]

  1. Se \displaystyle 			\lim_{n \rightarrow +\infty} a_n=\ell>0, allora esiste N\in\mathbb{N} tale che a_n>0 per ogni n>N, ossia a_n>0 definitivamente.
  2.  

  3. Se \displaystyle 			\lim_{n \rightarrow +\infty} a_n=\ell<0, allora esiste N\in\mathbb{N} tale che a_n<0 per ogni n>N, ossia a_n<0 definitivamente.

    \[\quad\]

Il teorema stabilisce che, se il limite di una successione è strettamente positivo, allora la successione sarà definitivamente composta da termini positivi.

Dimostrazione. Mostriamo solo il caso \ell>0, in quanto l’altro caso è analogo o si ottiene, ad esempio, considerando la successione -a_n.

    \[\quad\]

  • Se \ell \in(0,+\infty), allora dalla caratterizzazione 3 di 3.4 di limite di successione, scegliendo \varepsilon=\frac{\ell}{2}, esiste N tale che

        \begin{equation*} 			|a_n-\ell|<\frac{\ell}{2}\qquad \forall n>N, 		\end{equation*}

    ovvero

        \begin{equation*} 			a_n>\ell-\frac{\ell}{2}=\frac{\ell}{2}>0\qquad\forall n>N. 		\end{equation*}

  •  

  • Se \ell=+\infty sempre grazie alla caratterizzazione 2 di 3.4, per un valore M>0 fissato esiste N\in\mathbb{N} tale che

        \begin{equation*} 			a_n>M>0\qquad\,\forall n>N. 		\end{equation*}

Come già anticipato, la prossima proposizione è una generalizzazione del teorema 5.3.

Proposizione 5.4. Sia a_n,b_n due successioni tali che a_n \to \ell_a e b_n \to \ell_b e supponiamo che \ell_a < \ell_b. Allora esiste N \in \mathbb{N} tale che

(71)   \begin{equation*} 			a_n < b_n 			\qquad 			\forall n \geq N. 		\end{equation*}

    \[\quad\]

Dimostrazione. Se \ell_b = + \infty e \ell_a=-\infty allora per definizione si ha definitivamente a_n < 0 < b_n, e la conclusione risulta ovvia. Nel caso in cui almeno uno dei limiti sia finito, consideriamo la successione b_n - a_n. Per il teorema 4.1 e la proposizione 4.4 si ha

(72)   \begin{equation*} 		\lim_{n \to + \infty} (b_n - a_n) 		= 		\ell_b - \ell_a 		> 		0. 	\end{equation*}

Per il teorema 5.3, b_n - a_n è definitivamente positivo, concludendo la dimostrazione.

La generalizzazione del teorema della permanenza del segno data dalla proposizione 5.4 si può enunciare anche nella forma equivalente seguente,6 illustrata in figura 17.

Proposizione 5.5 (disuguaglinze al limite). Siano a_n,b_n due successioni tali che definitivamente valga a_n \leq b_n. Se a_n \to \ell_a e b_n \to \ell_b, allora si ha

(73)   \begin{equation*} 			\ell_a \leq \ell_b. 		\end{equation*}

    \[\quad\]

Dimostrazione. Se per assurdo fosse \ell_a > \ell_b, per la proposizione 5.4 si avrebbe a_n>b_n definitivamente, contro l’ipotesi a_n \leq b_n definitivamente.

    \[\quad\]

    \[\quad\]

    \[\quad\]

Figura 17: illustrazione della proposizione 5.5 nel caso \ell_a, \ell_b \in \mathbb{R}.

    \[\quad\]

    \[\quad\]

Osservazione 5.6. La conclusione della proposizione 5.5 non si può rafforzare in \ell_a < \ell_b, anche assumendo che a_n < b_n definitivamente, come mostra il prossimo esempio.

Esempio 5.7. Consideriamo le successioni definite da

(74)   \begin{equation*} 		a_n= \frac{1}{2n}, 		\quad 		b_n = \frac{1}{n} 		\qquad 		\forall n \in \mathbb{N} 	\end{equation*}

Chiaramente si ha a_n < b_n per ogni n \in \mathbb{N}. Tuttavia sono entrambe infinitesime, ossia convergono allo stesso limite \ell_a= \ell_b = 0. Infatti, b_n \to 0, come mostrato nell’esempio 3.6. Il fatto che a_n \to 0 segue da ciò, dal fatto che a_n=\frac{1}{2}b_n e dal punto 1 del teorema 4.1.

Siamo ora in grado di enunciare il teorema del confronto, un principio fondamentale dell’Analisi Matematica che assume diverse forme a seconda del contesto. Esso permette di determinare il limite di una successione b_n confrontandola con altre due successioni a_n e c_n aventi lo stesso limite \ell e che la limitano dal basso e dall’alto. Conosciuto informalmente come il teorema dei due carabinieri, l’analogia descrive le successioni a_n e c_n come carabinieri che “convergono” verso lo stesso punto, costringendo necessariamente anche la successione b_n, il prigioniero, a fare altrettanto.

Teorema 5.8 (del confronto o dei carabinieri). Siano a_n, b_n, c_n successioni, si supponga che esista N_1\in\mathbb{N} tale che

(75)   \begin{equation*} 			a_n \leq b_n \leq c_n, \;\;\; \forall n \geq N_1 		\end{equation*}

e che valga

    \[\lim_{n \to \infty} a_n = \lim_{n \to \infty} c_n  = \ell \in \mathbb{R}.\]

Allora

    \[\lim_{n \to \infty} b_n  = \ell .\]

    \[\quad\]

Dimostrazione. Per dimostrare il teorema, consideriamo un qualsiasi \varepsilon > 0 e mostriamo che esiste N \in \mathbb{N} tale che \ell - \varepsilon \leq b_n \leq \ell + \varepsilon per tutti gli n \geq N. Fissato un \varepsilon arbitrario, dalla definizione di limite e dall’ipotesi che

    \[\lim_{n \to \infty} a_n = \lim_{n \to \infty} c_n = \ell \in \mathbb{R},\]

si deduce l’esistenza di due numeri naturali N_2 e N_3 tali che

(76)   \begin{equation*} 		\ell - \varepsilon \leq a_n \leq \ell + \varepsilon \quad \forall n \geq N_2, \quad \text{e} \quad \ell - \varepsilon \leq c_n \leq \ell + \varepsilon \quad \forall n \geq N_3. 	\end{equation*}

Scegliendo N = \max\{N_1, N_2, N_3\}, entrambe le condizioni (75) e (76) sono soddisfatte simultaneamente, quindi si ottiene

    \[\ell - \varepsilon \leq a_n \leq b_n \leq c_n \leq \ell + \varepsilon \quad \forall n \geq N,\]

completando così la dimostrazione.

    \[\quad\]

    \[\quad\]

Figura 18: illustrazione del teorema 5.8. Le successioni a_n,b_n,c_n, rispettivamente in rosso, blu e verde, soddisfano a_n \leq b_n \leq c_n per ogni n \geq 3. Poiché per n \geq 8 vale a_n,c_n \in (\ell -\varepsilon,\ell+\varepsilon), allora anche b_n appartiene a tale intorno.

    \[\quad\]

    \[\quad\]

Esempio 5.9. Calcoliamo il seguente limite:

    \[\lim_{n \rightarrow +\infty}\dfrac{\cos n}{n}.\]

Si ricorda che

    \[-1 \leq \cos n \leq 1\qquad \forall n \in \mathbb{N},\]

da cui

    \[- \frac{1}{n} \leq  \frac{\cos n}{n} \leq  \frac{1}{n}\qquad \forall n \in \mathbb{N}\setminus\{0\}.\]

Posto a_n = - \frac{1}{n}, b_n=\frac{\cos n}{n} e c_n =  \frac{1}{n}, grazie all’esempio 2.3 e al punto 3 del teorema 4.1

    \[\lim_{n \to \infty} a_n = \lim_{n \to \infty} c_n  = 0.\]

Allora per il teorema 5.8, possiamo concludere che

    \[\lim_{n \to \infty} b_n  = 0.\]

Il teorema 5.8 trova applicazione anche in situazioni dove il limite è infinito. In questi casi, è sufficiente limitare la successione da un solo “lato”.

Teorema 5.10 (del confronto: \ell=\pm \infty). Siano \{a_n\}_{n \in \mathbb{N}},\{b_n\}_{n \in \mathbb{N}} due successioni e supponiamo che esista N_1 \in \mathbb{N} tale che

(77)   \begin{equation*} 			a_n \leq b_n 			\qquad 			\forall n \geq N_1. 		\end{equation*}

  1. Se a_n \to +\infty, allora b_n \to + \infty.
  2. Se b_n \to - \infty, allora a_n \to - \infty.

    \[\quad\]

In altre parole, se b_n è limitata dal basso da una successione divergente a +\infty, allora anch’essa diverge a +\infty; analogamente se a_n è limitata dall’alto da una successione divergente a -\infty, allora anch’essa diverge a -\infty.

Dimostrazione. Mostriamo solo il caso 1, in quanto il caso 2 si prova in modo simile oppure può essere ottenuto considerando le successioni -b_n, -a_n. Si fissi M \in \mathbb{R}. Poiché a_n \to + \infty, esiste N \geq N_1 tale che a_n \geq M per ogni n \geq N. Per (77) si ha

(78)   \begin{equation*} 		b_n \geq a_n \geq M 		\qquad 		\forall n \geq N, 	\end{equation*}

ossia, per l’arbitrarietà di M, abbiamo che b_n \to + \infty.

A questo punto abbiamo gli strumenti necessari per dimostrare la proposizione 3.18.

Dimostrazione della proposizione 3.18. Mostriamo l’equivalenza mediante una doppia implicazione.

    \[\quad\]

  • 1 \Rightarrow 2.
  • Consideriamo per ogni n\in\mathbb{N} con n\neq 0 l’intorno circolare di x_0 di raggio \delta=\frac{1}{n}

        \begin{equation*} 		I_\frac{1}{n}(x_0)=\left(x_0-\frac{1}{n},\,x_0+\frac{1}{n}\right). 	\end{equation*}

    Poiché x_0 è un punto di accumulazione per l’insieme A, allora l’intorno I_\frac{1}{n}(x_0) contiene almeno un elemento a_n\in A diverso da x_0:

    (79)   \begin{equation*} x_0-\frac{1}{n}<a_n<x_0+\frac{1}{n}. \end{equation*}

    Abbiamo creato una successione a_n a valori in A che converge a x_0 per il teorema del confronto 5.8, visto che le due successioni x_0-\frac{1}{n} e x_0+\frac{1}{n} convergono entrambe a x_0.

     

  • 2 \Rightarrow 1.
  • Per ipotesi a_n è una successione di valori nell’insieme A\setminus\{x_0\} che converge a x_0, quindi per definizione di limite 3.2 per ogni intorno I di x_0, esiste N \in \mathbb{N} tale che

    (80)   \begin{equation*} 	a_n \in I 	\qquad 	\forall n \geq N. \end{equation*}

    Quindi ogni intorno I di x_0 contiene almeno un elemento dell’insieme A diverso da x_0, ovvero x_0 è un punto di accumulazione.

Esempio 5.11. Sia a \in \mathbb{R}. Allora

    \[\boxcolorato{analisi}{{ 			\lim_{n \to + \infty} a^n= 			\begin{cases} 				+\infty			& \text{se } a >1\\ 				1 				& \text{se } a =1\\ 				0					& \text{se } a \in (-1,1)\\ 				\text{non esiste}	& \text{se } a \leq -1. 			\end{cases} 		}}\]

Analizziamo separatamente i diversi casi.

    \[\quad\]

  • a>1. In tal caso, possiamo scrivere a=1+x con x >0. Quindi, per la disuguaglianza di Bernoulli (35), si ha

    (81)   \begin{equation*} 			a^n 			= 			(1+x)^n 			\geq 			1+nx 			\qquad 			\forall n \in \mathbb{N}. 		\end{equation*}

    Poiché x>0, si ha 1+nx \to + \infty per i punti 1 e 2 della proposizione 4.4. Dunque, dal teorema 5.10, segue che a^n \to + \infty.

  •  

  • a=1. In tal caso, ovviamente a^n è costantemente pari a 1, dunque a^n \to 1.
  •  

  • a \in (-1,1). L’enunciato è ovvio per a=0. Se a \neq 0, vogliamo mostrare che |a|^n \to 0; osserviamo che esiste b >1 tale che |a|=\frac{1}{b}. Quindi

    (82)   \begin{equation*} 			\lim_{n \to + \infty} |a|^n 			= 			\lim_{n \to + \infty} \frac{1}{|b|^n} 			= 			0, 		\end{equation*}

    dove l’ultima uguaglianza segue da b^n \to +\infty per il caso a>1, e dal punto 3 della proposizione 4.4. Poiché abbiamo mostrato che |a|^n \to 0, per la proposizione 4.3 vale anche a^n \to 0.

  •  

  • a \leq -1. Consideriamo le sottosuccessioni dei termini pari e dei termini dispari, che sono rispettivamente positivi e negativi. Più precisamente, usando il caso a>1, si ottiene

    (83)   \begin{equation*} 		\lim_{k \to +\infty} a^{2k} 		= 		\begin{cases} 			1		& \text{se } a=-1\\ 			+\infty	& \text{se } a< -1, 		\end{cases} 		\qquad 		\lim_{k \to +\infty} a^{2k+1} 		= 		\begin{cases} 			-1		& \text{se } a=-1\\ 			-\infty	& \text{se } a < -1. 		\end{cases} 	\end{equation*}

    Avendo due estratte con limiti diversi, per la proposizione 3.16 dunque, la successione a^n non ha limite.

Osservazione 5.12. Utilizzando gli stessi ragionamenti dell’esempio 5.11, si può dimostrare che

    \[\boxcolorato{analisi}{x_n \to + \infty 			\quad 			\Longrightarrow 			\quad 			\lim_{n \to + \infty} a^{x_n} 			=\begin{cases} 				+\infty			& \text{se } a >1\\ 				 1 				& \text{se } a =1\\ 				0					& \text{se } a \in (0,1). 			\end{cases} 		}}\]

   


  1. In logica, si dice che si tratta della contronominale della proposizione 5.4.

Compattezza: il teorema di Bolzano-Weierstrass.

Abbiamo visto, col teorema 3.19, che una successione convergente è limitata. Il viceversa di tale risultato è in generale falso: la successione dell’esempio 3.20 è limitata ma non convergente. Il teorema di Bolzano-Weierstrass consiste però in una parziale inversione del teorema 3.19: esso infatti afferma che ogni successione limitata possiede sempre una sottosuccessione convergente. Dunque, il viceversa del teorema 3.19 vale a meno di passare a un’opportuna estratta della successione iniziale.

Proponiamo due dimostrazioni di questo risultato: la prima fa uso del teorema 5.2 sull’esistenza di estratte monotone, mentre la seconda è più costruttiva e intuitiva, anche se leggermente più articolata.

Teorema 5.13 (Bolzano-Weierstrass). Sia \{a_n\}_{n\in\mathbb{N}} una successione limitata. Allora a_n possiede una sottosuccessione convergente. Se inoltre a_n è a valori in un insieme A\subseteq \mathbb{R} chiuso, allora l’estratta converge a un elemento di A.

    \[\quad\]

Dimostrazione. Consideriamo una successione limitata a_n. Secondo il teorema 5.2, a_n ammette una sottosuccessione monotona a_{n_k}. Per il teorema 5.1, questa sottosuccessione ha un limite \ell. Inoltre, essendo a_{n_k} una sottosuccessione di una successione limitata, anch’essa è limitata. Di conseguenza, applicando nuovamente il teorema 5.1, deduciamo che a_{n_k} converge a un numero reale \ell.

L’ultima affermazione segue immediatamente dalla caratterizzazione degli insiemi chiusi data dalla proposizione 3.13.

Dimostrazione alternativa del teorema 5.13. Cominciamo osservando che, poiché per ipotesi la successione a_n è limitata, esistono b,c \in \mathbb{R} tali che

(84)   \begin{equation*} 		b \leq a_n \leq c 		\qquad 		\forall n \in \mathbb{N}. 	\end{equation*}

L’idea della dimostrazione consiste nel costruire per ricorrenza una successione “decrescente” [b_k,c_k] di intervalli (ossia con la proprietà che [b_{k+1},c_{k+1}] \subset [b_k,c_k]) dimezzando ogni volta l’intervallo precedente in modo tale che ognuno contenga infiniti termini della successione, e di costruire la sottosuccessione \{a_{n_k}\}_{k \in \mathbb{N}} selezionando l’elemento a_{n_k} dall’intervallo [b_k,c_k], ottenendo quindi

(85)   \begin{equation*} 		b_k \leq a_{n_k} \leq c_k = b_k + \frac{c-b}{2^k} 		\qquad 		\forall k \in \mathbb{N}. 	\end{equation*}

Da [b_{k+1},c_{k+1}] \subset [b_k,c_k] seguirà che la successione b_k è crescente e c_k è decrescente. Da tale monotonia e da c_k= b_k + \frac{c-b}{2^k} si avrà che c_k e b_k convergono allo stesso limite \ell e, per il teorema 5.8, anche a_{n_k} vi convergerà.

    \[\quad\]

  • Passo base: b_1, c_1, a_{n_1}. Dividiamo l’intervallo [b, c] a metà. Scegliamo la metà, chiamata [b_1, c_1], che contiene infiniti elementi di a_n. Selezioniamo poi un termine della successione a_n all’interno di questo intervallo e lo chiamiamo a_{n_1}. Di conseguenza, abbiamo

    (86)   \begin{equation*} 			b_1 \leq a_{n_1} \leq c_1 = b_1 + \frac{c - b}{2}. 		\end{equation*}

  •  

  • Passo induttivo: b_{k+1}, c_{k+1}, a_{n_{k+1}}. Supponiamo di aver già determinato l’intervallo [b_k, c_k] che contiene infiniti termini di a_n e di aver scelto a_{n_k} in modo tale che soddisfi (85). Il nostro obiettivo è ora selezionare b_{k+1}, c_{k+1}, a_{n_{k+1}}. Dividiamo [b_k, c_k] in due parti uguali e definiamo [b_{k+1}, c_{k+1}] come una delle metà che contiene infiniti termini di a_n (almeno una delle metà deve necessariamente soddisfare questa condizione). Quindi,

    (87)   \begin{equation*} 			b_k \leq b_{k+1} < c_{k+1} = b_{k+1} + \frac{c - b}{2^{k+1}} \leq c_k. 		\end{equation*}

    Scegliamo a_{n_{k+1}} all’interno di [b_{k+1}, c_{k+1}] in modo che n_{k+1} > n_k, garantendo così che la sottosuccessione sia crescente negli indici. Di conseguenza, abbiamo che

    (88)   \begin{equation*} 			b_{k+1} \leq a_{n_{k+1}} \leq c_{k+1} = b_{k+1} + \frac{c - b}{2^{k+1}}. 		\end{equation*}

Per induzione risultano dunque definite le successioni b_k,c_k,a_{n_k}: esse soddisfano (85). Poiché b_k è crescente e limitata (dall’alto dai termini della successione c_k), essa risulta convergente per il teorema 5.1 a un valore \ell. Inoltre, dall’ultima uguaglianza in (85), dal fatto che 2^k \to +\infty e dal punto 3 della proposizione 4.4 e 4.1, segue che anche c_k \to \ell. Quindi, di nuovo per (85) e per il teorema 5.8, si ha

(89)   \begin{equation*} 		\lim_{k \to +\infty}a_{n_k}= \ell. 	\end{equation*}

Riguardo l’ultima affermazione, se a_n è a valori in A, con A \subseteq \mathbb{R} chiuso, allora anche a_{n_k} è a valori in A. Per la caratterizzazione per successioni degli insiemi chiusi data dalla proposizione 3.13, il suo limite \ell \in A.

Per completezza, presentiamo un’altra formulazione equivalente del del Teorema di Bolzano-Weierstrass.

Teorema 5.14 (Bolzano-Weierstrass, forma topologica). Sia A\subset \mathbb{R} un insieme infinito e limitato. Allora esiste almeno un punto di accumulazione per A.

    \[\quad\]

Dimostrazione. Poiché A è infinito, possiamo costruire una successione con valori in A per induzione come segue. Fissiamo a_1 \in A, e supponendo di aver definito a_n, definiamo a_{n+1} scegliendo un elemento in A \setminus \{a_1,\dots,a_n\} (tale insieme non è vuoto in quanto A è infinito). La successione a_n è limitata e, in accordo al Teorema 5.13, possiede una sottosuccessione a_{n_k} convergente a un valore \ell \in \mathbb{R}.

Affermiamo che \ell è un punto di accumulazione di A. Infatti, sia \varepsilon>0. Poiché a_{n_k} \to \ell, esiste K \in \mathbb{N} tale che

(90)   \begin{equation*} 		a_{n_k} \in I_\varepsilon(\ell)=(\ell - \varepsilon, \ell+ \varepsilon) 		\qquad 		\forall k \geq K. 	\end{equation*}

Dato che per costruzione la sottosuccessione a_{n_k} è costituita da punti di A tutti distinti, (90) implica che nell’intorno I_\varepsilon(\ell) esiste almeno un punto di A diverso da \ell. Per la definizione 1.14, \ell è quindi un punto di accumulazione per A.

Osservazione 5.15. Per dimostrare il teorema 5.14 abbiamo quindi usato il teorema 5.13. Viceversa, il teorema 5.14 permette di dimostrare facilmente il teorema 5.13. Infatti, sia a_n una successione limitata. Se a_n assume un numero finito di valori, esiste \ell \in \mathbb{R} tale che a_n=\ell per infiniti indici n. Quindi a_n=\ell frequentemente e quindi, per la proposizione 2.21 esiste un’estratta a_{n_k} costantemente pari a \ell, che risulta ovviamente convergente. Se invece a_n assume infiniti valori, l’insieme A=\{a_n \colon n \in \mathbb{N}\} risulta infinito e limitato. Dunque, per il teorema 5.14, esso possiede un punto di accumulazione \ell. Usando la proposizione 1.17, si costruisce quindi facilmente una sottosuccessione a_{n_k} convergente a \ell.

Osservazione 5.16. Il teorema 5.2 implica che, se a_n non è limitata, allora possiede un’estratta divergente.

Il teorema di Bolzano-Weierstrass permette di stabilire la seguente proprietà degli insiemi chiusi.

Corollario 5.17. Se A \subseteq \mathbb{R} è un insieme chiuso e limitato, allora esso possiede massimo M e minimo m, ovvero esistono M,m \in A tali che

(91)   \begin{equation*} 			m \leq x \leq M 			\qquad 			\forall x \in A. 		\end{equation*}

    \[\quad\]

Osservazione 5.18. La parte importante del teorema è che m,M \in A. Infatti, l’esistenza di siffatti numeri reali è data dalla definizione di limitatezza di A.

Dimostrazione. Poiché A è limitato, esistono finiti m = \inf A e M=\sup A. Mostriamo quindi che m,M \in A. Per definizione di estremo inferiore

(92)   \begin{equation*} 		\exists a_n \in A \cap \left ( m, m+\frac{1}{n} \right ) 		\qquad 		\forall n \in \mathbb{N}. 	\end{equation*}

La successione a_n così definita è a valori in A e converge a m per il teorema del confronto 5.8. Per la proposizione 3.13, il suo limite m appartiene ad A perché A è chiuso. Analogamente si prova che M \in A.


 

Successioni di Cauchy

Leggi...

Un concetto strettamente correlato a quello di limite consiste nella nozione di successione di Cauchy. Intuitivamente, una successione \{a_n\}_{n\in\mathbb{N}} è di Cauchy se da un certo indice in poi gli elementi a_n sono arbitrariamente vicini tra loro al crescere dell’indice n. Vedremo dal teorema 6.2 che tale condizione è equivalente alla convergenza della successione.

Definizione 6.1 (successioni di Cauchy). Una successione \{a_n\}_{n\in\mathbb{N}} è di Cauchy se \forall\varepsilon>0 esiste N\in\mathbb{N} tale che

    \begin{equation*} 			|a_n-a_m|<\varepsilon\qquad\forall n,\,m>N. 		\end{equation*}

    \[\quad\]

Intuitivamente, una successione è di Cauchy, se per qualunque valore \varepsilon>0, da un certo indice N in poi gli elementi di \{a_n\}_n hanno distanza minore di \varepsilon.

Teorema 6.2. Una successione a_n è di Cauchy se e soltanto se è convergente.

    \[\quad\]

Dimostrazione. (\Leftarrow) Una successione che ammette un limite finito \ell è di Cauchy: dato \varepsilon >0, troviamo N>0 tale che |a_n -\ell|<\dfrac{\varepsilon}{2} per ogni n\geq N. Dunque per n,m >N si ha che

    \[|a_n-a_m|\leq |a_n-L|+|a_m-L|<\dfrac{\varepsilon}{2}+\dfrac{\varepsilon}{2}=\varepsilon,\]

ovvero la successione è di Cauchy.

(\Rightarrow) Supponiamo che a_n sia di Cauchy. Fissando \varepsilon>0, esiste N \in \mathbb{N} tale che

(93)   \begin{equation*} 			|a_n-a_m| < \frac{\varepsilon}{2} 			\quad 			\forall n,m \geq N. 		\end{equation*}

Scegliendo m=N si ottiene

(94)   \begin{equation*} 			|a_n| = |a_n - a_N + a_N| 			\leq 			|a_n - a_N| + |a_N| 			\leq 			\dfrac{\varepsilon}{2}+ |a_N| 			\quad 			\forall n \geq N. 		\end{equation*}

Si ha quindi

(95)   \begin{equation*} 			|a_n| 			\leq 			M 			\coloneqq 			\max \left\{ |a_1|,\dots,|a_{N-1}|, \frac{\varepsilon}{2} + |a_N| \right \} 			\qquad 			\forall n \in \mathbb{N}, 		\end{equation*}

ovvero la successione è limitata. Per il teorema 5.13, esiste una sua sottosuccessione a_{n_k} convergente a un numero reale \ell.

Mostriamo che l’intera successione a_n converge a \ell. A tal fine, osserviamo che a_{n_k} \to \ell implica l’esistenza di k \in \mathbb{N} tale che

(96)   \begin{equation*} 			n_k> N 			\qquad 			\text{e} 			\qquad 			|a_{n_k} - \ell| < \frac{\varepsilon}{2} 		\end{equation*}

Da ciò segue che

(97)   \begin{equation*} 			|a_n- \ell| 			\leq 			|a_n - a_{n_k}| + |a_{n_k}-\ell| 			< 			\frac{\varepsilon}{2} + \frac{\varepsilon}{2} 			= 			\varepsilon 			\qquad 			\forall n \geq N, 		\end{equation*}

dove la prima disuguaglianza segue dalla disuguaglianza triangolare, mentre la seconda segue da (93) per il primo addendo (in quanto n_k \geq k per il lemma 3.15) e da (96) per il secondo addendo. Per l’arbitrarietà di \varepsilon, (97) prova che a_n \to \ell.

Poiché abbiamo dimostrato che le successioni di Cauchy sono tutte e sole le successioni convergenti, possiamo affermare che per esse valgono le stesse proprietà delle successioni convergenti. Ad esempio, consideriamo due successioni a_n e b_n di Cauchy; allora:

    \[\quad\]

  1. La successione a_n + b_n è una successione di Cauchy.
  2.  

  3. La successione a_n \cdot b_n è una successione di Cauchy.

Riportiamo un corollario di questo teorema che risulta molto utile nella pratica: infatti, data una successione a_n, può essere semplice calcolare il limite \ell di una particolare sottosuccessione a_{n_k}. Se la successione a_n è di Cauchy, allora essa converge a \ell.

Corollario 6.3. Se a_n è una successione di Cauchy e una sua estratta a_{n_k} converge a \ell \in \mathbb{R}, allora

(98)   \begin{equation*} 			\lim_{n \to + \infty} a_n 			= 			\ell. 		\end{equation*}

    \[\quad\]

Dimostrazione. Poiché a_n è di Cauchy, allora essa è convergente a un numero reale \ell' per il teorema 6.2. Per il teorema 3.16 si ha quindi a_{n_k} \to \ell'. Poiché a_{n_k} \to \ell per ipotesi, dal teorema 3.12 deve aversi \ell'=\ell, e quindi

(99)   \begin{equation*} 		a_n \to \ell. 	\end{equation*}

La nozione di successione di Cauchy e la caratterizzazione data dal teorema 6.2 sono molto utili nella pratica perché permettono di stabilire la convergenza di una successione senza calcolare esplicitamente il limite (cosa che può essere anche impossibile), fornendone una stima. Vediamo in dettaglio la cosa con un esempio.

Esempio 6.4. Consideriamo la successione definita da

(100)   \begin{equation*} 		a_n 		= 		\sum_{k=1}^{n} \frac{(-1)^k}{k} 		\qquad 		\forall n \in \mathbb{N}. 	\end{equation*}

Proviamo che essa è convergente a un limite \ell \in \mathbb{R} e determiniamo una sua stima.

Mostriamo che a_{n+2} è strettamente compreso tra a_n e a_{n+1}. Per fissare le idee, supponiamo che n sia dispari (l’altro caso è analogo); allora abbiamo

(101)   \begin{gather*} 		a_{n+2} = a_{n+1} -\frac{1}{n+2} < a_{n+1} 		\\ 		a_{n+2} = a_{n} +\frac{1}{n+1} - \frac{1}{n+2} > a_n. 	\end{gather*}

Quindi a_{n} < a_{n+2} < a_{n+1}. Reiterando questa disuguaglianza, si ottiene (per n dispari)

(102)   \begin{equation*} 		a_n < a_m < a_{n+1} 		%= a_n + \frac{1}{n+1} 		\qquad 		\forall m \geq n+2. 	\end{equation*}

Poiché tutti i termini successivi sono strettamente contenuti tra a_n e a_{n+1} = a_n + \dfrac{(-1)^{n+1}}{n+1}, si ha

(103)   \begin{equation*} 		|a_m - a_h|< \frac{1}{n+1} 		\qquad 		\forall m,h \geq n+2, 	\end{equation*}

ossia che la successione a_n è di Cauchy e quindi, per il teorema 6.2 essa converge a un limite \ell. Calcolando il limite per m \to + \infty in (102), per la proposizione 5.5, si ha

(104)   \begin{equation*} 		a_n \leq \ell \leq a_{n+1} 		\quad 		\text{se $n$ è dispari}, 		\qquad 		a_{n+1} \leq \ell \leq a_{n} 		\quad 		\text{se $n$ è pari}, 	\end{equation*}

che fornisce una stima di precisione pari a \frac{1}{n+1} su \ell. Quindi, ad esempio, per n=1, si ha

(105)   \begin{equation*} 		-1 \leq \ell \leq - \frac{1}{2}. 	\end{equation*}

È possibile dimostrare che \ell= - \log 2.


 

Il numero di Nepero

Leggi...

In Matematica il numero di Nepero è una costante alla base della funzione esponenziale e del logaritmo naturale. E’ un numero irrazionale trascendente7 (come \pi) estremamente interessante; insieme alla relativa funzione esponenziale, lo ritroviamo in tantissimi ambiti: dallo studio dei circuiti e della termodinamica in fisica, alla densità di variabili aleatorie in calcolo delle probabilità.

Esistono diverse definizioni del numero di Nepero, ma in questa trattazione adotteremo l’approccio più comune, che lo definisce come il limite di una successione.

Teorema 7.1. Siano \{a_n\}_{n \in \mathbb{N}},\{b_n\}_{n \in \mathbb{N}} le due successioni definite da

(106)   \begin{equation*} 			a_n 			= 			\left( 1+\frac{1}{n} \right )^n, 			\quad 			b_n 			= 			\left( 1+\frac{1}{n} \right )^{n+1} 			\qquad 			\forall n \in \mathbb{N}. 		\end{equation*}

Allora

    \[\quad\]

  1. a_n è crescente, b_n è decrescente;
  2.  

  3. a_n \leq b_n per ogni n \in \mathbb{N};
  4.  

  5. a_n e b_n convergono allo stesso limite, indicato con “e”, soddisfacente 2 < e < 3.

    \[\quad\]

Dimostrazione.

  1. Dimostriamo che la successione è crescente;

    (107)   \begin{equation*} 		\begin{split} 			a_n\geq a_{n-1}&\Longleftrightarrow\, \left(1+\frac{1}{n}\right)^n\geq \left(1+\frac{1}{n-1}\right)^{n-1}\\&\Longleftrightarrow\, \left(\frac{n+1}{n}\right)^n\geq\left(\frac{n}{n-1}\right)^{n-1}\\&\Longleftrightarrow\,\left(\frac{n+1}{n}\right)^n\geq\left(\frac{n}{n-1}\right)^{n}\cdot \left(\frac{n}{n-1}\right)^{-	1}\\&\Longleftrightarrow\left(\frac{n+1}{n}\right)^n\cdot \left(\frac{n}{n-1}\right)^{-n}\geq \frac{n-1}{n}\\&\Longleftrightarrow\,\left(\frac{n^2-1}{n^2}\right)^n\geq \frac{n-1}{n}\\&\Longleftrightarrow\left(1-\frac{1}{n^2}\right)^n\geq 1-\frac{1}{n}. 		\end{split} 	\end{equation*}

    Utilizziamo la disuguaglianza di Bernoulli

        \begin{equation*} 		(1+x)^n\geq 1+nx\qquad\forall x>-1, 	\end{equation*}

    per x=-\frac{1}{n^2} che risulta ovviamente maggiore di -1 per n>1, ottenendo dunque

        \begin{equation*} 		\left(1-\frac{1}{n^2}\right)^n\geq1+n\cdot \left(-\frac{1}{n^2}\right)=1-\frac{1}{n} \qquad\forall n>1,  	\end{equation*}

    quindi a_n\geq a_{n-1} per n\geq 2.

    Dimostriamo che la successione b_n è decrescente; analogamente a quanto fatto per a_n dimostriamo che:

        \begin{equation*} 		\begin{split} 		b_{n-1}>b_n&\Longleftrightarrow\left(\frac{n^2+1-1}{n^2-1}\right)^n>\frac{n+1}{n}\\&\Longleftrightarrow\, \left(1+\frac{1}{n^2-1}\right)^n>1+\frac{1}{n}. 		\end{split} 	\end{equation*}

    L’ultima maggiorazione è vera sempre grazie alla disuguaglianza di Bernoulli per x=\dfrac{1}{n^2-1},

        \begin{equation*} 		\left(1+\frac{1}{n^2-1}\right)^n\geq 1+\frac{n}{n^2-1}>1+\frac{1}{n}, 	\end{equation*}

    dove nell’ultimo passaggio abbiamo sfruttato la disuguaglianza \frac{n}{n^2-1}>\frac{1}{n}, che è vera perché equivale a n^2>n^2-1 per ogni n\geq 2.

  2.  

  3. Dalla definizione di a_n e b_n otteniamo:

        \begin{equation*} 		b_n=\left(1+\frac{1}{n}\right)^{n+1}=\left(1+\frac{1}{n}\right)^n\cdot\left(1+\frac{1}{n}\right)=a_n\cdot\left(1+\frac{1}{n}\right)>a_n 	\end{equation*}

    per ogni n\geq 1. Di conseguenza,

    (108)   \begin{equation*} 	b_1\geq b_n>a_n\geq a_1\qquad\forall n\geq 1. \end{equation*}

  4.  

  5. L’equazione precedente (108) dimostra quindi che a_n e b_n sono successioni monotone e limitate, quindi per il teorema 5.1 sono convergenti rispettivamente a \ell_a e \ell_b. Utilizzando la proposizione 5.5, otteniamo la seguente stima:

    (109)   \begin{equation*} 	a_n \leq \ell_a \leq \ell_b \leq b_n 	\qquad 	\forall n \in \mathbb{N}. \end{equation*}

    Dimostriamo ora che \ell_a = \ell_b. Per farlo, osserviamo che:

        \[ b_n = \left(1 + \frac{1}{n}\right)^{n+1} = \left(1 + \frac{1}{n}\right)^n \left(1 + \frac{1}{n}\right) = a_n \left(1 + \frac{1}{n}\right) = a_n + \frac{a_n}{n} \quad \forall n \in \mathbb{N}. \]

    Poiché a_n è limitata e la successione \dfrac{1}{n} tende a zero, per il teorema 4.6 abbiamo \dfrac{a_n}{n} \to 0, quindi:

        \[ \ell_b = \lim_{n \to +\infty} b_n = \lim_{n \to +\infty} a_n + \lim_{n \to +\infty} \frac{a_n}{n} = \ell_a + 0 = \ell_a. \]

    Pertanto, a_n e b_n convergono allo stesso limite e = \ell_a = \ell_b. Calcolando (109) per n = 10, otteniamo:

        \[ 2.59\dots = a_{10} < e < b_{10} = 2.85\dots. \]

    In particolare, 2 < e < 3.

Il limite comune delle due successioni del teorema 7.1 riveste notevole importanza in Matematica e nelle altre scienze e merita pertanto una definizione.

Definizione 7.2 (numero di Nepero). Il limite e delle successioni a_n e b_n del teorema 7.1 viene detto numero di Nepero:

(110)   \begin{equation*} 			e 			= 			\lim_{n \to + \infty} \left( 1+\frac{1}{n} \right )^n 			= 			\lim_{n \to + \infty} 			\left( 1+\frac{1}{n} \right )^{n+1}. 		\end{equation*}

    \[\quad\]

Come abbiamo anticipato, il numero di Nepero è molto importante, e vi sono numerose successioni che vi convergono. Tra di esse, vogliamo mostrare che e è il limite della successione c_n = \sum_{k=0}^n \frac{1}{k!}, ovvero le somme dei reciproci dei fattoriali dei numeri interi non negativi. Per dimostrare ciò, premettiamo le seguenti proprietà.

Lemma 7.3. Siano n,k \in \mathbb{N} con k \leq n. Allora si ha

  1. \binom{n}{k} \frac{1}{n^k} \leq \frac{1}{k!};
  2. \dfrac{1}{k!} 			\leq 			\dfrac{1}{2^{k-1}};
  3. \sum_{j=0}^n \dfrac{1}{2^j} 			= 			1 + \dfrac{1}{2} + \dfrac{1}{4} + \dots + \dfrac{1}{2^n} 			= 			2 - \dfrac{1}{2^n}.

    \[\quad\]

Dimostrazione.

  1. Dalla definizione di coefficiente binomiale, otteniamo:

        \begin{equation*} 			\binom{n}{k}\frac{1}{n^k} = \frac{n!}{(n-k)!\cdot k!}\cdot\frac{1}{n^k} = \frac{n\cdot(n-1)\cdot(n-2)\cdot\ldots\cdot (n-k+1)}{n^k}\cdot\frac{1}{k!} \leq \frac{1}{k!}, 		\end{equation*}

    dove nell’ultima disuguaglianza abbiamo usato il fatto che \frac{n\cdot(n-1)\cdot(n-2)\cdot\ldots\cdot (n-k+1)}{n^k} \leq 1.

  2.  

  3. Dimostriamo l’asserto per induzione:

        \[\quad\]

    • Per k=0, abbiamo che \frac{1}{0!} = 1 e \frac{1}{2^{-1}} = 2, quindi \frac{1}{0!} \leq \frac{1}{2^{-1}}.
    •  

    • Supponiamo vera la tesi per k, dimostriamo l’asserto per k+1:

          \begin{equation*} 				\frac{1}{(k+1)!} = \frac{1}{k!} \cdot \frac{1}{k+1} \leq \frac{1}{2^{k-1}}\frac{1}{k+1}. 			\end{equation*}

      Per ottenere la tesi, osserviamo che \dfrac{1}{k+1} \leq \dfrac{1}{2} se e solo se k+1 \geq 2, il che è vero per k\geq 1. Quindi possiamo concludere che \dfrac{1}{(k+1)!} \leq \dfrac{1}{2^k}.

  4.  

  5. Dimostriamo per induzione l’uguaglianza:

    (111)   \begin{equation*} 			\sum_{k=0}^{n}\left(\frac{1}{2}\right)^k = 2 - \frac{1}{2^n}. 		\end{equation*}

        \[\quad\]

    • Per n=0:

          \begin{equation*} 				\sum_{k=0}^{0}\frac{1}{2^k} = \frac{1}{2^0} = 1 = 2 - \frac{1}{2^0}. 			\end{equation*}

    •  

    • Supponiamo vera l’ipotesi per n e dimostriamo l’asserto per n+1:

          \begin{equation*} 				\sum_{k=0}^{n+1}\frac{1}{2^k} = \sum_{k=0}^{n}\frac{1}{2^k} + \frac{1}{2^{n+1}} = 2 - \frac{1}{2^n} + \frac{1}{2^{n+1}} = 2 + \frac{-2+1}{2^{n+1}} = 2 - \frac{1}{2^{n+1}}. 			\end{equation*}

    Per induzione, (111) è dimostrata.

Possiamo ora mostrare la seguente caratterizzazione del numero di Nepero.

Teorema 7.4 (caratterizzazione di e). Si ha

(112)   \begin{equation*} 			\lim_{n \to + \infty} \sum_{k=0}^n \frac{1}{k!} = e. 		\end{equation*}

    \[\quad\]

Dimostrazione. Consideriamo le successioni

(113)   \begin{equation*} 		a_n 		= 		\left( 1+\frac{1}{n} \right )^n, 		\quad 		b_n 		= 		\left( 1+\frac{1}{n} \right )^{n+1} 		\qquad 		\forall n \in \mathbb{N}, 	\end{equation*}

che sappiamo convergere a e per il teorema 7.1, e definiamo la successione \{c_n\}_{n \in \mathbb{N}} come

(114)   \begin{equation*} 		c_n = \sum_{k=0}^n \frac{1}{k!} 		\qquad 		\forall n \in \mathbb{N}. 	\end{equation*}

Dividiamo la dimostrazione in vari punti.

    \[\quad\]

  • c_n è crescente. Tale asserto è ovvio in quanto c_n è una somma di termini positivi:

    (115)   \begin{equation*} 			c_n 			= 			\sum_{k=0}^n \frac{1}{k!} 			< 			\frac{1}{(n+1)!} + \sum_{k=0}^n \frac{1}{k!} 			= 			\sum_{k=0}^{n+1} \frac{1}{k!} 			= 			c_{n+1} 			\qquad 			\forall n \in \mathbb{N}. 		\end{equation*}

  •  

  • c_n < 3 per ogni n \in \mathbb{N}. Si ha infatti

    (116)   \begin{equation*} 			c_n 			= 			\sum_{k=0}^n \frac{1}{k!} 			= 			1 + \sum_{k=1}^n \frac{1}{k!} 			\leq 			1 + \sum_{k=0}^{n-1} \frac{1}{2^{k}} 			= 			1 + 2 - \frac{1}{2^{n-1}} 			< 			3 			\qquad 			\forall n \in \mathbb{N}, 		\end{equation*}

    dove nella prima disuguaglianza abbiamo usato 2, mentre l’ultima uguaglianza segue da 3.

    Da questi due primi punti e dal teorema 5.1 segue quindi che c_n \to \ell_c, ossia che c_n è convergente con \ell_c \leq 3.

  •  

  • a_n \leq c_n per ogni n \in \mathbb{N}. Vale

    (117)   \begin{equation*} 			a_n 			= 			\left( 1+\frac{1}{n} \right )^n 			= 			\sum_{k=0}^n \binom{n}{k} \frac{1}{n^k} 			\leq 			\sum_{k=0}^n  \frac{1}{k!} 			= 			c_n 			\qquad 			\forall n \in \mathbb{N}, 		\end{equation*}

    dove nella seconda uguaglianza abbiamo usato la formula del binomio di Newton, mentre la disuguaglianza segue da 1. Da a_n \leq c_n e dalla proposizione 5.5, si ha quindi

    (118)   \begin{equation*} 			e \leq \ell_c. 		\end{equation*}

  •  

  • c_m \leq e per ogni m \in \mathbb{N}. Fissiamo m \in \mathbb{N}. Osserviamo innanzitutto che per ogni n \in \mathbb{N}, si ha

    (119)   \begin{equation*} 			\begin{split} 			a_n& 			= 			\sum_{k=0}^n \binom{n}{k} \frac{1}{n^k} 			\\ 			&= 			\sum_{k=0}^n \frac{n!}{(n-k)! k!} \frac{1}{n^k} 			\\ 			&= 			\sum_{k=0}^n \frac{1}{k!} \frac{n (n-1) \cdots (n-k+1)}{n^k} 			\\&= 			\sum_{k=0}^n \frac{1}{k!} \frac{n}{n} \cdot \frac{n-1}{n} \cdots \frac{n-k+1}{n}. 			\end{split} 		\end{equation*}

    Abbiamo quindi

    (120)   \begin{equation*} 			a_n 			\geq 			\sum_{k=0}^m \frac{1}{k!} \frac{n}{n} \cdot \frac{n-1}{n} \cdots \frac{n-k+1}{n} 			\qquad 			\forall n \geq m. 		\end{equation*}

    Si ha inoltre

    (121)   \begin{equation*} 			\lim_{n \to + \infty} \frac{n-j}{n} 			= 			\lim_{n \to +\infty} \left(1 - \frac{j}{n}\right) 			= 			1 			\qquad 			\forall j \in \{0,\dots,m\}. 		\end{equation*}

    Passando al limite per n \to + \infty in (120), si ha

    (122)   \begin{equation*} 			e 			= 			\lim_{n \to + \infty} a_n 			\geq 			\lim_{n \to + \infty} 			\left ( \sum_{k=0}^m \frac{1}{k!} \frac{n}{n} \cdot \frac{n-1}{n} \cdots \frac{n-k+1}{n} \right ) 			= 			\sum_{k=0}^m \frac{1}{k!} 			= 			c_m, 		\end{equation*}

    dove la prima uguaglianza segue dal teorema 7.1, la disuguaglianza segue da (120) e dalla proposizione 5.5, mentre la seconda uguaglianza segue da (121) e al punto 3 del teorema 4.1, grazie al fatto che m è fissato e i fattori in (120) sono in numero pari al massimo a m.

    Dato che c_m \leq e per ogni m \in \mathbb{N}, passando al limite per m \to + \infty in (122), di nuovo per la proposizione 5.5 otteniamo

    (123)   \begin{equation*} 		e \geq \lim_{m \to + \infty} c_m 		= 		\ell_c. 	\end{equation*}

    Tale disequazione, insieme a (118), prova che e= \ell_c, ossia che c_n \to e.

Osservazione 7.5. La stima a_n \leq c_n \leq 3 per ogni n \in \mathbb{N} fornisce un’altra dimostrazione del fatto che e \leq 3.    


  1. Un numero x si dice trascendente se non esiste alcun polinomio a coefficienti razionali che ha come radice x.

Forma indeterminata 1 alla infinito.

Il termine generale della successione a_n definita in (106) è costituito dal prodotto di n fattori pari a 1+\frac{1}{n}. Nonostante si abbia

(124)   \begin{equation*} 	\lim_{n \to +\infty} \left ( 1 + \frac{1}{n}\right) 	= 	1, \end{equation*}

per calcolarne il limite di a_n non si poteva usare il punto 3 del teorema 4.1 sul limite del prodotto, perché esso non è costituito da un numero fissato di fattori.

Notando che a_n e b_n sono costituiti da potenze in cui la base tende a 1 e l’esponente tende a +\infty, ci si può chiedere se tutte le successioni siffatte tendano a e. La risposta è negativa, come mostrano i prossimi esempi. Essi provano che

    \[\boxcolorato{analisi}{{ 		1^\infty \text{ è una forma indeterminata}. 	}}\]

Esempio 7.6. Consideriamo la successione x_n definita come segue:

(125)   \begin{equation*} 		x_n 		= 		\left ( 1 + \frac{1}{n}\right )^{n^2} 		\qquad 		\forall n \in \mathbb{N} 	\end{equation*}

e mostriamo che \displaystyle \lim_{n \to + \infty} x_n = + \infty. Osserviamo che

(126)   \begin{equation*} 		x_n 		= 		\left ( \left ( 1 + \frac{1}{n}\right )^{n} \right )^n 		\geq 		2^n 		\qquad 		\forall n \in \mathbb{N}, 	\end{equation*}

dove la disuguaglianza segue dal fatto che \left ( 1 + \dfrac{1}{n}\right )^{n} \geq 2 per ogni n \in \mathbb{N}, dimostrato nel teorema 7.1. Poiché 2^n \to +\infty come mostrato nell’esempio 3.9, per il teorema 5.10 abbiamo x_n \to + \infty.

Il lettore può trovare per esercizio altri esempi in cui il limite è un qualsiasi numero reale non negativo.


Esponenziale naturale.

In questa sezione, presenteremo il prossimo risultato.

Proposizione 7.7. Si ha

(127)   \begin{equation*} 			\lim_{n \to +\infty}\left (1+\frac{x}{n}\right )^n=e^x 			\qquad 			\forall x \in \mathbb{R}. 		\end{equation*}

Inoltre, se x>0, la successione \left (1+\dfrac{x}{n}\right )^n è crescente.

    \[\quad\]

Dimostrazione. Osserviamo che la tesi è già provata per x=1. Allo stesso modo in cui si è fatto nella dimostrazione del teorema 7.1, si dimostra che le due successioni definite da

(128)   \begin{equation*} 		a_n(x) 		= 		\left( 1+\frac{x}{n} \right )^n, 		\quad 		b_n(x) 		= 		\left( 1+\frac{x}{n} \right )^{n+1} 		\qquad 		\forall n \in \mathbb{N} 	\end{equation*}

sono rispettivamente crescenti e decrescenti8 e convergono allo stesso limite \ell(x). Occorre quindi soltanto provare che \ell(x)=e^x per ogni x \in \mathbb{R}.

    \[\quad\]

  • Nel caso in cui x sia un numero razionale, scriviamo x=\dfrac{m}{k} con m \in \mathbb{N} e k \in \mathbb{Z} \setminus \{ 0 \}. Per tutti gli n multipli di m, ossia tali che n=hm per qualche h \in \mathbb{N} si ha

    (129)   \begin{equation*} 			\left( 1+\frac{x}{n} \right )^n 			= 			\left (\left( 1+\frac{x}{n} \right )^{\frac{n}{x}} 			\right )^x 			= 			\left (\left( 1+\frac{1}{hk} \right )^{hk} 			\right )^x. 		\end{equation*}

    Poiché per h \to + \infty l’ultimo membro dell’uguaglianza tende a e^x per il teorema 7.1, allora la successione a_n(x) possiede una sottosuccessione convergente a e^x. Poiché a_n è convergente, per la proposizione 3.16 deve aversi

    (130)   \begin{equation*} 			\lim_{n \to +\infty} \left( 1+\frac{x}{n} \right )^n 			= 			e^x. 		\end{equation*}

  •  

  • x \in \mathbb{R} \setminus \mathbb{Q}. Supponiamo senza perdita di generalità che x >0 (l’altro caso è analogo). Per ogni q,r \in \mathbb{Q} tali che q < x < r, per la monotonia delle potenze si ha

    (131)   \begin{equation*} 			\left( 1+\frac{q}{n} \right )^n 			< 			\left( 1+\frac{x}{n} \right )^n 			< 			\left( 1+\frac{r}{n} \right )^n 			\qquad 			\forall n \in \mathbb{N}. 		\end{equation*}

    Passando ai limiti per n \to + \infty, per il punto precedente e la proposizione 5.5 si ha

    (132)   \begin{equation*} 			e^q 			\leq 			\ell(x) 			\leq 			e^r. 		\end{equation*}

    Poiché q e r sono arbitrari, scegliendo una successione q_k \to x e r_k \to x, per la definzione A.1 di potenza a esponente reale si ottiene

    (133)   \begin{equation*} 			\ell(x)=e^x. 		\end{equation*}

Ora, possiamo introdurre un utile corollario di questo teorema.

Corollario 7.8. Vale la disuguaglianza

(134)   \begin{equation*} 			e^x > 1+ x 			\qquad 			\forall x > 0. 		\end{equation*}

    \[\quad\]

Dimostrazione. Poiché la successione \left( 1+\dfrac{x}{n} \right )^n è strettamente crescente se x>0 per la proposizione 7.7, si ha

(135)   \begin{equation*} 		e^x 		= 		\lim_{n \to + \infty} \left( 1+\frac{x}{n} \right )^n 		> 		\left( 1+\frac{x}{1} \right )^1 		= 		1 + x 		\qquad 		\forall x >0. 	\end{equation*}

   


  1. Ciò vale soltanto definitivamente se x<0. Infatti, se x<0 ed è molto grande in modulo, la base 1 + \dfrac{x}{n} della potenza è negativa per i primi n \in \mathbb{N}, pertanto la potenza \left( 1+\dfrac{x}{n} \right )^n ha segno oscillante finché \frac{x}{n} non diventa in modulo minore di 1.

 

Criteri di convergenza

Introduzione.

In questa sezione esamineremo vari criteri che consentono di analizzare la convergenza e la regolarità di una successione.

Criterio del rapporto.

In questa sezione ci focalizziamo sul criterio del rapporto. Questo strumento fondamentale ci aiuta a determinare il limite di una successione positiva attraverso il calcolo del limite del rapporto tra due termini consecutivi, a condizione che tale limite esista.

Teorema 8.1 (criterio del rappporto). Sia \{a_n\}_{n\in\mathbb{N}} una successione a termini positivi tale che

    \begin{equation*} 			\lim_{n \rightarrow +\infty} \frac{a_{n+1}}{a_n}=\ell. 		\end{equation*}

    \[\quad\]

  1. Se \ell>1, allora

        \[\lim_{n \rightarrow +\infty}a_n=+\infty,\]

  2.  

  3. Se \ell<1, allora

        \[\lim_{n \rightarrow +\infty}a_n=0,\]

    \[\quad\]

Dimostriamo il teorema 8.1 come conseguenza della seguente proposizione, che esprime un risultato più forte.

Proposizione 8.2. Sia a_n una successione a termini positivi.

    \[\quad\]

  1. Se esiste \alpha>1 tale che \dfrac{a_{n+1}}{a_n} \geq \alpha definitivamente, allora \displaystyle\lim_{n \to + \infty} a_n = + \infty.
  2.  

  3. Se esiste \alpha \in (0,1) tale che \dfrac{a_{n+1}}{a_n} \leq \alpha definitivamente, allora \displaystyle\lim_{n \to + \infty} a_n = 0.

    \[\quad\]

Dimostrazione. Mostriamo separatamente i diversi casi.

    \[\quad\]

  1. Se \alpha>1 con \dfrac{a_{n+1}}{a_n} \geq \alpha definitivamente, allora esiste N \in \mathbb{N} tale che

    (136)   \begin{equation*} 			a_{n+1} \geq \alpha a_n 			\qquad 			\forall n \geq N. 		\end{equation*}

    Quindi a_{N+1} \geq \alpha a_N, a_{N+2} \geq \alpha a_{N+1} \geq \alpha^2 a_N. Generalizzando, abbiamo che

    (137)   \begin{equation*} 			a_{N+k} \geq \alpha^k a_N 			\qquad 			\forall k \in \mathbb{N}. 		\end{equation*}

    Poiché \alpha>1, per l’esempio 5.11 si ha \alpha^k \to + \infty. Quindi, grazie a (137) e al teorema 5.10 si ha

    (138)   \begin{equation*} 			\lim_{k \to + \infty} a_{N+k} = +\infty, 		\end{equation*}

    ossia a_n \to + \infty.

  2.  

  3. Analogamente al punto precedente, si ottiene che esiste N \in \mathbb{N} tale che

    (139)   \begin{equation*} 			0 \leq a_{N+k} \leq \alpha^k a_N 			\qquad 			\forall k \in \mathbb{N}. 		\end{equation*}

    Dato che \alpha \in (0,1), si ha \alpha^k \to 0. Da (139) e dal teorema 5.8 del confronto, si ha a_n \to 0.

Possiamo ora dimostrare il criterio del rapporto.

Dimostrazione del teorema 8.1. Se \dfrac{a_{n+1}}{a_n} \to \ell con \ell>1, scegliamo \alpha\in \mathbb{R} con 1 < \alpha < \ell. Per la definizione 3.2 \dfrac{a_{n+1}}{a_n}> \alpha definitivamente e quindi, per la proposizione 8.2 si ha a_n \to +\infty.

Analogamente, se \dfrac{a_{n+1}}{a_n} \to \ell con \ell\in (0,1), scegliamo \alpha \in \mathbb{R} con \ell < \alpha < 1. Poiché per la definizione 3.2 vale \dfrac{a_{n+1}}{a_n} < \alpha definitivamente, si ha a_n \to 0 per la proposizione 8.2.

Osserviamo che il criterio del rapporto non dà informazioni se \dfrac{a_{n+1}}{a_n} \to 1. Infatti consideriamo il seguente esempio.

Esempio 8.3. Siano a_n = n e b_n = \dfrac{1}{n}. È noto che a_n \to +\infty, mentre \dfrac{1}{n} \to 0. Tuttavia, osserviamo che:

(140)   \begin{gather*} 		\lim_{n \to + \infty} \frac{a_{n+1}}{a_n} 		= 		\lim_{n \to + \infty} \frac{n+1}{n} 		= 		\lim_{n \to + \infty} \left( 1 + \frac{1}{n} \right) 		= 		1, 		\\ 		\lim_{n \to + \infty} \frac{b_{n+1}}{b_n} 		= 		\lim_{n \to + \infty} \frac{n}{n+1} 		= 		\lim_{n \to + \infty} \left( 1 - \frac{1}{n+1} \right) 		= 		1. 	\end{gather*}

In entrambi i casi, il limite del rapporto dei termini successivi converge a 1, ma le caratteristiche delle successioni a_n e b_n sono nettamente diverse. Si invita il lettore a esplorare ulteriori esempi dove \dfrac{a_{n+1}}{a_n} \to 1, ma a_n \to \ell per un qualsiasi \ell \in (0,+\infty).


Criterio della radice.

Introduciamo ora un ulteriore criterio di convergenza: il criterio della radice. Questo metodo si basa sostanzialmente sul confronto dei termini di una successione con quelli della successione \sqrt[n]{a} nei casi a>1 e a \in (0,1).

Teorema 8.4 (criterio della radice). Sia \{a_n\}_{n\in\mathbb{N}} una successione a termini positivi tale che

    \begin{equation*} 			\lim_{n \rightarrow +\infty} \sqrt[n]{a_n}=\ell. 		\end{equation*}

    \[\quad\]

  1. Se \ell>1, allora

        \[\lim_{n \rightarrow +\infty}a_n=+\infty,\]

  2.  

  3. Se \ell<1, allora

        \[\lim_{n \rightarrow +\infty}a_n=0,\]

    \[\quad\]

Anche in questo caso, otteniamo il teorema 8.4 come corollario della proposizione seguente.

Proposizione 8.5. Sia a_n una successione a termini positivi.

  1. Se esiste \alpha>1 tale che \sqrt[n]{a_n} \geq \alpha definitivamente, allora \displaystyle\lim_{n \to + \infty} a_n = + \infty.
  2. Se esiste \alpha \in (0,1) tale che \sqrt[n]{a_n} \leq \alpha definitivamente, allora \displaystyle\lim_{n \to + \infty} a_n = 0.

    \[\quad\]

Dimostrazione. Analizziamo i diversi casi separatamente.

    \[\quad\]

  • 1 Consideriamo il caso in cui \alpha > 1 e \sqrt[n]{a_n} \geq \alpha definitivamente. Allora esiste un N \in \mathbb{N} tale che per ogni n \geq N si ha

    (141)   \begin{equation*} 		a_{n} \geq \alpha^n. 	\end{equation*}

    Dato che \alpha > 1, sappiamo dall’esempio 5.11 che \alpha^n \to + \infty. Quindi, applicando il teorema 5.10, concludiamo che

    (142)   \begin{equation*} 		\lim_{n \to + \infty} a_n = +\infty. 	\end{equation*}

  •  

  • 2 Nel caso in cui \sqrt[n]{a_n} \leq \alpha definitivamente con \alpha < 1, esiste un N \in \mathbb{N} per cui

    (143)   \begin{equation*} 		a_{n} \leq \alpha^n \quad \forall n \geq N. 	\end{equation*}

    Poiché \alpha < 1, e di nuovo per l’esempio 5.11, \alpha^n \to 0. Dunque, applicando il teorema 5.8, risulta che

    (144)   \begin{equation*} 		\lim_{n \to + \infty} a_n = 0. 	\end{equation*}

Possiamo ora dimostrare il teorema 8.4.

Dimostrazione del teorema 8.4. Se \sqrt[n]{a_n} \to \ell con \ell>1, scegliendo \alpha \in (1,\ell) per definizione di limite si ha definitivamente \sqrt[n]{a_n} \geq \alpha. Per la proposizione 8.5 vale quindi a_n \to +\infty.

Analogamente, se \sqrt[n]{a_n} \to \ell con \ell\in (0,1), allora fissando \alpha \in (\ell,1) per definizione di limite si ha a_n \leq \alpha definitivamente. Quindi a_n \to 0 di nuovo per la proposizione 8.5.

Osservazione 8.6. Anche il criterio della radice non fornisce informazioni nel caso in cui \sqrt[n]{a_n} \to 1. Esaminiamo i seguenti esempi per chiarire questo punto.

Esempio 8.7 (\sqrt[n]{q} \to 1). Sia a_n=q con q \in (1,+\infty) (il caso q \in (0,1) è analogo). Vale ovviamente a_n \to q in quanto la successione è costante. Mostriamo che \sqrt[n]{a_n}=\sqrt[n]{q} \to 1. Osserviamo innanzitutto che, per la monotonia della radice n-esima, si ha \sqrt[n]{q} >1 per ogni n \in \mathbb{N}.

Quindi si può scrivere \sqrt[n]{q}=1+(\sqrt[n]{q}-1) con \sqrt[n]{q}-1>0. Allora, applicando la disuguaglianza di Bernoulli (35) si ha

(145)   \begin{equation*} 		q 		= 		(1 +(\sqrt[n]{q} - 1))^n 		\geq 		1  + n(\sqrt[n]{q} - 1) 		\geq 		0 		\quad 		\iff 		\quad 		\frac{q-1}{n} 		\geq 		\sqrt[n]{q} - 1 		\geq 		0 		\qquad 		\forall n \in \mathbb{N}. 	\end{equation*}

Poichè \dfrac{q-1}{n} \to 0, per il teorema 5.8 vale anche \sqrt[n]{q}-1 \to 0, ovvero

(146)   \begin{equation*} 		\lim_{n \to +\infty} \sqrt[n]{q} 		=1. 	\end{equation*}

Quindi tutte le successioni costanti a_n=q soddisfano \sqrt[n]{a_n} \to 1, ma hanno limiti diversi.

Esempio 8.8. Consideriamo la successione a_n=2^{\sqrt{n}}. Fissando N \in \mathbb{N}, per la monotonia della funzione x \mapsto 2^x e poiché \sqrt{n} \to +\infty, si ha che 2^{\sqrt{n}} \geq 2^N definitivamente. Dato che N è arbitrario e \lim_{N \to +\infty}2^N=+\infty, deve aversi

(147)   \begin{equation*} 		\lim_{n \to +\infty} 2^{\sqrt{n}} 		= 		+\infty. 	\end{equation*}

D’altra parte si ha

(148)   \begin{equation*} 		\lim_{n \to + \infty} \sqrt[n]{2^{\sqrt{n}}} 		= 		\lim_{n \to + \infty} 2^{\frac{1}{\sqrt{n}}}. 	\end{equation*}

Per mostrare che tale limite è 1, osserviamo che la successione 2^{\frac{1}{\sqrt{n}}} è decrescente, quindi ha limite per il teorema 5.1. Inoltre, considerando la sottosuccessione n_j=j^2, si ha

(149)   \begin{equation*} 		2^{\frac{1}{\sqrt{n_j}}} 		= 		2^{\frac{1}{\sqrt{j^2}}} 		= 		2^{\frac{1}{j}} 		\qquad 		\forall j \in \mathbb{N} 	\end{equation*}

e

(150)   \begin{equation*} 		\lim_{j \to + \infty} 2^{\frac{1}{j}} 		= 		1 	\end{equation*}

per l’esempio 8.7. Dato che la successione \sqrt[n]{2^{\sqrt{n}}} ha limite e una sua estratta ha limite 1, allora essa ha globalmente limite 1 per il teorema 3.16. Dunque \sqrt[n]{a_n}= \sqrt[n]{2^{\sqrt{n}}} \to 1 ma a_n \to + \infty.


Confronto tra i criteri della radice e del rapporto.

Possiamo mostrare che il criterio della radice è più generale del criterio del rapporto: se il criterio del rapporto fornisce la convergenza o la divergenza di una successione, allora anche il criterio della radice lo fa. Mostriamo inoltre con un controesempio che il viceversa è falso: esistono successioni per cui il criterio del rapporto non dà informazioni sulla sua convergenza, mentre il criterio della radice fornisce la convergenza.

Proposizione 8.9. Sia a_n una successione a termini positivi e si supponga che

(151)   \begin{equation*} 			\lim_{n \to +\infty} \frac{a_{n+1}}{a_n} = \ell 		\end{equation*}

per qualche \ell \in [0,+\infty) \cup \{+\infty\}. Allora si ha anche

(152)   \begin{equation*} 			\lim_{n \to + \infty} \sqrt[n]{a_n} 			= 			\ell. 		\end{equation*}

    \[\quad\]

Dimostrazione. Consideriamo solo il caso \ell \in (0,+\infty) in quanto i casi \ell=0 e \ell=+\infty si ottengono in maniera del tutto analoga, con una versione “unilaterale” del seguente argomento.

Consideriamo \varepsilon>0 tale che 0 < \ell - 2\varepsilon.

Poiché \frac{a_{n+1}}{a_n} \to \ell esiste N \in \mathbb{N} tale che

(153)   \begin{equation*} 		\ell - \varepsilon < \frac{a_{n+1}}{a_n} < \ell + \varepsilon 		\qquad 		\forall n \geq N. 	\end{equation*}

Ciò vuol dire che

(154)   \begin{equation*} 		a_n(\ell - \varepsilon) < {a_{n+1}} < a_n(\ell + \varepsilon) 		\qquad 		\forall n \geq N 	\end{equation*}

Iterando questa disuguaglianza da N a n, otteniamo

(155)   \begin{equation*} 		a_N (\ell - \varepsilon)^{n-N} 		< 		a_{n} 		< 		a_N (\ell + \varepsilon)^{n-N} 		\qquad 		\forall n \geq N, 	\end{equation*}

ovvero

(156)   \begin{equation*} 		\frac{a_N}{(\ell - \varepsilon)^N} 		(\ell - \varepsilon)^n 		< 		a_{n} 		< 		\frac{a_N}{(\ell + \varepsilon)^N} 		(\ell + \varepsilon)^n 		\qquad 		\forall n \geq N. 	\end{equation*}

Calcolando la radice n-esima dei membri della disuguaglianza, si ottiene

(157)   \begin{equation*} 		(\ell - \varepsilon) 		\sqrt[n]{\frac{a_N}{(\ell - \varepsilon)^N}} 		< 		\sqrt[n]{a_n} 		< 		(\ell + \varepsilon) 		\sqrt[n]{\frac{a_N}{(\ell + \varepsilon)^N}} 		\qquad 		\forall n \geq N. 	\end{equation*}

Poiché i termini \frac{a_N}{(\ell - \varepsilon)^N} e \frac{a_N}{(\ell + \varepsilon)^N} sono positivi e non dipendono da n, per l’esempio 8.7 si ha

(158)   \begin{equation*} 		\lim_{n \to + \infty} 		(\ell - \varepsilon) 		\sqrt[n]{\frac{a_N}{(\ell - \varepsilon)^N}} 		= 		\ell - \varepsilon, 		\qquad 		\lim_{n \to + \infty} 		(\ell + \varepsilon) 		\sqrt[n]{\frac{a_N}{(\ell + \varepsilon)^N}} 		= 		\ell + \varepsilon. 	\end{equation*}

Per (157) e per la proposizione 5.4, si ha

(159)   \begin{equation*} 		\ell - 2\varepsilon 		\leq 		\sqrt[n]{a_n} 		\leq 		\ell + 2\varepsilon 		\qquad 		\text{definitivamente}. 	\end{equation*}

Per l’arbitrarietà di \varepsilon>0 si ha \sqrt[n]{a_n} \to \ell.

Mostriamo ora un esempio di successione la cui divergenza può essere stabilita con il criterio della radice, ma non con il criterio del rapporto.

Esempio 8.10. Consideriamo la successione a_n definita da

(160)   \begin{equation*} 		a_n 		= 		2^{n + (-1)^n} 		\qquad 		\forall n \in \mathbb{N}. 	\end{equation*}

La successione è chiaramente divergente in quanto

(161)   \begin{equation*} 		a_n 		= 		2^{n + (-1)^n} 		\geq 		2^{n-1} 		\qquad 		\forall n \in \mathbb{N} 	\end{equation*}

usando il teorema 5.10. Si ha però

(162)   \begin{equation*} 		\frac{a_{n+1}}{a_n} 		= 		\begin{cases} 			\dfrac{2^{n}}{2^{n+1}}=\dfrac{1}{2}				& \text{se $n$ è pari}\\[11pt] 			\dfrac{2^{n+2}}{2^{n-1}}=8	& \text{se $n$ è dispari}, 		\end{cases} 	\end{equation*}

che mostra che \lim_{n \to +\infty} \frac{a_{n+1}}{a_n} non esiste, quindi il criterio del rapporto non dà informazioni sul carattere della successione. Invece, volendo applicare il criterio della radice, si ottiene

(163)   \begin{equation*} 		\sqrt[n]{a_n} 		= 		2^{\frac{n + (-1)^n}{n}} 		= 		2^{1 + \frac{(-1)^n}{n}} 		= 		2 \cdot \sqrt[n]{2^{(-1)^n}} 		\qquad 		\forall n \in \mathbb{N}. 	\end{equation*}

Poiché la successione 2^{(-1)^n} è pari alternativamente a 2 e \dfrac{1}{2}, per l’esempio 8.7 si ha

(164)   \begin{equation*} 		\lim_{n \to + \infty} \sqrt[n]{2^{(-1)^n}} 		= 		1, 	\end{equation*}

da cui, inserendo in (163) otteniamo

(165)   \begin{equation*} 		\lim_{n \to +\infty} \sqrt[n]{a_n} 		= 		\lim_{n \to +\infty} 2 \cdot \sqrt[n]{2^{(-1)^n}} 		= 		2. 	\end{equation*}

Quindi il criterio della radice afferma che a_n è divergente.


 

Forma indeterminata \left [\frac{\infty}{\infty} \right ]

Introduzione.

In questa sezione riportiamo alcuni semplici esempi di come è possibile sfruttare i teoremi di convergenza per calcolare dei limiti. Inizieremo inoltre a enunciare i primi limiti notevoli, ovvero quei limiti che si presentano in una forma indeterminata e che poi, con specifiche dimostrazioni e/o criteri, vengono risolti. Dopo aver determinato il valore del limite di queste specifiche forme indeterminate è possibile, come per ogni dimostrazione, sfruttare il risultato e applicarlo a esercizi più complessi.

Forma indeterminata infinito su infinito.

Esempio 9.1. Calcolare

    \begin{equation*} 		\lim_{n \rightarrow +\infty} \frac{n^n}{n!}. 	\end{equation*}

Entrambe le successioni, quella al numeratore e quella al denominatore divergono a +\infty, quindi ci troviamo di fronte a un a forma indeterminata del tipo \left[\infty/\infty\right]. La divergenza di questa successione si potrebbe ottenere come conseguenza del teorema del confronto 5.10: si ha infatti

(166)   \begin{equation*} \frac{n^n}{n!} = \frac{n}{1} \cdot \frac{n}{2} \cdots \frac{n}{n-1} \cdot \frac{n}{n} > n, \end{equation*}

dove l’ultima disuguaglianza deriva dal fatto che il primo fattore è n, mentre tutti gli altri sono maggiori di 1. Poiché la successione n è divergente, per il teorema 5.10, tale è anche \frac{n^n}{n!}.

Mostriamo però anche un altro modo per ottenere tale risultato, che consente di dedurre delle conseguenze interessanti. Definitivamente la successione è a termini positivi, quindi possiamo applicare il criterio del rapporto 8.1 e ottenere

    \begin{equation*} 		\begin{split} 			\lim_{n \rightarrow +\infty}\frac{a_{n+1}}{a_n}&=\lim_{n \rightarrow +\infty} \frac{(n+1)^{n+1}}{(n+1)!}\cdot \frac{n!}{n^n}=\lim_{n \rightarrow +\infty} \frac{(n+1)^{n+1}}{n^n}\cdot \frac{\cancel{n!}}{(n+1)\cancel{n!}}=\\&=\lim_{n \rightarrow +\infty}\left(\frac{n+1}{n}\right)^n=\lim_{n \rightarrow +\infty}\left(1+\frac{1}{n}\right)^n=e>1, 		\end{split} 	\end{equation*}

e concludere che

    \[\boxcolorato{analisi}{ 		{ 		\lim_{n \rightarrow +\infty} \frac{n^n}{n!}=+\infty. 		}}\]

Questo per la proposizione 8.9 ci permette di concludere che

(167)   \begin{equation*} 	\lim_{n \to + \infty} \sqrt[n]{\frac{n^n}{n!}} 	= 	\lim_{n \to + \infty} \frac{n}{\sqrt[n]{n!}} 	= 	e. \end{equation*}

In particolare deve aversi

    \[\boxcolorato{analisi}{{ 		\lim_{n \to + \infty} \sqrt[n]{n!} 		= 		+\infty. 	}}\]

altrimenti il limite in (167) non potrebbe valere e.

Il risultato (167) dell’esempio 9.1 suggerisce di confrontare la successione \frac{n^n}{n!} con e^n, cioè suggerisce che esse tendano a +\infty in modo confrontabile. Infatti produciamo il prossimo risultato che è molto utile come stima del fattoriale, in maniera simile alla cosiddetta formula di Stirling.

Proposizione 9.2 (Stima del fattoriale). Vale

(168)   \begin{equation*} 			\frac{n^n}{e^{n-1}} 			\leq 			n! 			\leq 			\frac{n^n}{e^n} ne 			\qquad 			\forall n \in \mathbb{N}. 		\end{equation*}

    \[\quad\]

Dimostrazione. Consideriamo le successioni a_n e b_n definite come

(169)   \begin{equation*} 		a_n = \frac{n! e^{n}}{n^n}, \quad b_n = \frac{n! e^n}{n^{n+1}} \qquad \forall n \in \mathbb{N}. 	\end{equation*}

    \[\quad\]

  • a_n è crescente. Calcolando il rapporto \frac{a_{n+1}}{a_n}, dimostriamo che è maggiore di 1:

    (170)   \begin{equation*} 			\frac{a_{n+1}}{a_n} = \frac{e}{\left(1 + \frac{1}{n}\right)^n} > 1 \qquad \forall n \in \mathbb{N}, 		\end{equation*}

    per il teorema 7.1, che implica \left(1 + \frac{1}{n}\right)^n < e per ogni n \in \mathbb{N}.

  •  

  • b_n è decrescente. Analogamente, per b_n si ha

    (171)   \begin{equation*} 			\frac{b_{n+1}}{b_n} = \frac{e}{\left(1 + \frac{1}{n}\right)^{n+1}} < 1 \qquad \forall n \in \mathbb{N}, 		\end{equation*}

    grazie al 7.1, che stabilisce \left(1 + \frac{1}{n}\right)^{n+1} > e per ogni n \in \mathbb{N}.

    Quindi, dal fatto che a_n è crescente

    (172)   \begin{equation*} 		n! \frac{e^n}{n^n} > e \quad \forall n \in \mathbb{N}, 	\end{equation*}

    e dal fatto che b_n è decrescente si può dedurre che

    (173)   \begin{equation*} 		n! \frac{e^n}{n^{n+1}} < e \quad \forall n \in \mathbb{N}, 	\end{equation*}

    completando la dimostrazione.

    Osservazione 9.3 (formula di Stirling). L’approssimazione del fattoriale nella proposizione precedente può essere migliorata; vale infatti la seguente formula di Stirling sul comportamento asintotico del fattoriale, che ci limitiamo a enunciare:

        \[\boxcolorato{analisi}{{ 			n! 			\sim 			\frac{n^n}{e^n} \sqrt{2	\pi n}. 	}}\]

    Esempio 9.4. Calcoliamo

        \begin{equation*} 		\lim_{n \rightarrow +\infty} \frac{n!}{a^n}, 	\end{equation*}

    dove a>0 è un parametro fissato.

    Studiamo separatamente i casi:

        \[\quad\]

    • 0<a<1. Possiamo riscrivere la successione come

          \begin{equation*} 			\frac{n!}{a^n}=n!\cdot \left(\frac{1}{a}\right)^n\qquad\forall n\in\mathbb{N}, 		\end{equation*}

      che risulta divergente a +\infty perché è prodotto di due successioni divergenti a +\infty.

    •  

    • a=1. Si ha

          \begin{equation*} 			\lim_{n\to+\infty} \frac{n!}{a^n}=\lim_{n\to+\infty} n!=+\infty. 		\end{equation*}

    •  

    • a>1. Entrambe le successioni al numeratore e al denominatore divergono a +\infty, quindi ci troviamo di fronte a un a forma indeterminata del tipo \left[\infty/\infty\right].

      La successione è a termini positivi, quindi possiamo applicare il criterio del rapporto 8.1 e ottenere

          \begin{equation*}	 			\lim_{n \rightarrow +\infty}\frac{a_{n+1}}{a_n}=\lim_{n \rightarrow +\infty}\frac{(n+1)!}{a^{n+1}}\cdot\frac{a^n}{n!}=\lim_{n \rightarrow +\infty} \frac{(n+1)\cancel{n!}}{\cancel{n!}}\cdot \frac{1}{a}. 		\end{equation*}

      Allora possiamo concludere che

          \[\boxcolorato{analisi}{{ 				\lim_{n \rightarrow +\infty} \frac{n!}{a^n}=+\infty, 			}}\]

      poiché \displaystyle\lim_{n \to \infty} (n+1)=+\infty e a>0.

    In conclusione, per ogni a>0 il limite richiesto vale +\infty.

    Esempio 9.5. Calcoliamo

        \begin{equation*} 		\lim_{n \rightarrow +\infty} \frac{a^n}{n^b}, 	\end{equation*}

    dove a>1 e b\in\mathbb{R} sono parametri fissati. Come per gli esempi precedenti possiamo applicare il criterio del rapporto 8.1 a questa forma indeterminata \left[\infty/\infty\right].

        \begin{equation*} 		\begin{split} 			\lim_{n \rightarrow +\infty}\frac{a_{n+1}}{a_n}&=\lim_{n \rightarrow +\infty} \frac{a^{n+1}}{(n+1)^b}\cdot\frac{n^b}{a^n}=\lim_{n \rightarrow +\infty}\left(\frac{n}{n+1}\right)^b\cdot a=\\&=\lim_{n \rightarrow +\infty} \left(\frac{n+1-1}{n+1}\right)^b\cdot a=\lim_{n \rightarrow +\infty}\left(1-\frac{1}{n}\right)^b\cdot a=a>1. 		\end{split} 	\end{equation*}

    Per il criterio del rapporto 8.1 possiamo concludere che

        \[\boxcolorato{analisi}{{ 				\lim_{n \rightarrow +\infty} \frac{a^n}{n^b}=+\infty. 		}}\]

    Possiamo generalizzare l’esempio precedente.

    Esempio 9.6 (\lim_{n \to + \infty} \frac{a^{x_n}}{x_n^\beta}). con x_n \to +\infty).Se x_n è una successione, mostriamo che

        \[\boxcolorato{analisi}{{ 			x_n \to + \infty 			\quad 			\Rightarrow 			\quad 			\lim_{n \to +\infty} \frac{a^{x_n}}{x_n^\beta} 			= 			+\infty 			\qquad 			\forall a >1,\,\, 			\forall \beta \in \mathbb{R}. 		}}\]

    Osserviamo che, poiché x_n\to +\infty, essa è definitamente positiva per il teorema di permanenza del segno 5.3 e quindi la successione è definitivamente ben definita.

    Procediamo ora suddividendo la dimostrazione in base ai diversi casi relativi all’esponente \beta.

        \[\quad\]

    • \beta<1. Scrivendo a=e^{\log a}, si ottiene

      (174)   \begin{equation*} 			\frac{a^{x_n}}{x_n^\beta} 			= 			\frac{e^{x_n \log a}}{x_n^\beta} 			> 			\frac{x_n \log a}{x_n^\beta} 			= 			x_n^{1-\beta} \log a 			\qquad 			\forall n \in \mathbb{N}, 		\end{equation*}

      dove la disuguaglianza deriva dal corollario 7.8. Poiché l’esponente 1-\beta>0 e x_n \to + \infty, si ha x_n^{1-\beta} \to + \infty. Per il teorema del confronto 5.10, si ha

      (175)   \begin{equation*} 			\lim_{n \to + \infty} \frac{a^{x_n}}{x_n^\beta} 			= 			+ \infty. 		\end{equation*}

    •  

    • \beta \geq 1. Possiamo facilmente ricondurci al caso precedente fissando N \in \mathbb{N} tale che \dfrac{\beta}{N}< 1 e scrivendo

      (176)   \begin{equation*} 			\frac{a^{x_n}}{x_n^\beta} 			= 			\frac{\Big( a^{\frac{x_n}{N}} \Big)^N}{\Big(x_n^{\frac{\beta}{N}} \Big)^N} 			= 			\left ( 			\frac{\Big(a^{\frac{1}{N}} \Big)^{x_n} }{x_n^{\frac{\beta}{N}} } 			\right )^N 			\qquad 			\forall n \in \mathbb{N}. 		\end{equation*}

      Poiché ora l’esponente al denominatore è \dfrac{\beta}{N}<1, si può usare il punto precedente e ottenere che

      (177)   \begin{equation*} 			\frac{\Big(a^{\frac{1}{N}} \Big)^{x_n} }{x_n^{\frac{\beta}{N}} } 			\to + \infty. 		\end{equation*}

      Per il teorema 4.1, si ha quindi

      (178)   \begin{equation*} 			\dfrac{a^{x_n}}{x_n^\beta} 			= 			\left ( 			\frac{a^{\frac{x_n}{N}} }{x_n^{\frac{\beta}{N}} } 			\right )^N 			\to + \infty. 		\end{equation*}

    Esempio 9.7. Mostriamo che

        \[\boxcolorato{analisi}{{ 			x_n \to + \infty 			\quad 			\Rightarrow 			\quad 			\lim_{n \to +\infty} \frac{\log_a x_n}{x_n^\beta} 			= 			0 			\qquad 			\forall a \in (0,1) \cup (1,+\infty),\,\, 			\forall \beta >0. 		}}\]

    Analogamente al caso precedente la successione x_n\to +\infty, essa è definitamente positiva per il teorema di permanenza del segno 5.3 e quindi la successione è definitivamente ben definita.

        \[\quad\]

    • a \in (0,1). In questo caso, il risultato è immediato: \log_a x_n \to 0 mentre x_n^\beta \to +\infty, pertanto il rapporto è infinitesimo in virtù del teorema 4.1.
    •  

    • a > 1 e \beta = 1. Scegliamo \varepsilon > 0 arbitrario. Utilizzando le proprietà dei logaritmi, otteniamo

      (179)   \begin{equation*} 			\frac{\log_a x_n}{x_n} < \varepsilon 			\iff 			\log_a x_n < \varepsilon x_n 			\stackrel{a>1}{\iff} 			x_n < a^{\varepsilon x_n} 			\iff 			\frac{(a^\varepsilon)^{x_n}}{x_n} > 1. 		\end{equation*}

      Questa disuguaglianza si verifica definitivamente poiché x_n \to +\infty, applicando l’esempio 9.6 con a^\varepsilon al posto di a e \beta = 1 (notiamo che a^\varepsilon > 1 poiché a > 1 e \varepsilon > 0). Pertanto, per l’arbitrarietà di \varepsilon, concludiamo che

      (180)   \begin{equation*} 			\lim_{n \to +\infty} \frac{\log_a x_n}{x_n} 			= 			0. 		\end{equation*}

    • a >1 e \beta>0. Nel caso generale, osserviamo che possiamo definire una successione t_n\coloneqq x_n^\beta che soddisfa nuovamente t_n \to +\infty in quanto \beta>0 e possiamo scrivere

      (181)   \begin{equation*} 			\frac{\log_a x_n}{x_n^\beta} 			= 			\frac{\log_a t_n^{\frac{1}{\beta}}}{t_n} 			= 			\frac{1}{\beta} \frac{\log_a t_n}{t_n}. 		\end{equation*}

      Possiamo quindi applicare il punto precedente con \beta=1 con t_n al posto di x_n e ottenere la tesi.

    Esempio 9.8 (\sqrt[n]{n} \to 1). Mostriamo che

        \[\boxcolorato{analisi}{{ 			\lim_{n \to + \infty} \sqrt[n]{n} 			= 			1. 		}}\]

    Consideriamo la successione a_n=n. Poiché si ha

    (182)   \begin{equation*} 		\lim_{n \to + \infty} 		\frac{a_{n+1}}{a_n} 		= 		\lim_{n \to + \infty} 		\frac{n+1}{n} 		= 		1, 	\end{equation*}

    per la proposizione 8.9 si ha anche

    (183)   \begin{equation*} 		\lim_{n \to + \infty} \sqrt[n]{a_n} 		= 		\lim_{n \to + \infty} \sqrt[n]{n} 		= 		1. 	\end{equation*}

    Esempio 9.9. Verifichiamo in modo alternativo a quanto fatto in 9.8 che

        \begin{equation*} 		\lim_{n \rightarrow +\infty} \sqrt[n]{n}=1. 	\end{equation*}

    Dalla definizione di logaritmo possiamo riscrivere \sqrt[n]{n} come e^{\log \sqrt[n]{n}} e ottenere

        \begin{equation*} 		\lim_{n \rightarrow +\infty} \sqrt[n]{n}=\lim_{n \rightarrow +\infty} e^{\log \sqrt[n]{n}}=\lim_{n \rightarrow +\infty} e^{\log (n)^\frac{1}{n}}=\lim_{n \rightarrow +\infty} e^{\frac{\log n}{n}}=1. 	\end{equation*}

    dove l’ultima uguaglianza vale per l’esempio 9.7.


Gerarchia degli infiniti.

Le successioni che abbiamo incontrato negli esempi precedenti sono tutte divergenti a +\infty e possono essere “ordinate” in una lista in cui il rapporto tra il termine successivo e il precedente tende a infinito:

    \[\boxcolorato{analisi}{{ 		\log_a n,\, n^b,\, a^n,\,n!,\, n^n\qquad a>1,\,b>0. 	}}\]

Tale lista viene informalmente denominata gerarchia degli infiniti.


Forma indeterminata infinito alla 0.

Grazie all’esempio 9.8 possiamo mostrare che

    \[\boxcolorato{analisi}{{ 		\infty^0 \text{ è una forma indeterminata.} 	}}\]

Infatti l’esempio 9.8 mostra un caso in cui tale forma ha limite 1. Facciamo vedere che esistono esempi in cui il limite è pari a +\infty o a qualche altro numero reale.

Esempio 9.10. Consideriamo la successione definita da a_n=n, che possiamo scrivere come

(184)   \begin{equation*} 		a_n=n= (n^n)^{\frac{1}{n}} 		\qquad 		\forall n \in \mathbb{N}, 	\end{equation*}

ossia come un limite del tipo \infty^0. Chiaramente però a_n è una successione divergente.

Esempio 9.11. Sulla falsariga dell’esempio precedente, consideriamo la successione costante a_n=c, dove c>1 è fissato. Come sopra possiamo riscriverla come

(185)   \begin{equation*} 		a_n= 		c 		= 		(c^n)^{\frac{1}{n}}, 	\end{equation*}

che è di nuovo della forma \infty^0 (si osservi che c^n \to + \infty perché c>1 per l’esempio 5.11). Essendo a_n costantemente pari a c, essa tende a c.


Forma indeterminata 0 alla 0.

Osserviamo che, dagli esempi precedenti, facilmente si ottiene che

    \[\boxcolorato{analisi}{{ 		0^0 \text{ è una forma indeterminata.} 	}}\]

Infatti, consideriamo le seguenti successioni.

Esempio 9.12. Mostriamo tre esempi di successioni del tipo a_n^{b_n} con a_n \to 0, b_n \to 0, ma tali che a_n^{b_n} hanno limiti diversi.

    \[\quad\]

  • Poniamo a_n=b_n=\dfrac{1}{n}. Si ha

    (186)   \begin{equation*} 			\lim_{n \to +\infty}  a_n^{b_n} 			= 			\lim_{n \to +\infty} \left (\frac{1}{n} \right )^{\frac{1}{n}} 			= 			\lim_{n \to +\infty} \frac{1}{\sqrt[n]{n}} 			= 			1, 		\end{equation*}

    dove l’ultima uguaglianza segue dall’esempio 9.8.

  •  

  • Si consideri ora a_n=\frac{1}{n^n} e b_n=\frac{1}{n}. Si ha

    (187)   \begin{equation*} 			\lim_{n \to +\infty}  a_n^{b_n} 			= 			\lim_{n \to +\infty} \left (\frac{1}{n^n} \right )^{\frac{1}{n}} 			= 			\lim_{n \to +\infty} \frac{1}{{n}} 			= 			0. 		\end{equation*}

  •  

  • Poniamo infine a_n=c^n con c \in (0,1) e b_n=\frac{1}{n}. Per l’esempio 5.11, poiché c \in (0,1), vale a_n \to 0, quindi a_n^{b_n} è una forma del tipo 0^0. D’altra parte si ha

    (188)   \begin{equation*} 			\lim_{n \to +\infty}  a_n^{b_n} 			= 			\lim_{n \to +\infty} \left (c^n \right )^{\frac{1}{n}} 			= 			\lim_{n \to +\infty} c 			= 			c. 		\end{equation*}


 

Confronti asintotici

Leggi...

Consideriamo il caso in cui due successioni a_n e b_n siano differenti, ma al limite mostrino un comportamento simile. Per esempio, prendiamo

(189)   \begin{equation*} 	a_n = \frac{1}{n}, 	\quad 	b_n = \frac{1}{n} + \frac{1}{n^3} 	\qquad 	\forall n \in \mathbb{N}. \end{equation*}

Sebbene le due successioni siano diverse, poiché b_n = a_n + \frac{1}{n^3}, il termine aggiuntivo \frac{1}{n^3} diventa trascurabile all’infinito, essendo molto più piccolo in modulo rispetto a \frac{1}{n}. Questo fenomeno può essere descritto con la seguente definizione.

Definizione 10.1 (successioni asintotiche). Due successioni \{a_n\}_{n\in\mathbb{N}} e \{b_n\}_{n\in\mathbb{N}} di numeri reali con b_n\neq 0 definitivamente, si dicono asintotiche se

    \begin{equation*} 			\lim_{n \rightarrow +\infty}\frac{a_n}{b_n}=1. 		\end{equation*}

In tal caso scriveremo che a_n\sim b_n per n\rightarrow +\infty.

Esempio 10.2. Consideriamo la successione polinomiale a_n, definita come

(190)   \begin{equation*} 		a_n=\sum_{k=0}^{p}c_k n^k\qquad\forall n\in\mathbb{N}, 	\end{equation*}

dove p>0 e c_p\neq 0. In questo caso, si può affermare che a_n è asintoticamente equivalente a c_p n^p. Infatti:

    \begin{equation*} \lim_{n \rightarrow +\infty} \dfrac{c_p n^p+c_{p-1} n^{p-1}+\dots+ c_1 n+c_0}{c_p n^p}=\lim_{n \rightarrow +\infty}\dfrac{\cancel{ n^p}\left(c_p+\dfrac{c_{p-1}}{n}+\dots+ \dfrac{c_1}{n^{p-1}}+\dfrac{c_0}{n^p}\right)}{c_p \cancel{n^p}}=1. 	\end{equation*}

Questo metodo di analisi può essere esteso a successioni simili a quella definita in 10.2 ma con esponenti razionali, dimostrando che la successione si comporta asintoticamente come la potenza di n con l’esponente più alto.

La precedente discussione serve a confrontare successioni che assumono un comportamento simile per n \to +\infty. Risulta molto utile introdurre una ulteriore notazione per indicare invece che una successione a_n è trascurabile rispetto a un’altra successione b_n al limite per n \to +\infty.

Definizione 10.3 (o-piccolo). Siano \{a_n\}_{n\in\mathbb{N}} e \{b_n\}_{n\in\mathbb{N}} due successioni a valori in \mathbb{R} tale che b_n\neq 0 definitivamente. Diremo che a_n è o-piccolo di b_n se vale

    \begin{equation*} 			\lim_{n \rightarrow +\infty}\frac{a_n}{b_n}=0. 		\end{equation*}

In tal caso scriveremo che a_n=o(b_n) per n\rightarrow +\infty.

    \[\quad\]

Osservazione 10.4. Se a_n è una successione infinitesima, allora a_n=o(1), infatti

    \begin{equation*} 		\lim_{n \rightarrow +\infty}a_n=0\Rightarrow \,\lim_{n\to+\infty}\frac{a_n}{1}=0. 	\end{equation*}

Quindi dire che una successione a_n è un o-piccolo di b_n vuol dire che la successione a_n risulta in qualche maniera trascurabile rispetto alla successione b_n.

Esempio 10.5. Nell’esempio 9.1 abbiamo dimostrato che

    \begin{equation*} 		\lim_{n \rightarrow +\infty} \frac{n^n}{n!}=+\infty, 	\end{equation*}

ovvero per il punto 3 della proposizione 4.4

    \begin{equation*} 		\lim_{n \rightarrow +\infty} \frac{n!}{n^n}=0, 	\end{equation*}

quindi n!=o(n^n) per n\to+\infty o, come scriveremo spesso negli esercizi, \frac{n!}{n^n}=o(1).

Esempio 10.6. Nell’esempio 9.4 abbiamo dimostrato che

    \begin{equation*} 		\lim_{n \rightarrow +\infty} \frac{n!}{a^n}=+\infty, 	\end{equation*}

per a>1, ovvero per il punto 3 della proposizione 4.4

    \begin{equation*} 		\lim_{n \rightarrow +\infty} \frac{a^n}{n!}=0, 	\end{equation*}

quindi a^n=o(n!) per n\to+\infty o, come scriveremo spesso negli esercizi, \frac{a^n}{n!}=o(1).

Esempio 10.7. Nell’ esempio 9.5 abbiamo dimostrato che

    \begin{equation*} 		\lim_{n \rightarrow +\infty} \frac{a^n}{n^b}=+\infty, 	\end{equation*}

per a>1 e b\in\mathbb{R}, ovvero per il punto 3 della proposizione 4.4

    \begin{equation*} 		\lim_{n \rightarrow +\infty} \frac{n^b}{a^n}=0, 	\end{equation*}

quindi n^b=o(a^n) per n\to+\infty o, come sfrutteremo spesso negli esercizi, \dfrac{n^b}{a^n}=o(1).

Esempio 10.8. Nell’esempio 9.7 abbiamo dimostrato che

    \begin{equation*} 		\lim_{n \rightarrow +\infty} \frac{\log_an}{n^b}=0, 	\end{equation*}

per a>1 e b>0, quindi \log_an=o(n^b) per n\to+\infty o, come scriveremo spesso negli esercizi, \frac{\log_an}{n^b}=o(1).

Il prossimo risultato chiarisce come due successioni asintotiche siano essenzialmente equivalenti nel calcolo di un limite.

Teorema 10.9 (proprietà delle successioni asintotiche). Siano \{a_n\}_{n\in\mathbb{N}} e \{b_n\}_{n\in\mathbb{N}} due successioni; allora valgono le seguenti proprietà.

    \[\quad\]

  1. Se a_n\sim b_n e \displaystyle\lim_{n \rightarrow +\infty}b_n=\ell\in\mathbb{R}, allora \displaystyle\lim_{n \rightarrow +\infty}a_n=\ell\in\mathbb{R}.
  2.  

  3. Se a_n\sim b_n e b_n\sim c_n, allora a_n\sim c_n.
  4. (Principio di sostituzione.) Siano a_n, b_n, c_n\neq 0 definitivamente; se a_n\sim b_n, allora per ogni successione c_n si ha

        \begin{equation*} 				a_n\cdot c_n\sim b_n\cdot c_n,\qquad\frac{a_n}{c_n}\sim \frac{b_n}{c_n},\qquad\frac{c_n}{a_n}\sim\frac{c_n}{b_n}. 			\end{equation*}

    \[\quad\]

Dimostrazione.

  1. Poiché a_n e b_n sono due successioni asintotiche è sufficiente notare che

        \begin{equation*} 		\lim_{n \rightarrow +\infty}a_n=\lim_{n \rightarrow +\infty}\frac{a_n}{b_n}\cdot b_n=1\cdot \ell=\ell 	\end{equation*}

    sfruttando l’osservazione precedente e il teorema 4.1.

  2.  

  3. Per le stesse proprietà

        \begin{equation*} 		\lim_{n \rightarrow +\infty}\frac{a_n}{c_n}=\lim_{n \rightarrow +\infty}\frac{a_n}{b_n}\cdot\frac{b_n}{c_n}=1\cdot 1=1. 	\end{equation*}

  4.  

  5. Il principio di sostituzione è una semplice conseguenza della definizione di successioni asintotiche:

        \begin{equation*} 		\lim_{n \rightarrow +\infty} \frac{a_n\cdot c_n}{b_n\cdot c_n}=\lim_{n \rightarrow +\infty}\frac{a_n}{b_n}=1, 	\end{equation*}

    quindi a_n\cdot c_n\sim b_n\cdot c_n. Analogamente

        \begin{equation*} 		\lim_{n \rightarrow +\infty} \frac{\dfrac{a_n}{c_n}}{\dfrac{b_n}{c_n}}=\lim_{n \rightarrow +\infty}\frac{a_n}{c_n}\cdot \frac{c_n}{b_n}=\lim_{n \rightarrow +\infty}\frac{a_n}{b_n}=1. 	\end{equation*}

    Infine

        \begin{equation*} 		\lim_{n \rightarrow +\infty} \frac{\dfrac{c_n}{a_n}}{\dfrac{c_n}{b_n}}=\lim_{n \rightarrow +\infty}\dfrac{c_n}{a_n}\cdot \dfrac{b_n}{c_n}=\lim_{n \rightarrow +\infty}\dfrac{b_n}{a_n}=1. 	\end{equation*}

Osservazione 10.10. Non vale il viceversa della prima proprietà descritta dal teorema precedente, ovvero esistono delle successioni che hanno lo stesso limite, ma non sono asintotiche, ad esempio se consideriamo a_n=n^2 e b_n=n. Sappiamo che

    \begin{equation*} 		\lim_{n \rightarrow +\infty} a_n=\lim_{n \rightarrow +\infty} b_n=+\infty, 	\end{equation*}

ma

    \begin{equation*} 		\lim_{n \rightarrow +\infty}\frac{a_n}{b_n}=\lim_{n \rightarrow +\infty} \frac{n^2}{n}=\lim_{n \to \infty} n=+\infty, 	\end{equation*}

quindi le due successioni non sono asintotiche.

Seguono alcuni esempi di calcolo di limiti di successioni fratte che hanno come ulteriore obiettivo quello di mostrare come la forma \frac{\infty}{\infty} sia indeterminata.

Esempio 10.11. Calcoliamo

    \begin{equation*} 		\lim_{n \rightarrow +\infty}\frac{n^2+4}{2n^2+3}. 	\end{equation*}

Dall’esempio 10.2 sappiamo che n^2+4\sim n^2 e 2n^2+3\sim 2n^2; allora per il principio di sostituzione 3 nel teorema 10.9

    \begin{equation*} 		\lim_{n \rightarrow +\infty}\frac{n^2+4}{2n^2+3}=\lim_{n \rightarrow +\infty} \frac{n^2}{2n^2}=\lim_{n \rightarrow +\infty}\frac{1}{2}=\frac{1}{2}. 	\end{equation*}

Esempio 10.12. Calcoliamo

    \begin{equation*} 		\lim_{n \rightarrow +\infty}\frac{n^3+2n+1}{4n^4+4n^2+3}. 	\end{equation*}

Dall’esempio 10.2 sappiamo che n^3+2n+1\sim n^3 e 4n^4+4n^2+3\sim 4n^4; allora grazie al principio di sostituzione 3 nel teorema 10.9

    \begin{equation*} 		\lim_{n \rightarrow +\infty}\frac{n^3+2n+1}{4n^4+4n^2+3}=\lim_{n \rightarrow +\infty} \frac{n^3}{4n^4}=\lim_{n \rightarrow +\infty}\frac{1}{4n}=0. 	\end{equation*}

Esempio 10.13. Calcoliamo

    \begin{equation*} 		\lim_{n \rightarrow +\infty}\frac{n+4+3n\sqrt{n}}{2n-4\sqrt{n}+3}. 	\end{equation*}

Poiché n\sqrt{n}=n\cdot n^{1/2}=n^{3/2}, allora sempre per l’esempio 10.2 possiamo concludere che n+4+3n\sqrt{n}\sim 3n\sqrt{n} e 2n-4\sqrt{n}+3\sim 2n. Grazie al principio di sostituzione 3 del teorema 10.9

    \begin{equation*} 		\lim_{n \rightarrow +\infty}\frac{n+4+3n\sqrt{n}}{2n-4\sqrt{n}+3}=\lim_{n \rightarrow +\infty} \frac{3n\sqrt{n}}{2n}=\lim_{n \rightarrow +\infty} \frac{3\sqrt n}{2}=+\infty. 	\end{equation*}

Osservazione 10.14. Possiamo quindi concludere che se a_n e b_n sono due funzioni polinomiali di grado rispettivamente h e k, allora

    \begin{equation*} 		\lim_{n \to + \infty}\frac{a_n}{b_n}=\begin{cases} 			0&\text{ se }h<k\\ 			\infty&\text{ se }h>k\\ 			\dfrac{a_h}{b_k}&\text{ se }h=k. 		\end{cases} 	\end{equation*}

Come illustrato nell’esempio 10.2, questo ragionamento può essere generalizzato anche per le successioni che contengono potenze di n con esponenti razionali.


 

I teoremi di Stolz-Cesaro

Leggi...

Le pagine successive sono dedicate al teorema di Stolz-Cesàro, che è una versione discreta del teorema di De L’Hôpital, e ai relativi corollari. Iniziamo con un risultato fondamentale che afferma essenzialmente che se il rapporto tra due successioni a_n e b_n tende a \ell, allora anche il rapporto tra le somme dei primi n termini tenderà a \ell.

Teorema 11.1. Siano \{a_n\}_{n\in\mathbb{N}} e \{b_n\}_{n\in\mathbb{N}} due successioni tali che

    \[\quad\]

  1. b_n>0 per ogni n\in\mathbb{N};
  2. \displaystyle\lim_{n \rightarrow +\infty}\sum_{k=1}^{n} b_k=+\infty;
  3.  

  4. \displaystyle\lim_{n \rightarrow +\infty} \frac{a_n}{b_n}=\ell\qquad\ell\in\mathbb{R}\cup\{+\infty,\,-\infty\}.

Allora

    \begin{equation*} 			\lim_{n \rightarrow +\infty} \frac{\sum_{k=1}^{n}a_k}{\sum_{k=1}^{n}b_k}=\ell. 		\end{equation*}

    \[\quad\]

Dimostrazione. Supponiamo \ell\in\mathbb{R}; per la generalizzazione al caso \ell\in\{+\infty,\,-\infty\} basterà considerare solo un verso delle disuguaglianze di questa dimostrazione.

Per ipotesi, fissato un valore \varepsilon>0 esiste n_1\in\mathbb{N} tale che

    \begin{equation*} 		\left|\frac{a_n}{b_n}-\ell\right|<\varepsilon\Longrightarrow\, \ell-\varepsilon<\frac{a_n}{b_n}<\ell+\varepsilon\qquad\forall n\geq n_1. 	\end{equation*}

Poiché sempre per ipotesi la successione b_n è positiva, si ha

    \begin{equation*} 		(\ell-\varepsilon)b_n<a_n<(\ell+\varepsilon)b_n\qquad\forall n\geq n_1, 	\end{equation*}

e quindi per n\geq n_1

    \begin{equation*} 		(\ell-\varepsilon)\sum_{k=n_1}^{n}b_k<\sum_{k=n_1}^{n}a_k<(\ell+\varepsilon)\sum_{k=n_1}^{n}b_k. 	\end{equation*}

Aggiungendo ai membri delle disuguaglianze la quantità \sum_{k=1}^{n_1-1}a_k, otteniamo

    \begin{equation*} 		\sum_{k=1}^{n_1-1}a_k+(\ell-\varepsilon)\sum_{k=n_1}^{n}b_k<\sum_{k=1}^{n}a_k<\sum_{k=1}^{n_1-1}a_k+(\ell+\varepsilon)\sum_{k=n_1}^{n}b_k, 	\end{equation*}

ovvero

    \begin{equation*} 		\sum_{k=1}^{n_1-1}(a_k-(\ell-\varepsilon)b_k)+(\ell-\varepsilon)\sum_{k=1}^{n}b_k<\sum_{k=1}^{n}a_k<\sum_{k=1}^{n_1-1}(a_k-(\ell+\varepsilon)b_k)+(\ell+\varepsilon)\sum_{k=1}^{n}b_k. 	\end{equation*}

Dividendo i membri della disuguaglianza per \sum_{k=1}^{n}b_k>0, otteniamo

(191)   \begin{equation*} 		\frac{\sum_{k=1}^{n_1-1}(a_k-(\ell-\varepsilon)b_k)}{\sum_{k=1}^{n}b_k}+\ell-\varepsilon<\frac{\sum_{k=1}^{n}a_k}{\sum_{k=1}^{n}b_k}<\frac{\sum_{k=1}^{n_1-1}(a_k-(\ell+\varepsilon)b_k)}{\sum_{k=1}^{n}b_k}+\ell+\varepsilon. 	\end{equation*}

Poiché \sum_{k=1}^{n}b_k\longrightarrow +\infty per n\to +\infty, allora

    \begin{equation*} 		\frac{\sum_{k=1}^{n_1-1}(a_k-(\ell-\varepsilon)b_k)}{\sum_{k=1}^{n}b_k}\rightarrow 0,\qquad\qquad\qquad \frac{\sum_{k=1}^{n_1-1}(a_k-(\ell+\varepsilon)b_k)}{\sum_{k=1}^{n}b_k}\rightarrow 0, 	\end{equation*}

quindi è possibile determinare n_2\geq n_1 tale che

    \begin{equation*} 		\begin{split} 			&\left|\frac{\sum_{k=1}^{n_1-1}(a_k-(\ell-\varepsilon)b_k)}{\sum_{k=1}^{n}b_k}\right|<\varepsilon\Rightarrow \frac{\sum_{k=1}^{n_1-1}(a_k-(\ell-\varepsilon)b_k)}{\sum_{k=1}^{n}b_k}>-\varepsilon\qquad\forall n\geq n_2\\& \left|\frac{\sum_{k=1}^{n_1-1}(a_k-(\ell+\varepsilon)b_k)}{\sum_{k=1}^{n}b_k}\right|<\varepsilon\Rightarrow \frac{\sum_{k=1}^{n_1-1}(a_k-(\ell+\varepsilon)b_k)}{\sum_{k=1}^{n}b_k}<\varepsilon\qquad\forall n\geq n_2. 		\end{split} 	\end{equation*}

Usando le disuguaglianze (191) otteniamo che

    \begin{equation*} 		\ell-2\varepsilon<\frac{\sum_{k=1}^{n}a_k}{\sum_{k=1}^{n}b_k}<\ell+2\varepsilon\qquad\forall n\geq n_2. 	\end{equation*}

Per l’arbitrarietà di \varepsilon otteniamo la tesi.

Osservazione 11.2. Consideriamo due successioni

    \begin{equation*} 		\begin{split} 			&a_n=\left(\frac{1}{2}\right)^n\\ 			&b_n=\begin{cases} 				2&\text{ se }n=1\\ 				a_n&\text{ se }n>1. 			\end{cases} 		\end{split} 	\end{equation*}

Verifichiamo l’importanza dell’ipotesi 2 con un controesempio. Ovviamente b_n>0 per ogni n\in\mathbb{N} e

    \begin{equation*} 	\lim_{n \to + \infty}\frac{a_n}{b_n}=1, \end{equation*}

ma

    \begin{equation*} 	\lim_{n \to +\infty} \sum_{k=1}^{n} b_k=\frac{5}{2}, \end{equation*}

dove abbiamo sfruttato l’uguaglianza \sum_{n=0}^{+\infty} q^n=\dfrac{1}{1-q} se |q|<1. In questo caso

    \begin{equation*} 	\lim_{n \rightarrow +\infty} \frac{\sum_{k=1}^{n}a_k}{\sum_{k=1}^{n}b_k}=\frac{1}{\dfrac{5}{2}}=\frac{2}{5}\neq 1. \end{equation*}

Corollario 11.3. Sia \{a_n\}_{n\in\mathbb{N}} una successione convergente a \ell; allora anche la media aritmetica dei suoi termini converge a \ell, ovvero

    \begin{equation*} 			\lim_{n \rightarrow +\infty}\frac{\sum_{k=1}^{n}a_k}{n}=\ell. 		\end{equation*}

    \[\quad\]

Dimostrazione. Consideriamo la successione b_n=1 per ogni n\in\mathbb{N}, allora

    \[\quad\]

  1. b_n>0 per ogni n\in\mathbb{N},/li>  

  2. \displaystyle\lim_{n \to \infty} \sum_{k=1}^{n} b_n=\lim_{n \to \infty} \sum_{k=1}^{n} 1=\lim_{n \to \infty} n=+\infty./li>  

  3. \lim_{n \to + \infty}\dfrac{a_n}{b_n}=\lim_{n \to + \infty}a_n=\ell.

Allora per il teorema 11.1

    \begin{equation*} 		\lim_{n \rightarrow +\infty}\frac{\sum_{k=1}^{n}a_n}{\sum_{k=1}^{n}b_n}=\lim_{n \rightarrow +\infty}\frac{\sum_{k=1}^{n}a_n}{n}=\ell. 	\end{equation*}

Esempio 11.4. Il viceversa del corollario 11.3 è falso. Consideriamo la successione a_n=(-1)^n: dall’esempio 3.20 sappiamo che essa non ammette limite, mentre la successione della media aritmetica dei suoi termini soddisfa

    \begin{equation*} 		\frac{\sum_{k=1}^{n}a_k}{n}=\frac{\sum_{k=1}^{n}(-1)^n}{n}=\begin{cases} 			\dfrac{-1}{n}&\text{per }n\text{ dispari}\\\\ 			0&\text{per }n\text{ pari} 		\end{cases} 	\end{equation*}

e quindi tende a 0.

Enunciamo e dimostriamo un ulteriore teorema di Stolz-Cesaro, che formalizza un’idea che intuitivamente può essere descritta come segue: se consideriamo due successioni di numeri reali a_n e b_n tali che i rapporti tra gli incrementi tendeno a un valore \ell allora è ragionevole aspettarsi che il rapporto tra i termini n-esimi delle due successioni tenderà a \ell.

Teorema 11.5 (Stolz-Cesaro). Siano \{a_n\}_{n\in\mathbb{N}} e \{b_n\}_{n\in\mathbb{N}} due successioni di numeri reali. Se

    \[\quad\]

  1. b_n>0 per ogni n\in\mathbb{N},
  2.  

  3. b_n è strettamente crescente,
  4.  

  5. b_n è illimitata,
  6.  

  7. esiste \ell\in\mathbb{R}\cup\{+\infty,\,-\infty\} tale che

        \begin{equation*} 				\lim_{n \rightarrow +\infty}\frac{a_{n+1}-a_n}{b_{n+1}-b_n}=\ell, 			\end{equation*}

allora

    \begin{equation*} 			\lim_{n \rightarrow +\infty}\frac{a_n}{b_n}=\ell. 		\end{equation*}

    \[\quad\]

Dimostrazione. Poniamo per semplicità a_0=0 e b_0=0 e definiamo le due nuove successioni

    \begin{equation*} 		a_n'=a_{n}-a_{n-1},\qquad\qquad\qquad b_n'=b_n-b_{n-1}, 	\end{equation*}

per n\geq 1. Allora

(192)   \begin{equation*} 		\begin{split} 			&\sum_{k=1}^{n}a_k'=\sum_{k=1}^{n}(a_k-a_{k-1})=a_n-a_0=a_n\qquad\forall n\in\mathbb{N},\\& 			\sum_{k=1}^{n}b_k'=\sum_{k=1}^{n}(b_k-b_{k-1})=b_n-b_0=b_n\qquad\forall n\in\mathbb{N}. 		\end{split} 	\end{equation*}

In particolare la successione \{b_n'\} ha le seguenti proprietà:

    \[\quad\]

  • b_n'>0 per ogni n\in\mathbb{N} perché per ipotesi b_n è strettamente crescente.
  •  

  • Per (192) \displaystyle \lim_{n \rightarrow +\infty}\sum_{k=1}^{n}b_k'=\lim_{n \rightarrow +\infty} b_n=+\infty perché la successione \{b_n\}_{n\in\mathbb{N}} è strettamente crescente e illimitata.

Inoltre

    \begin{equation*} 		\lim_{n \rightarrow +\infty}\frac{a_{n+1}-a_n}{b_{n+1}-b_n}=\lim_{n \rightarrow +\infty} \frac{a_n'}{b_n'}=\ell, 	\end{equation*}

quindi per il corollario 11.3 si ha

    \begin{equation*} 		\lim_{n \rightarrow +\infty} \frac{\sum_{k=1}^{n}a_k'}{\sum_{k=1}^{n}b_k'}=\lim_{n \rightarrow +\infty}\frac{a_n}{b_n}=\ell. 	\end{equation*}

Esempio 11.6. Consideriamo le due successioni

    \begin{equation*} 		\begin{split} 			&a_n=\sum_{k=1}^{n}\frac{k}{k+1}\\ 			&b_n=n 		\end{split} 	\end{equation*}

per ogni n\in\mathbb{N}. Ovviamente la successione b_n è positiva, strettamente crescente e illimitata. Inoltre

    \begin{equation*} 	\lim_{n \to + \infty}\frac{a_{n+1}-a_n}{b_{n+1}-b_n}=\lim_{n \to + \infty}\frac{\sum_{k=1}^{n+1}\frac{k}{k+1}-\sum_{k=1}^{n}\frac{k}{k+1}}{n+1-n}=\lim_{n \to + \infty} \frac{n+1}{n+2}=1. \end{equation*}

Allora per il teorema 11.5 possiamo concludere che

    \begin{equation*} 	\lim_{n \to + \infty}\frac{\sum_{k=1}^{n}\frac{k}{k+1}}{n}=1. \end{equation*}

Il viceversa del teorema 11.5 è in generale falso, ossia non si può asserire l’esistenza del limite \displaystyle\lim_{n \to +\infty} \frac{a_{n+1}- a_{n}}{b_{n+1}- b_{n}} basandosi sull’esistenza di \displaystyle\lim_{n \to +\infty} \frac{a_n}{b_n}, come mostra il seguente esempio.

Esempio 11.7. Consideriamo le successioni a_n,b_n definite da

(193)   \begin{equation*} 		a_n=(-1)^n, 		\quad 		b_n=n 		\qquad 		\forall n \in \mathbb{N}. 	\end{equation*}

b_n è positiva, crescente, illimitata e vale

(194)   \begin{equation*} 		\lim_{n \to +\infty} \frac{a_n}{b_n} 		= 		\lim_{n \to +\infty} \frac{(-1)^n}{n} 		= 		0. 	\end{equation*}

Tuttavia il limite

(195)   \begin{equation*} 		\lim_{n \to +\infty} \frac{a_{n+1}- a_{n}}{b_{n+1}- b_{n}} 		= 		\lim_{n \to +\infty} \frac{(-1)^{n+1}-(-1)^n}{(n+1)-n} 		= 		\lim_{n \to +\infty} 2\cdot (-1)^{n+1} 	\end{equation*}

non esiste.

Corollario 11.8. Sia \{a_n\}_{n\in\mathbb{N}} una successione tale che

    \begin{equation*} 			\lim_{n \rightarrow +\infty}(a_{n+1}-a_n)=\ell, 		\end{equation*}

con \ell\in\mathbb{R}\cup\{+\infty,\,-\infty\}. Allora

    \begin{equation*} 			\lim_{n \rightarrow +\infty}\frac{a_n}{n}=\ell. 		\end{equation*}

    \[\quad\]

Dimostrazione. Consideriamo la successione b_n=n, allora questa è positiva, strettamente crescente e illimitata. Inoltre

    \begin{equation*} 		\lim_{n \rightarrow +\infty}\frac{a_{n+1}-a_n}{b_{n+1}-b_n}=\lim_{n \rightarrow +\infty}\frac{a_{n+1}-a_n}{n+1-n}=\lim_{n \rightarrow +\infty} (a_{n+1}-a_n)=\ell, 	\end{equation*}

quindi per il teorema 11.5 si ha

    \begin{equation*} 		\lim_{n \rightarrow +\infty} \frac{a_n}{b_n}=\lim_{n \rightarrow +\infty}\frac{a_n}{n}=\ell. 	\end{equation*}

Corollario 11.9. Sia \{a_n\}_{n\in\mathbb{N}} una successione positiva tale che \displaystyle\lim_{n\to +\infty}a_n=\ell\in [0;\infty)\cup\{+\infty\}. Allora

    \begin{equation*} 			\lim_{n \rightarrow +\infty}\sqrt[n]{\prod_{k=1}^{n}a_k}=\ell. 		\end{equation*}

    \[\quad\]

Dimostrazione. Consideriamo la successione b_n=\log a_n. Si ha

(196)   \begin{equation*} 		\frac{1}{n} \sum_{k=1}^{n}b_k 		= 		\frac{1}{n} \sum_{k=1}^{n}\log a_k 		= 		\log \left ( \left (\prod_{k=1}^{n} a_k \right )^{\frac{1}{n}} \right ) 	\end{equation*}

Per le proprietà di continuità dei logaritmi si ha

(197)   \begin{equation*} 		\lim_{n \to +\infty}b_n= 		\begin{cases} 			-\infty 		& \text{se $\ell=0$} 			\\ 			\log \ell		& \text{se $\ell\in (0,+\infty)$} 			\\ 			+\infty			& \text{se $\ell=+\infty$} 		\end{cases} 	\end{equation*}

e chiamiamo B tale limite. Applicando il corollario 11.3 alla successione b_n e tenendo conto di (196) si ottiene

(198)   \begin{equation*} 		B 		= 		\lim_{n \to +\infty} \frac{1}{n} \sum_{k=1}^{n}b_k 		= 		\lim_{n \to +\infty} 		\log \left ( \left (\prod_{k=1}^{n} a_k \right )^{\frac{1}{n}} \right ) 	\end{equation*}

Dalle proprietà della funzione esponenziale, inversa del logaritmo, si ha quindi

(199)   \begin{equation*} 		\ell = \lim_{n \to +\infty} 		\sqrt[n]{\prod_{k=1}^{n} a_k } 	\end{equation*}


 

Il teorema ponte

Leggi...

Il teorema ponte stabilisce l’equivalenza tra il calcolo del limite di una funzione f in un punto x_0 e il calcolo dei limiti delle successioni f(x_n) con x_n successione avente limite x_0. Presentiamo qui una dimostrazione concisa che sfrutta gli intorni, compresi quelli dei punti all’infinito, introdotti nella sezione 1.1. Una dimostrazione dettagliata dei diversi casi può essere consultata in [8, Il Teorema Ponte].

Teorema 12.1 (teorema ponte). Sia A \subseteq \mathbb{R}, sia f \colon A \to \mathbb{R}, sia x_0 \in \mathbb{R} \cup \{-\infty,+\infty\} un punto di accumulazione per A e sia \ell \in \mathbb{R} \cup \{-\infty,+\infty\}. Allora sono equivalenti le seguenti affermazioni:

  1. \displaystyle 				\lim_{x \to x_0} f(x) 				=\ell;
  2. per ogni successione x_n a valori in A tale che x_n \to x_0 e x_n \neq x_0 definitivamente, si ha

    (200)   \begin{equation*} 				\lim_{n \to + \infty} f(x_n) 				= 				\ell. 			\end{equation*}

    \[\quad\]

Dimostrazione. Dimostriamo separatamente che 1 implica 2 e poi che 2 implica 1.

    \[\quad\]

  • 1 \Rightarrow 2. Sia x_n una successione a valori in A tale che x_n \to x_0 e x_n \neq x_0 definitivamente. Per mostrare che f(x_n) \to \ell, fissiamo un qualunque intorno J di \ell e ricordiamo che, per definizione di limite di funzione esiste un intorno I di x_0 tale che

    (201)   \begin{equation*} 			f(x) \in J 			\qquad 			\forall x \in A \cap I \setminus \{x_0\}. 		\end{equation*}

    Poiché x_n \to x_0 e x_n \neq x_0 definitivamente, esiste N \in \mathbb{N} tale che

    (202)   \begin{equation*} 			x_n \in A \cap I \setminus \{x_0\} 			\qquad 			\forall n \geq N. 		\end{equation*}

    Unendo (202) e (201), si ottiene

    (203)   \begin{equation*} 			f(x_n) \in J 			\qquad 			\forall n \geq N. 		\end{equation*}

    Per l’arbitrarietà dell’intorno J di \ell, vale che f(x_n) \to \ell.

  •  

  • 2 \Rightarrow 1. Supponiamo per assurdo che \lim_{x \to x_0} f(x) \neq \ell, cioè che esiste un intorno J di \ell tale che ogni intorno I di x_0 contenga x \in I \setminus \{x_0\} tale che f(x) \notin J e costruiamo una successione x_n soddisfacente le ipotesi in 2 ma tale che f(x_n) non converga a \ell.

    A tal fine, consideriamo la famiglia di intorni di x_0 seguente:

    (204)   \begin{equation*} 			I_n 			= 			\begin{cases} 				\left ( x_0 - \dfrac{1}{n}, x_0 + \dfrac{1}{n} \right )					& \text{se } x_0 \in \mathbb{R}\\[10pt] 				(n,+\infty)			& \text{se } x_0=+\infty\\[6pt] 				(-\infty,-n)		& \text{se } x_0=-\infty 			\end{cases} 			\qquad 			\forall n \in \mathbb{N}. 		\end{equation*}

    Per quanto detto, per ogni n \in \mathbb{N} esiste x_n \in A \cap I_n \setminus \{x_0\} tale che

    (205)   \begin{equation*} 			f(x_n) \notin J 			\qquad 			\forall n \in \mathbb{N}. 		\end{equation*}

    Chiaramente la successione x_n è a valori in A e vale x_n \neq x_0 per ogni n \in \mathbb{N}. Mostriamo ora che x_n \to x_0; ciò segue però dalla definizione (204) degli intorni I_n:

    (206)   \begin{equation*} 			\begin{cases} 				x_0 - \dfrac{1}{n} < x_n < x_0 + \dfrac{1}{n}					& \text{se } x_0 \in \mathbb{R}\\[10pt] 				x_n > n		& \text{se } x_0=+\infty\\[6pt] 				x_n < -n		& \text{se } x_0=-\infty 			\end{cases} 			\qquad \forall n \in \mathbb{N}. 		\end{equation*}

    In tutti i casi infatti, per i teoremi del confronto 5.8 e 5.10, si ha che x_n \to x_0. D’altra parte, per (205) ovviamente la successione f(x_n) non ha limite \ell. Ciò contraddice l’ipotesi 2 e tale assurdo dimostra quindi 1.

Quindi, se conosciamo il limite della funzione per x\to \ell, possiamo essere sicuri che, se al posto della variabile x sostituiamo i termini di una successione che tende a \ell e calcoliamo il limite per n\to+\infty, il valore del limite stesso non cambia. Mostriamo ora qualche esempio di applicazione del teorema 12.1

Esempio 12.2.

  1. Sappiamo dai limiti notevoli per le funzioni di variabile reale che

        \begin{equation*} 		\lim_{x\to 0}\frac{\sin x}{x}=1. 	\end{equation*}

    Adesso grazie al teorema 12.1 siamo in grado di affermare che,

    (207)   \begin{equation*} \boxcolorato{analisi}{{ 			a_n \to 0 \,\,\, \text{e} \,\,\, a_n \neq 0 \text{ definitivamente} 			\quad 			\Longrightarrow 			\quad 			\lim_{n \to + \infty} \frac{\sin a_n}{a_n}=1. 		}}\end{equation*}

    Ad esempio se consideriamo a_n=\frac{1}{n}, si ha

        \begin{equation*} 		\lim_{n \to +\infty} n\cdot\sin \left(\dfrac{1}{n}\right)=\lim_{n \to +\infty} \frac{\sin \left(\dfrac{1}{n}\right)}{\dfrac{1}{n}}=1. 	\end{equation*}

  2. Un altro limite notevole per le funzioni di variabile reale è

    (208)   \begin{equation*} 		\lim_{x\to 0}\frac{1-\cos x}{x^2}=1. 	\end{equation*}

    Adesso grazie al teorema 12.1 siamo in grado di affermare che

        \[\boxcolorato{analisi}{{ 			a_n \to 0 \,\,\, \text{e} \,\,\, a_n \neq 0 \text{ definitivamente} 			\quad 			\Longrightarrow 			\quad 			\lim_{n \to + \infty} \frac{1 - \cos a_n}{a_n^2}= \frac{1}{2}. 		}}\]

  3. Sempre grazie al teorema ponte 12.1 e ai limiti notevoli per le funzioni reali di variabile reale sappiamo che

        \[\boxcolorato{analisi}{{ 			a_n \to 0 \,\,\, \text{e} \,\,\, a_n \neq 0 \text{ definitivamente} 			\quad 			\Longrightarrow 			\quad 			\lim_{n \to + \infty} \frac{e^{a_n}-1}{a_n}= 1. 		}}\]

  4. Analogamente a quanto detto per i limiti precedenti,

    (209)   \begin{equation*}\boxcolorato{analisi}{{ 			a_n \to 0 \,\,\, \text{e} \,\,\, a_n \neq 0 \text{ definitivamente} 			\quad 			\Longrightarrow 			\quad 			\lim_{n \to + \infty} \frac{\log (1+ a_n)}{a_n}= 1. 		}}\end{equation*}

Per le dimostrazioni dei limiti notevoli di funzioni sfruttati in questo esempio si rimanda a [7, teorema 93].


 

Successioni definite per ricorrenza

Leggi...

In molte situazioni, ad esempio per descrivere eventi che si ripetono a tempi discreti e la cui evoluzione dipende dagli stati precedenti, è utile considerare le successioni definite per ricorrenza (dette anche ricorsive) il cui termine n-esimo è definito da una funzione dei termini precedenti. Come primo esempio, possiamo considerare una successione espressa come:

    \begin{equation*} 	\begin{cases} 		a_0=\alpha\\ 		a_n=f(n,a_{n-1}),\quad\forall n\geq 1 	\end{cases} \end{equation*}

dove \alpha è un numero reale e f:\mathbb{N}\times\mathbb{R}\rightarrow \mathbb{R} è una funzione.

Esempio 13.1. La successione

    \begin{equation*} 		\begin{cases} 			a_0=0\\ 			a_n=2a_{n-1}+1\qquad\text{ per } n\geq 1 		\end{cases} 	\end{equation*}

è definita per ricorrenza. A partire dal primo termine è possibile determinare i termini successivi

    \begin{equation*} 		a_1=1,\qquad a_2=3,\qquad a_3=7,\qquad a_4=15 	\end{equation*}

e così via.

Diamo la seguente definizione.

Definizione 13.2 (successione definita per ricorrenza). Una successione definita per ricorrenza (o o ricorsiva) è la successione definita da

    \begin{equation*} 			\begin{cases} 				a_0,\,\dots,\,a_k&\text{ dati}\\ 				a_{n+1}=f(n,a_n,\dots,\,a_{n-k})&\text{ per } n\geq k 			\end{cases} 		\end{equation*}

dove k\in\mathbb{N} e f:D\subseteq\mathbb{N}\times \mathbb{R}^{k+1}\rightarrow\mathbb{R} è una funzione.

Esempio 13.3. Un classico esempio di successione definita per ricorrenza è quella che permette la costruzione dei numeri di Fibonacci:

    \begin{equation*} 		\begin{cases} 			a_0=0\\a_1=1\\a_n=a_{n-1}+a_{n-2}&\qquad \text{ per }n\geq 2. 		\end{cases} 	\end{equation*}

Come nell’esempio 13.1 è possibile determinare i termini successivi a partire da a_0 e a_1:

    \begin{equation*} 		a_2=1,\qquad a_3=2,\qquad a_4=3,\qquad a_5=5 	\end{equation*}

e così via.

Per la maggior parte delle successioni ricorsive è difficile determinare un’espressione analitica del temine generale, perciò è generalmente impossibile affrontare il loro studio con gli strumenti finora sviluppati per le successioni classiche. Si rimanda a [9, Successioni definite per ricorrenza — Esercizi misti] per numerosi esempi sulle diverse strategie per lo studio di questo tipo di successioni. In queste pagine cerchiamo di enunciare i risultati principali.

Esempio 13.4. Consideriamo la successione ricorsiva

    \begin{equation*} 		\begin{cases} 			a_0=0\\ 			a_n=\sqrt{1-3a_{n-1}}\qquad\text{ per }n\geq 1. 		\end{cases}. 	\end{equation*}

Affinché tale espressione definisca effettivamente una successione, gli elementi della successione devono soddisfare la condizione

    \[1-3a_{n-1}\geq 0\qquad \text{ per ogni } n\in\mathbb{N}\setminus\{0\},\]

ovvero a_{n-1}\leq \frac{1}{3}. Ma, calcolando i primi termini della successione, vediamo che dovrebbe aversi

    \begin{equation*} 		a_1=1,\qquad a_2=\sqrt{1-3}=\sqrt{-2}, 	\end{equation*}

che è ovviamente impossibile in \mathbb{R}.

Una volta stabilito che l’espressione data definisca effettivamente una successione per ricorrenza, sorge la questione di studiarne l’eventuale limite. Tale problema è generalmente affrontato in due passi distinti: si stabilisce inizialmente l’eventuale esistenza del limite, e successivamente lo si determina con ragionamenti ulteriori.

Esempio 13.5. Determiniamo se la successione definita per ricorrenza

    \begin{equation*} 		\begin{cases} 			a_0=1\\\\ 			a_{n+1}=\displaystyle\frac{2}{a_{n}} \qquad\text{ per }n\geq 0 		\end{cases} 	\end{equation*}

è convergente, divergente o indeterminata. Osserviamo che il termine generale a_{n+1} sia ben definito per ogni n\in\mathbb{N}. Infatti, dall’espressione che definisce a_{n+1}, esso esiste se e solo se a_n\neq 0. Dimostriamo per induzione che ciò è vero per ogni n\in\mathbb{N}:

    \[\quad\]

  • Passo base: per a_0\neq 0 per definizione;
  •  

  • Passo induttivo: per ipotesi induttiva a_n\neq 0, allora a_{n+1}=\frac{2}{a_n}\neq 0.

Proviamo adesso a calcolare esplicitamente i primi termini per capirne l’andamento

    \begin{equation*} 	a_0=1,\qquad a_1=2,\qquad a_2=1,\qquad a_3=2,\qquad a_4=1,\dots \end{equation*}

In questo modo possiamo ipotizzare che le due sottosuccessioni estratte, la prima dei soli termini dispari, la seconda dei soli termini par, possano essere riscritte come

    \begin{equation*} 	a_{2k+1}=2\qquad a_{2k}=1\qquad\forall k\in\mathbb{N}. \end{equation*}

Se riusciamo a dimostrarlo possiamo concludere che la successione di partenza è indeterminata. Se infatti per assurdo a_n ammettesse limite, allora ogni sottosuccessione estratta convergerebbe a tale limite per la proposizione 3.16.

Dimostriamo per induzione che a_{2k+1}=2 per ogni k\in\mathbb{N}. La dimostrazione per la sottosuccessione dei termini pari sarà analoga.

    \[\quad\]

  • Passo base: per a_1=\frac{2}{a_0}=\frac{2}{1}=2;
  •  

  • Passo induttivo: supponiamo vera la tesi per k e dimostriamolo per k+1:

        \begin{equation*} 		a_{2(k+1)+1}=a_{2k+3}=\frac{2}{a_{2k+2}}=\frac{2}{\dfrac{2}{a_{2k+1}}}=a_{2k+1}=2. 	\end{equation*}

Esempio 13.6. Calcoliamo \displaystyle\lim_{n\to+\infty}a_n dove a_n è la successione definita per ricorrenza

    \begin{equation*} 	\begin{cases} 		a_0=\alpha\in\mathbb{R}\\\\ 		a_{n+1}=a_n^2\qquad\text{ per }n\geq 0. 	\end{cases} \end{equation*}

Il termine generale a_n è ben definito per ogni n\in\mathbb{N} in quanto il termine successivo è il quadrato del precedente e tale operazione è sempre lecita. Determiniamo i primi valori della successione

    \begin{equation*} 	a_0=\alpha,\qquad a_1=\alpha^2,\qquad a_2=(\alpha^2)^2=\alpha^4,\qquad a_3=(\alpha^4)^2=\alpha^8,\dots \end{equation*}

Da questo possiamo dedurre che a_{n}=\alpha^{2^{n}} per ogni n\in\mathbb{N} (è facilmente dimostrabile per induzione). Inoltre, dato che a_n=\alpha^{2^{n}}=(\alpha^2)^{2^{n-1}} e 2^{n-1}\to+\infty per n\to+\infty, possiamo concludere per l’osservazione 5.12 che

    \begin{equation*} 	\lim_{n\to+\infty}a_n=\begin{cases} 		+\infty &\text{ se }|\alpha|>1\\ 		1&\text{ se }|\alpha|=1\\ 		0&\text{ se }|\alpha|<1. 	\end{cases} \end{equation*}

Nell’esempio appena esaminato è stato possibile determinare un’espressione esplicita del termine generale della successione e ciò ha consentito di calcolarne agevolmente il limite, utilizzando gli strumenti precedentemente sviluppati. Tale circostanza è però un caso fortuito e anzi, in generale, risulta impossibile determinare l’espressione esplicita del termine generale di una successione definita per ricorrenza. In tale maggioranza dei casi, occorre far leva su altre tecniche al fine di stabilire l’esistenza del limite e calcolarne l’eventuale valore. Una strategia che risulta spesso fruttuosa è la seguente.

Supponiamo di studiare una successione definita per ricorrenza da

(210)   \begin{equation*} 	\begin{cases} 		a_0 = \alpha 			& 		\\ 		a_{n+1} =f(a_n)			& \forall n \geq 1, 	\end{cases} \end{equation*}

dove \alpha \in D \subseteq \mathbb{R} è fissato e f \colon D \to \mathbb{R} è una funzione data, e assumiamo che tale espressione definisca effettivamente una successione. Supponiamo inoltre di sapere che a_n \to \ell. Poiché a_{n+1} è una sottosuccessione di a_n, deve aversi a_{n+1} \to \ell e, ricordando la definizione di a_{n+1}, ciò implica che

(211)   \begin{equation*} 	\ell = \lim_{n \to +\infty} a_{n+1} = 	\lim_{n \to +\infty} f(a_{n}). \end{equation*}

Tale equazione ha degli importanti risvolti sui possibili valori di \ell. Infatti, se ad esempio \ell \in D ed f è continua in \ell, per il teorema ponte 12.1 deve aversi

    \[\boxcolorato{analisi}{{ 		\ell = f(\ell), 	}}\]

cioè \ell è necessariamente un punto fisso di f. Come abbiamo già anticipato, questo pone forti restrizioni sui possibili valori del limite di una successione definita per ricorrenza come in (210). Infatti, essi devono necessariamente essere ricercati tra i punti fissi di f, a patto che le condizioni sopra enunciate siano valide.

Più in generale, se a_n non è costantemente pari a \ell e f ha limite in \ell, allora sempre il teorema ponte 12.1 applicato a (211) fornisce

(212)   \begin{equation*} 	\ell = \lim_{x \to \ell} f(x). \end{equation*}

È essenziale considerare che tali condizioni sono solo necessarie affinché il limite della successione (210) sia \ell, ma non sufficienti. Esse cioè non implicano che il limite di tale successione esita e tale esistenza deve essere stabilita con altri metodi. Solo una volta ottenuta tale esistenza si possono applicare questi ragionamenti, al fine di determinare il valore dell’eventuale limite.

Esempio 13.7. Studiamo il comportamento della seguente successione definita per ricorrenza:

    \begin{equation*} 		\begin{cases} 			a_0=\dfrac{1}{2}\\[8pt] 			a_{n+1}=\sqrt{5a_n}\qquad\text{ per }n\geq 0. 		\end{cases} 	\end{equation*}

\end{esempio} Dimostriamo per induzione che la successione è ben definita, ovvero che

    \[5a_n\geq0\qquad\text{ per ogni }n\in\mathbb{N}.\]

    \[\quad\]

  1. Per n=0 sappiamo che a_0=\frac{1}{2}\geq 0.
  2.  

  3. Supponiamo vera l’ipotesi per n e dimostriamo l’asserto per n+1; poiché a_n\geq 0 per ipotesi induttiva allora

        \begin{equation*} 		a_{n+1}=\sqrt{5a_n} 	\end{equation*}

    è ben definita è banalmente maggiore uguale a 0.

Calcoliamo alcuni termini della successione

    \begin{equation*} 	a_1=\sqrt{\frac{5}{2}}\qquad a_2=\sqrt{5\sqrt{\frac{5}{2}}}=\sqrt[4]{\frac{5^3}{2}}\qquad a_3=\sqrt{5\sqrt[4]{\frac{5^3}{2}}}=\sqrt[8]{\frac{5^7}{2}} \end{equation*}

e congetturiamo che la successione sia monotona crescente. Dimostriamo per induzione che a_n\leq a_{n+1} per ogni n\in\mathbb{N}.

    \[\quad\]

  1. L’ipotesi è verificata per n=0; infatti

        \begin{equation*} 		a_1=\sqrt{\frac{5}{2}}>\sqrt{1}=1>\frac{1}{2}=a_0. 	\end{equation*}

  2.  

  3. Supponiamo vera l’ipotesi per n e dimostriamo l’asserto per n+1:

        \begin{equation*} 		a_{n+1}=\sqrt{5a_n}\leq \sqrt{5a_{n+1}}=a_{n+2}, 	\end{equation*}

    e nella disuguaglianza abbiamo sfruttato la monotonia della funzione f(x)=\sqrt{5x}; quindi, per induzione, la proposizione è vera per ogni n\in\mathbb{N}.

Poiché la successione a_n è monotona crescente, il teorema 5.1 assicura l’esistenza del limite e inoltre, poiché a_n è una successione positiva, il teorema 5.3 ci assicura che L>0. Dato che f è una funzione continua in [0, +\infty), se L è finito, per (212) esso deve soddisfare

    \begin{equation*} 	L=\sqrt{5L}\iff L^2=5L\iff L(L-5)=0\iff L=0\vee L=5. \end{equation*}

Gli unici valori possibili sono quindi L=5 e L=+\infty. Se il limite fosse 5, dato che la successione è crescente dovrebbe valere a_n\leq 5 per ogni n\in\mathbb{N}. Se invece il limite fosse +\infty, allora dovrebbe accadere che a_n>5 definitivamente. Dimostriamo per induzione che a_n\leq 5 per ogni n\in\mathbb{N}.

    \[\quad\]

  1. a_0=\frac{1}{2}\leq 5.
  2.  

  3. Supponiamo vera l’ipotesi per n e dimostriamo l’asserto per n+1:

        \begin{equation*} 		a_{n+1}=\sqrt{5a_n}\leq \sqrt{5\cdot 5}=5. 	\end{equation*}

Per induzione la proprietà è verificata per ogni n\in\mathbb{N}; quindi

    \begin{equation*} 		\lim_{n\to+\infty}a_n=5 	\end{equation*}

e ciò conclude lo studio della successione.

Un metodo grafico per visualizzare l’andamento dei termini della successione è il diagramma a ragnatela. Disegniamo il grafico della funzione f in un piano cartesiano e, partendo dal punto di coordinate \displaystyle\left(\dfrac{1}{2}; 0\right), procediamo lungo la retta verticale fino a intersecare il grafico della funzione nel punto

    \[P_0=\displaystyle\left(a_0;\,f(a_0)\right)\Rightarrow P_0\left(\frac{1}{2};\,\sqrt{\frac{5}{2}}\right).\]

Successivamente si procede lungo la retta orizzontale fino a intersecare la bisettrice del primo e del terzo quadrante y=x nel punto Q_0=(f(a_0);f(a_0))=(a_1;\,a_1) per poi raggiungere verticalmente il grafico della funzione nel punto

    \[P_1=\left(a_1;\,f(a_1)\right)\Rightarrow P_1=\left(\sqrt{\frac{5}{2}};\sqrt[4]{\frac{5^3}{2}}\right),\]

e così via. Dal grafico

    \[\quad\]

    \[\quad\]

Figura 19: rappresentazione della costruzione dei punti P_i del grafico a ragnatela.

    \[\quad\]

    \[\quad\]

si osserva che i punti P_n si avvicinano al punto fisso 5 della funzione f.

Osservazione 13.8. La costruzione fatta nell’esercizio precedente suggerisce una strategia utile allo studio delle successioni definite per ricorrenza della forma

    \begin{equation*} 		\begin{cases} 			a_0=\alpha\in\mathbb{R}\\[3pt] 			a_{n+1}=f(a_n)\qquad\forall n\geq 0. 		\end{cases} 	\end{equation*}

    \[\quad\]

  1. Disegnare il grafico della funzione f.v Riportare il valore di \alpha sull’asse delle ascisse.
  2.  

  3. Considerare le successioni di punti

        \begin{equation*} 			\begin{split} 				&P_0=(\alpha;\,f(\alpha))\qquad \qquad \,\,\,\,Q_0=(f(\alpha);\,f(\alpha))\\& P_1=(f(\alpha);\,f(f(\alpha)))\qquad Q_1=(f(f(\alpha));\,f((\alpha)))\\&\vdots\qquad\qquad\qquad\qquad\qquad\vdots 			\end{split} 		\end{equation*}

    tale che P_i sono punti appartenenti al grafico della funzione f e Q_j sono punti della bisettrice del primo e del terzo quadrante.

Allora l’ascissa del punto P_i è l’elemento i-esimo della successione definita per ricorrenza. In questo modo è possibile osservare graficamente il carattere della successione e l’eventuale convergenza a un punto fisso della funzione f.

Esempio 13.9 (sezione aurea). Consideriamo la successione a_n di Fibonacci studiata nell’esempio 13.3 e definiamo una nuova successione come

(213)   \begin{equation*} 		b_n = \frac{a_n}{a_{n-1}} 		\qquad 		\forall n \in \mathbb{N}. 	\end{equation*}

Poiché a_{n+1}=a_{n} + a_{n-1} per ogni n \in \mathbb{N}, si ha

(214)   \begin{equation*} 		b_{n+1} = \frac{a_{n+1}}{a_{n}} 		= 		\frac{a_n +a_{n-1}}{a_{n}} 		= 		1 + \frac{a_{n-1}}{a_n} 		= 		1 + \frac{1}{b_n} 		\qquad 		\forall n \in \mathbb{N}. 	\end{equation*}

Ricordando che b_1=\frac{a_1}{a_0}=1, b_n è quindi la successione definita per ricorrenza da

(215)   \begin{equation*} 		\begin{cases} 			b_0 = 1			& 			\\ 			b_{n+1}=1 + \dfrac{1}{b_n} = f(b_n)	&	\forall n \in \mathbb{N}. 		\end{cases} 	\end{equation*}

È possibile mostrare per induzione che b_n \geq 1 per ogni n \in \mathbb{N} e quindi ciò mostra anche che b_n è ben definito per ogni n \in \mathbb{N}.

Studiamo la seguente disuguaglianza per x \geq 1:

(216)   \begin{equation*} 		f(x) \geq x 		\iff 		1 + \frac{1}{x} \geq  x 		\iff 		\frac{x^2-x-1}{x} \leq 0 		\overset{x \geq 1}{\iff} 		x^2-x-1 \leq 0 		\overset{x \geq 1}{\iff} 		x \in \left [1, \frac{1+ \sqrt{5}}{2} \right ] 	\end{equation*}

e l’uguaglianza vale se e solo se x=b \coloneqq \frac{1+ \sqrt{5}}{2}. In virtù di (211), in particolare ciò mostra che, se b_n ha limite \ell finito, allora esso è necessariamente pari a b.

Proviamo ora che

(217)   \begin{equation*} 		b_{2n+1} <  b, 		\quad 		b_{2n} >  b 		\qquad 		\forall n \in \mathbb{N}. 	\end{equation*}

Infatti, b_1=1<b. Inoltre, se b_n> b, allora

(218)   \begin{equation*} 		b_{n-1} - b =b_{n+1}-b=1+\frac{1}{b_n}-b<1+\frac{1}{b}-b=\frac{b+1-b^2}{b}<0, 	\end{equation*}

dove si è sfruttato che b è la soluzione dell’equazione f(x)=x. Analogamente si prova che, se b_n<b, allora vale b_{n+1}> b. Osserviamo che la successione b_{2k} dei termini pari è decrescente, infatti per ogni k \in \mathbb{N} si ha

(219)   \begin{equation*} 		\begin{split} 			b_{2k+2}-b_{2k}&=1+\dfrac{1}{b_{2k+1}}-b_{2k}=1+\dfrac{1}{1+\dfrac{1}{b_{2k}}}-b_{2k}=1+\dfrac{b_{2k}}{b_{2k}+1}-b_{2k}=\\&=\dfrac{b_{2k}+1+b_{2k}-b^2_{2k}-b_{2k}}{b_{2k}+1}=\dfrac{b_{2k}+1-b^2_{2k}}{b_{2k}}<0 		\end{split} 	\end{equation*}

Poiché la successione b_{2k} è decrescente e limitata dal basso da 1, essa ha limite \beta_0 \in [1,+\infty) per il teorema 5.3. Gli stessi calcoli di (219) mostrano che

(220)   \begin{equation*} 		b_{2k+2} = 1 + \frac{b_{2k}}{1+b_{2k}}= g(b_{2k}). 	\end{equation*}

La funzione g è continua in [1,+\infty), quindi passando al limite per k \to +\infty nella precedente relazione si ottiene

(221)   \begin{equation*} 		\beta_0 = g(\beta_0). 	\end{equation*}

Poiché per x \geq 1 si ha

(222)   \begin{equation*} 		g(x)= x 		\iff 		1 + \frac{x}{1+x} = x 		\iff 		\frac{1+x + x - x - x^2}{1+x} = 0 		\iff 		x^2 - x - 1 = 0 		\overset{x \geq 1}{\iff} 		x=b, 	\end{equation*}

segue che \beta_0=b. Dunque la sottosuccessione dei termini di indice pari ha limite b.

Usando b_{2k+1}= f(b_{2k}), il teorema ponte 12.1 e la continuità di f implicano che anche la sottosuccessione b_{2k} dei termini dispari abbia limite \beta_1 \in [1,+\infty) e che vale

(223)   \begin{equation*} 		\beta_1 = \lim_{k \to +\infty} b_{2k+1} 		= 		\lim_{k \to + \infty} f(b_{2k}) 		= 		f(b) \overset{13.9}{=}b. 	\end{equation*}

Dato che anche la sottosuccessione dei termini di indice dispari ha limite b, l’intera successione b_k converge a b.


 

Esercizi

Introduzione.

In questa sezione affrontiamo esercizi sul calcolo di limiti di successione che sfruttano i risultati dei paragrafi precedenti.

 
 

Esercizi risolti

Esercizio 14.1  (\bigstar\largewhitestar\largewhitestar\largewhitestar\largewhitestar). Determinare i punti di accumulazione dell’insieme

(224)   \begin{equation*} 			A= \left \{ \frac{1}{n}+\frac{1}{n^2} \colon n \in \mathbb{N} \right \}. 		\end{equation*}

Svolgimento.

L’insieme A è costituito dai termini della successione a_n=\frac{1}{n}+\frac{1}{n^2}. La successione è strettamente decrescente e proviamo che da ciò segue che l’unico punto di accumulazione di A è il limite della successione, pari a 0. Esso è infatti di accumulazione per A in virtù della proposizione 3.18, in quanto la successione a_n in A \setminus \{0\} converge a 0. Sia ora x \in \mathbb{R}\setminus \{0\} e proviamo che esso non è di accumulazione per A. A tal fine, distinguiamo alcuni casi.

    \[\quad\]

  • x<0. Poiché A \subset (0,+\infty) l’intorno \left (\frac{3}{2}x, \frac{x}{2} \right ) non contiene alcun punto di A, quindi x è esterno ad A.
  •  

  • x\in (0,+\infty) \cap A. Si ha x=\frac{1}{n}+\frac{1}{n^2} per qualche n \in \mathbb{N}. Poiché la successione degli a_k è decrescente, l’intorno \left (a_{n+1},a_{n-1}\right ) di x non contiene alcun punto di A al di fuori di x.
  •  

  • x\in (0,+\infty) \setminus A. Se x>a_1, allora per la monotonia di a_n, l’intorno (a_1,+\infty) di x non contiene alcun punto di A. Se invece x \in (a_{n+1},a_n), questo insieme è un intorno di x disgiunto da A.

 
 

Esercizio 14.2  (\bigstar\largewhitestar\largewhitestar\largewhitestar\largewhitestar). Calcolare

(225)   \begin{equation*} 			\lim_{n \rightarrow +\infty} (2^n-n). 		\end{equation*}

Svolgimento.

Sappiamo che \displaystyle\lim_{n \to + \infty} n=+\infty e che \displaystyle\lim_{n \to + \infty} 2^n=+\infty dall’esempio 3.9; quindi il limite (225) è una forma indeterminata [+\infty-\infty].

Raccogliendo 2^n otteniamo

    \begin{equation*} 	\lim_{n \to \infty} 2^n\left(1-\frac{n}{2^n}\right)=\lim_{n \rightarrow +\infty} 2^n(1-o(1))=+\infty. \end{equation*}

dove nell’ultima uguaglianza abbiamo sfruttato il risultato dell’esempio 10.7.


 
 

Esercizio 14.3  (\bigstar\bigstar\largewhitestar\largewhitestar\largewhitestar). Calcolare

    \begin{equation*} 			\lim_{n \to +\infty}\left(1+\frac{1}{3n}\right)^{2n}. 		\end{equation*}

Svolgimento.

In questo caso possiamo applicare a a=\frac{1}{3} il limite notevole

    \begin{equation*} 	\lim_{n \rightarrow +\infty}\left(1+\frac{a}{n}\right)^n=e^a\qquad\forall a\in\mathbb{R}, \end{equation*}

mostrato nella proposizione 7.7 e ottenere

    \begin{equation*} 	\lim_{n \to \infty}\left(1+\frac{1}{3n}\right)^{2n}=\lim_{n \to \infty}\left[\left(1+\frac{\dfrac{1}{3}}{n}\right)^{n}\right]^2=\left(e^\frac{1}{3}\right)^2=e^\frac{2}{3}. \end{equation*}


 
 

Esercizio 14.4  (\bigstar\bigstar\largewhitestar\largewhitestar\largewhitestar). Calcolare

    \begin{equation*} 			\lim_{n \rightarrow +\infty} \sqrt[n]{\arctan n +2}. 		\end{equation*}

Svolgimento.

Sappiamo che la successione \arctan n è limitata:

    \begin{equation*} 	0\leq \arctan n\leq \frac{\pi}{2}\qquad\forall n\in\mathbb{N}, \end{equation*}

quindi

    \begin{equation*} 	2\leq \arctan n+2\leq \frac{\pi}{2}+2\qquad\forall n\in\mathbb{N}. \end{equation*}

Allora

    \begin{equation*} 	\sqrt[n]{2}\leq \sqrt[n]{\arctan n+2}\leq \sqrt[n]{\frac{\pi}{2}+2}\qquad\forall n\in\mathbb{N}. \end{equation*}

Per l’esempio 8.7, si ha

    \begin{equation*} 	\lim_{n \rightarrow +\infty} \sqrt[n]{2}=\lim_{n \rightarrow +\infty} \sqrt[n]{\frac{\pi}{2}+2}=1, \end{equation*}

allora per il teorema 5.8 possiamo concludere che

    \begin{equation*} 	\lim_{n \rightarrow +\infty}\sqrt[n]{\arctan n +2}=1. \end{equation*}


 
 

Esercizio 14.5  (\bigstar\bigstar\bigstar\largewhitestar\largewhitestar). Calcolare

    \begin{equation*} 			\lim_{n \rightarrow +\infty} \frac{n^2-n\sin n}{3n^2+\cos n}. 		\end{equation*}

Svolgimento.

Raccogliamo n^2 sia al numeratore che al denominatore e otteniamo

    \begin{equation*} 	\lim_{n \to \infty} \frac{n^2\left(1-\dfrac{\sin n}{n}\right)}{n^2\left(3+\dfrac{\cos n}{n}\right)}. \end{equation*}

Poiché le successioni \sin n e \cos n sono limitate, mentre \frac{1}{n} è infinitesima, allora per il teorema 4.6 sappiamo che

    \begin{equation*} 	\lim_{n \rightarrow +\infty}\frac{\sin n}{n}=0\qquad\Rightarrow\qquad \frac{\sin n}{n}=o(1) \end{equation*}

e analogamente che

    \begin{equation*} 	\lim_{n \rightarrow +\infty}\dfrac{\cos n}{n}=0\qquad\Rightarrow\qquad \frac{\cos n}{n}=o(1). \end{equation*}

Quindi

    \begin{equation*} 	\lim_{n \to \infty} \frac{n^2\left(1-\dfrac{\sin n}{n}\right)}{n^2\left(3+\dfrac{\cos n}{n}\right)}=\lim_{n \rightarrow +\infty}\frac{1-o(1)}{3+o(1)}=\frac{1}{3}. \end{equation*}


 
 

Esercizio 14.6  (\bigstar\bigstar\bigstar\bigstar\largewhitestar). Calcolare

    \begin{equation*} 			\lim_{n \rightarrow +\infty} \frac{\sqrt{n^3+9n^2}-\sqrt{n^4+1}}{n^2+2}. 		\end{equation*}

Svolgimento.

In questo caso al numeratore abbiamo una forma indeterminata del tipo \left[+\infty-\infty\right] e per funzioni irrazionali di questo tipo una tecnica molto utile è la razionalizzazione del numeratore.

    \begin{equation*} 	\begin{split} 		\lim_{n \rightarrow +\infty} \frac{\sqrt{n^3+9n^2}-\sqrt{n^4+1}}{n^2+2}&=\lim_{n \rightarrow +\infty} \frac{\sqrt{n^3+9n^2}-\sqrt{n^4+1}}{n^2+2}\cdot \frac{\sqrt{n^3+9n^2}+\sqrt{n^4+1}}{\sqrt{n^3+9n^2}+\sqrt{n^4+1}}=\\&=\lim_{n \rightarrow +\infty}\frac{|n^3+9n^2|-|n^4+1|}{(n^2+2)(\sqrt{n^3+9n^2}+\sqrt{n^4+1})}. 	\end{split} \end{equation*}

Poiché sia n^4+1 che n^3+9n^2 sono somma di termini positivi (n è positivo), si ha

    \begin{equation*} 	\lim_{n \rightarrow +\infty}\frac{|n^3+9n^2|-|n^4+1|}{(n^2+2)(\sqrt{n^3+9n^2}+\sqrt{n^4+1})}=\lim_{n \rightarrow +\infty} \frac{-n^4+n^3+9n^2-1}{(n^2+2)(\sqrt{n^3+9n^2}+\sqrt{n^4+1})}. \end{equation*}

Ora cerchiamo di determinare delle semplici successioni asintotiche al numeratore e al denominatore. Sia per il numeratore che per il denominatore è possibile sfruttare l’esempio 10.2

    \[\quad\]

  • -n^4+n^3+9n^2-1\sim -n^4
  •  

  • n^2+2\sim n^2;
  •  

  • \sqrt{n^3+9n^2}\sim n^\frac{3}{2};
  •  

  • \sqrt{n^4+1}\sim n^\frac{4}{2}=n^2;

Per il principio di sostituzione 3 del teorema 10.9 possiamo concludere che

    \begin{equation*} 	\lim_{n \rightarrow +\infty} \frac{-n^4+n^3+9n^2-1}{(n^2+2)(\sqrt{n^3+9n^2}+\sqrt{n^4+1})}=\lim_{n \rightarrow +\infty}\frac{-n^4}{n^4}=-1. \end{equation*}


 
 

Esercizio 14.7  (\bigstar\bigstar\bigstar\largewhitestar\largewhitestar). Calcolare

    \begin{equation*} 			\lim_{n \rightarrow +\infty} \sqrt[n]{n^4+1}. 		\end{equation*}

Svolgimento.

Proponiamo due svolgimenti: uno molto breve che usa il teorema dei carabinieri e un altro che fa uso delle proprietà della funzione esponenziale.

Svolgimento 1.

Osserviamo che

    \begin{equation*} 	1\leq \sqrt[n]{n^4+1} \leq \sqrt[n]{2n^4}=\sqrt[n]{2}(\sqrt[n]{n})^4. \end{equation*}

Per gli esempi 8.7 e 9.8 il membro di destra converge a 1. Dal teorema 5.8 otteniamo che \displaystyle \lim_{n\to +\infty}a_n=1.


Svolgimento 2.

Dalla definizione di logaritmo, possiamo riscrivere la successione \sqrt[n]{n^4+1} come e^{\log (\sqrt[n]{n^4+1})}

    \begin{equation*} 	\lim_{n \rightarrow +\infty} \sqrt[n]{n^4+1}=\lim_{n \rightarrow +\infty}e^{\log(\sqrt[n]{n^4+1})}=\lim_{n \rightarrow +\infty}e^{\log (n^4+1)^\frac{1}{n}}=\lim_{n \rightarrow +\infty}e^\frac{\log (n^4+1)}{n}. \end{equation*}

Concentriandoci sull’andamento dell’esponente possiamo dedurre che

    \begin{equation*} 	\lim_{n\to +\infty} \frac{\log (n^4+1)}{n}=\lim_{n\to +\infty} \frac{\log (n^4(1+\dfrac{1}{n^4}))}{n}=\lim_{n\to +\infty} \frac{4\log n^4}{n}+\frac{\log(1+\dfrac{1}{n^4})}{n}=0 \end{equation*}

per l’esempio 9.7. Allora per la continuità della funzione esponenziale e il teorema 12.1 possiamo concludere che

    \begin{equation*} 	\lim_{n \rightarrow +\infty} \sqrt[n]{n^4+1}=\lim_{n \rightarrow +\infty} e^{o(1)}=1. \end{equation*}


 
 

Esercizio 14.8  (\bigstar\bigstar\largewhitestar\largewhitestar\largewhitestar). Calcolare

    \begin{equation*} 			\lim_{n \to \infty} n\cdot \tan \left(\frac{1}{n}\right). 		\end{equation*}

Svolgimento.

Dalla definizione di tangente

    \begin{equation*} 	\lim_{n \to \infty} n\cdot \tan \left(\dfrac{1}{n}\right)=\lim_{n \to \infty} n\cdot \frac{\sin \left(\dfrac{1}{n}\right)}{\cos\left(\dfrac{1}{n}\right)} 	=\lim_{n \to \infty} \frac{\sin \left(\dfrac{1}{n}\right)}{\dfrac{1}{n}}\cdot \frac{1}{\cos\left(\dfrac{1}{n}\right)}=1\cdot 1=1, \end{equation*}

dove abbiamo utilizzato

    \begin{equation*} 	\lim_{n\to + \infty} \frac{\sin \left(\dfrac{1}{n}\right)}{\dfrac{1}{n}}=1, \end{equation*}

per il punto 1 dell’esempio 12.2.


 
 

Esercizio 14.9  (\bigstar\bigstar\bigstar\largewhitestar\largewhitestar). Calcolare

    \begin{equation*} 			\lim_{n \to +\infty}\frac{1-\cos \left(\dfrac{3}{n}\right)}{\sin \left(\dfrac{3}{n^2}\right)}. 		\end{equation*}

Svolgimento.

Per trattare la forma indeterminata \left[\dfrac{0}{0}\right], consideriamo i due limiti notevoli 1 e 2 dell’esempio 12.2 per a_n \to 0 con a_n \neq 0 definitivamente:

    \begin{equation*} 		\lim_{n \to +\infty}\frac{1-\cos a_n}{a^2_n} = \frac{1}{2}, \quad \text{e} \quad \lim_{n \to +\infty} \frac{\sin a_n}{a_n} = 1. 	\end{equation*}

Moltiplichiamo e dividiamo \displaystyle\frac{1-\cos \left(\dfrac{3}{n}\right)}{\sin \left(\dfrac{3}{n^2}\right)} per \left(\dfrac{3}{n}\right)^2 e otteniamo:

    \begin{equation*} 		\lim_{n \to +\infty}\frac{1-\cos \left(\dfrac{3}{n}\right)}{\sin \left(\dfrac{3}{n^2}\right)} = \lim_{n \to +\infty}\frac{1-\cos \left(\dfrac{3}{n}\right)}{\left(\dfrac{3}{n}\right)^2} \cdot \frac{\left(\dfrac{3}{n}\right)^2}{\sin \left(\dfrac{3}{n^2}\right)}. 	\end{equation*}

Scomponiamo \left(\dfrac{3}{n}\right)^2 in \dfrac{3}{n^2} \cdot 3 e applichiamo i limiti notevoli:

    \[\begin{aligned} 		\lim_{n \to +\infty}\frac{1-\cos \left(\dfrac{3}{n}\right)}{\left(\dfrac{3}{n}\right)^2} \cdot \frac{\dfrac{3}{n^2}}{\sin \left(\dfrac{3}{n^2}\right)} \cdot 3 &= \lim_{n \to +\infty}\frac{1-\cos \left(\dfrac{3}{n}\right)}{\left(\dfrac{3}{n}\right)^2} \cdot \lim_{n \to +\infty}\frac{\dfrac{3}{n^2}}{\sin \left(\dfrac{3}{n^2}\right)} \cdot 3 \\ 		&= \frac{1}{2} \cdot 1 \cdot 3 = \frac{3}{2}. 	\end{aligned}\]


 
 

Esercizio 14.10  (\bigstar\bigstar\bigstar\largewhitestar\largewhitestar). Calcolare

(226)   \begin{equation*} 			\lim_{n \to +\infty} n^2\cdot\left(1-2^{1/n^2}\right). 		\end{equation*}

Svolgimento.

Poiché \displaystyle\lim_{n\to+\infty} 2^{\frac{1}{n^2}}=1 per il teorema 12.1 e per la continuità della funzione esponenziale, il limite (226) è una forma indeterminata \left[\infty\cdot 0\right].

L’idea è di sfruttare il limite notevole visto in 12.2

    \begin{equation*} 	\lim_{n \to +\infty} \frac{e^{a_n}-1}{a_n}=1, \end{equation*}

valido per ogni successione \{a_n\}_{n\in\mathbb{N}} infinitesima con a_n\neq 0 definitivamente. Grazie all’identità \displaystyle2^\frac{1}{n^2}=e^{\log2^\frac{1}{n^2}} e alle proprietà dei logaritmi, otteniamo

    \begin{equation*} 	\begin{split} 	\lim_{n \to +\infty} n^2\cdot\left(1-2^{1/n^2}\right)&=\lim_{n \to +\infty} \frac{1-e^{\frac{1}{n^2}\log 2}}{\frac{1}{n^2}}=\\&=\lim_{n \to +\infty} \frac{1-e^{\frac{1}{n^2}\log 2}}{\frac{1}{n^2}}\cdot \frac{\log 2}{\log 2}=\\&=\lim_{n \to +\infty} \frac{1-e^{\frac{1}{n^2}\log 2}}{\frac{1}{n^2}\log 2}\cdot \log 2=\\&=-\log 2. 	\end{split} \end{equation*}


 
 

Esercizio 14.11  (\bigstar\bigstar\bigstar\largewhitestar\largewhitestar). Calcolare

    \begin{equation*} 			\lim_{n \to +\infty}\frac{\log(1+e^{-n})}{\sin \left(\dfrac{1}{n}\right)}. 		\end{equation*}

Svolgimento.

Per trattare la forma indeterminata \left[\dfrac{0}{0}\right], consideriamo i due limiti notevoli 1 e 4 dell’esempio 12.2 per a_n \to 0 con a_n \neq 0 definitivamente:

    \begin{equation*} 		\lim_{n \to +\infty}\frac{\log(1+a_n)}{a_n} =1, \quad \text{e} \quad \lim_{n \to +\infty} \frac{\sin a_n}{a_n} = 1. 	\end{equation*}

Moltiplichiamo e dividiamo \displaystyle\frac{\log(1+e^{-n})}{\sin \left(\dfrac{1}{n}\right)} per le due quantità \left(e^{-n}\right), \displaystyle\dfrac{1}{n} e otteniamo:

    \begin{equation*} 		\lim_{n \to +\infty}\frac{\log(1+e^{-n})}{\sin \left(\dfrac{1}{n}\right)}=\lim_{n \to +\infty} \frac{\log(1+e^{-n})}{e^{-n}}\cdot \frac{\sin \left(\dfrac{1}{n}\right)}{\left(\dfrac{1}{n}\right)}\cdot \frac{e^{-n}}{n}= %		\lim_{n \to +\infty} \frac{\log(1+e^{-n})}{e^{-n}}\cdot \frac{\sin \left(\dfrac{1}{n}\right)}{\left(\dfrac{1}{n}\right)}\cdot \frac{1}{e^n\cdot n} %		= 		1\cdot 1\cdot 0=0. 	\end{equation*}


 
 

Esercizio 14.12  (\bigstar\bigstar\bigstar\largewhitestar\largewhitestar). Calcolare

    \begin{equation*} 			\lim_{n \to +\infty}\sqrt[n]{\frac{n!}{2^n+1}}. 		\end{equation*}

Svolgimento.

Vogliamo sfruttare la proposizione 8.9 per la successione a_n=\dfrac{n!}{2^n+1}:

    \begin{equation*} 	\begin{split} 		\lim_{n \to +\infty}\frac{a_{n+1}}{a_n}&=\lim_{n \to +\infty}\frac{(n+1)!}{2^{n+1}+1}\cdot \frac{2^n+1}{n!}=\\&=\lim_{n \to +\infty} \frac{(n+1)\cancel{n!}}{\cancel{n!}}\cdot\frac{2^n+1}{2^{n+1}+1}=\\&=\lim_{n \to +\infty} \left(n+1\right)\frac{2^n\left(1+\dfrac{1}{2^n}\right)}{2^{n+1}\left(1+\frac{1}{2^{n+1}}\right)}=\\&=\lim_{n \to +\infty} \left(n+1\right)\frac{(1+o(1))}{2(1+o(1))}=+\infty. 	\end{split} \end{equation*}

Per la proposizione 8.9 possiamo concludere che

    \begin{equation*} 	\lim_{n \to +\infty}\sqrt[n]{a_n}=	\lim_{n \to +\infty}\sqrt[n]{\frac{n!}{2^n+1}}=+\infty. \end{equation*}


 
 

Esercizio 14.13  (\bigstar\bigstar\bigstar\largewhitestar\largewhitestar). Calcolare

    \begin{equation*} 			\lim_{n \to +\infty}\sqrt[n]{\binom{3n}{n}}. 		\end{equation*}

Svolgimento.

Consideriamo la successione a_n = \binom{3n}{n}. Calcoliamo il limite del rapporto \dfrac{a_{n+1}}{a_n}:

    \[\begin{aligned} 		\lim_{n \to +\infty}\frac{a_{n+1}}{a_n} &= \lim_{n \to +\infty} \frac{(3(n+1))!}{(n+1)!(2n+2)!} \cdot \frac{n!(2n)!}{(3n)!}\\ 		&= \lim_{n \to +\infty} \frac{(3n+3)(3n+2)(3n+1)(3n)!}{(n+1)n!(2n+2)(2n+1)(2n)!} \cdot \frac{n!(2n)!}{(3n)!} \\ 		&= \lim_{n \to +\infty} \frac{(3n+3)(3n+2)(3n+1)}{(n+1)(2n+2)(2n+1)}. 	\end{aligned}\]

Utilizzando le approssimazioni polinomiali (esempio 10.2), possiamo scrivere (3n+3)(3n+2)(3n+1) \sim 27n^3 e (2n+2)(2n+1) \sim 4n^2. Applicando il principio di sostituzione 3 del teorema 10.9, otteniamo:

    \[\begin{aligned} 		\lim_{n \to +\infty}\frac{a_{n+1}}{a_n} &= \lim_{n \to +\infty}\frac{27n^3}{n(4n^2)} 		= \frac{27}{4}. 	\end{aligned}\]

Infine, per la proposizione 8.9 concludiamo che:

    \begin{equation*} 		\lim_{n \to +\infty}\sqrt[n]{a_n} = \lim_{n \to +\infty}\sqrt[n]{\binom{3n}{n}} = \frac{27}{4}. 	\end{equation*}


 
 

Esercizio 14.14  (\bigstar\bigstar\bigstar\largewhitestar\largewhitestar). Calcolare

    \begin{equation*} 			\lim_{n \rightarrow +\infty} \dfrac{(n!)^2}{n^n}. 		\end{equation*}

Svolgimento.

Si osserva che b_n=\frac{(n!)^2}{n^n} è una successione a termini positivi, quindi possiamo applicare il criterio del rapporto:

    \[\begin{aligned} 	\lim_{n\to + \infty} \dfrac{b_{n +1}}{b_n}&= \lim_{n\to+\infty} \dfrac{((n+1)!)^2}{(n+1)^{n+1}}\cdot \dfrac{n^n}{(n!)^2}=\\ 	&=\lim_{n\to+\infty} \dfrac{(n+1)^2\cancel{(n!)^2}}{(n+1)^{n+1}}\cdot \dfrac{n^n}{\cancel{(n!)^2}}=\\&=\lim_{n\to +\infty} \dfrac{(n+1)^2}{(n+1)^{n+1}}\cdot n^n=\\ 	&=\lim_{n\to+\infty} \dfrac{n^2 \left(1+ \dfrac{1}{n} \right)^2}{\cancel{n^n} \left(1+\dfrac{1}{n} \right)^n n (1+\dfrac{1}{n})}\cdot \cancel{n^n}= 	\\ 	&=\lim_{n\to+\infty} \dfrac{n^{\cancel{2}}(1+\dfrac{1}{n})}{\left(1+\dfrac{1}{n} \right)^n \cancel{n}}=\\ 	&=\lim_{n\to+\infty} \dfrac{n(1+\dfrac{1}{n})}{\left( 1+\dfrac{1}{n}\right)^n}=+\infty,	 \end{aligned}\]

dove si è usata la definizione 7.2. Si conclude per il criterio del rapporto che:

    \[\lim_{n\to+\infty} \dfrac{(n!)^2}{n^n}=+\infty.\]


 
 

Esercizio 14.15  (\bigstar\bigstar\bigstar\largewhitestar\largewhitestar). Calcolare

    \begin{equation*} 			\lim_{n \to + \infty} \dfrac{(2^n\cdot n! + n^n)^2}{(2n)!+(n+1)^{2n}}. 		\end{equation*}

Svolgimento.

Al fine di calcolare il limite è necessario determinare quali termini risultano trascurabili per n\to +\infty; andiamo quindi a utilizzare il criterio del rapporto sia al numeratore che al denominatore.

    \[\quad\]

  • Numeratore. \dfrac{2^n\cdot n!}{n^n}

        \begin{equation*} 			\begin{split} 				\lim_{n \to + \infty}\frac{a_{n+1}}{a_n}&=	\lim_{n \to + \infty}\frac{2^{n+1}\cdot (n+1)!}{(n+1)^{n+1}}\cdot\frac{n^n}{2^n\cdot n!}=\lim_{n\to +\infty} \frac{2\cdot (n+1)}{(n+1)^{n+1}}\cdot n^n=\\&=\lim_{n\to +\infty} 2\frac{n^n}{(n+1)^n}=\lim_{n\to +\infty}2\left(\frac{n+1}{n}\right)^{-n}=2e^{-1}<1. 			\end{split} 		\end{equation*}

    Per il criterio del rapporto 8.1 possiamo concludere che \displaystyle\lim_{n\to +\infty}\dfrac{2^n\cdot n!}{n^n}=0.

  •  

  • Denominatore. \dfrac{(2n)!}{n^{2n}}

        \begin{equation*} 			\begin{split} 				\lim_{n \to + \infty}\frac{a_{n+1}}{a_n}&=\lim_{n \to + \infty} \frac{(2n+2)!}{(n+1)^{2n+2}}\cdot \frac{n^{2n}}{(2n!)}=\\&=\lim_{n \to + \infty}\frac{(2n+2)\cdot(2n+1)\cdot \cancel{(2n)!}}{\cancel{(2n)!}}\cdot \left(\frac{n+1-1}{n+1}\right)^{2n}\cdot \frac{1}{(n+1)^2}=\\&=\lim_{n \to + \infty}\frac{(2n+2)\cdot (2n+1)}{(n+1)^2}\cdot \left(1-\frac{1}{n+1}\right)^{2n+2-2}=\\&=\lim_{n \to + \infty}\frac{(2n+2)\cdot (2n+1)}{(n+1)^2}\cdot \left(1-\frac{1}{n+1}\right)^{2(n+1)}\cdot \left(1-\frac{1}{n+1}\right)^{-2}. 			\end{split} 		\end{equation*}

    Facciamo le seguenti osservazioni:

    – dall’esempio 10.2 sappiamo che (2n+2)\cdot (2n+1)\sim 4n^2 e (n+1)^2\sim n^2;

    – dalla proposizione 7.7 \displaystyle\lim_{n \to + \infty}\left(1-\frac{1}{n+1}\right)^{2(n+1)}=e^{-2};

    \displaystyle\lim_{n \to + \infty}\left(1-\frac{1}{n+1}\right)^{-2}=1.

    Possiamo concludere che

        \begin{equation*} 			\lim_{n \to + \infty}\frac{a_{n+1}}{a_n}=4\cdot e^{-2}\cdot 1<1 		\end{equation*}

    e quindi per il criterio del rapporto 8.1 si ha

        \[\lim_{n \to +\infty} \dfrac{(2n)!}{n^{2n}}=0.\]

Pertanto, tornando al limite originale, possiamo raccogliere per n^n al numeratore e per n^{2n} al denominatore e ottenere

(227)   \begin{equation*} 	\begin{split} 	\lim_{n \to + \infty} \dfrac{(2^n \cdot n! + n^n)^2}{(2n)!+(n+1)^{2n}} &= \lim_{n \to +\infty} \dfrac{n^{2n} (1+o(1))}{(2n)!+n^{2n} \left(1+\frac{1}{n}\right)^{2n}}=\\&=\lim_{n \to +\infty} \dfrac{(2^n \cdot n! + n^n)^2}{(2n)!+(n+1)^{2n}}=\\&=\lim_{n \to + \infty}\dfrac{1+o(1)}{o(1)+\left(1+\frac{1}{n}\right)^{2n}} = \dfrac{1}{e^2} 	\end{split} \end{equation*}


 
 

Esercizio 14.16  (\bigstar\bigstar\bigstar\largewhitestar\largewhitestar). Studiare il comportamento della successione definita per ricorrenza

    \begin{equation*} 		\begin{cases} 			a_1=\dfrac{1}{2}\\\\ 			a_{n+1}=na_n^2\qquad\text{ per }n\geq 1. 		\end{cases} 	\end{equation*}

Svolgimento.

La successione è a termini positivi; dal calcolo dei primi valori

    \begin{equation*} 		a_1=\frac{1}{2},\qquad a_2=\left(\frac{1}{2}\right)^2,\qquad a_3=2\cdot \left(\frac{1}{2}\right)^4=\left(\frac{1}{2}\right)^3,\qquad a_4=3\cdot \left(\frac{1}{2}\right)^6\leq \left(\frac{1}{2}\right)^4, 	\end{equation*}

possiamo ipotizzare che

    \begin{equation*}\label{eq:es14_13} 		0\leq a_{n}\leq 2^{-n}\qquad\forall n\in\mathbb{N}. 	\end{equation*}

Proviamolo per induzione.

    \[\quad\]

  • la tesi è banalmente vera per n=1;
  •  

  • supponiamo vera l’ipotesi per n e dimostriamo l’asserto per n+1; si ha

        \begin{equation*} 			a_{n+1}=na_n^2\leq n 2^{-2n}. 		\end{equation*}

    Per cui è sufficiente verificare che

        \begin{equation*} 			n2^{-2n}\leq 2^{-n-1}\qquad\forall n\in\mathbb{N} 		\end{equation*}

    che è equivalente a

        \begin{equation*} 			n2^{-2n}\leq 2^{-n-1}\Longleftrightarrow n\leq 2^{n-1} 		\end{equation*}

    che possiamo dimostrare nuovamente per induzione.

        \[\quad\]

    • Per n=1 abbiamo 1\leq 2^0=1.
    •  

    • Dimostriamo la tesi per n+1 supponendo vera l’ipotesi per n:

          \begin{equation*} 				2^{n}=2^{n-1+1}=2^{n-1}2\geq n\cdot 2\geq n+1. 			\end{equation*}

    Possiamo quindi concludere che

        \begin{equation*} 			a_{n+1}\leq 2^{-n-1} 		\end{equation*}

    ovvero la tesi.

Dunque per il teorema del confronto 5.8 e da 14.1 otteniamo che

    \begin{equation*} 		\lim_{n\to+\infty} a_n=0. 	\end{equation*}


 
 

Esercizio 14.17  (\bigstar\bigstar\bigstar\bigstar\largewhitestar). Calcolare

    \[\lim_{n\to +\infty} \dfrac{(n+1)!}{n^n}.\]

Svolgimento 1.

Si ha

(228)   \begin{equation*} 0 \leq \frac{(n+1)!}{n^n} = \frac{2 \cdot 3 \cdots n \cdot (n+1)}{n \cdot n \cdots n} \leq \frac{2}{n} \cdot 1 \cdot 1 \cdots 1 \cdot 2 = \frac{4}{n} \qquad \forall n \in \mathbb{N}, \end{equation*}

dato che al numeratore abbiamo un fattore 2, n-2 fattori minori o uguali a n e un fattore n+1 che è minore o uguale a 2n. Poiché la successione \dfrac{4}{n} è infinitesima, per il teorema 5.8 tale è anche \dfrac{(n+1)!}{n^n}.


Svolgimento 2.

Si osserva che: \displaystyle b_n=\frac{(n+1)!}{n^n} è una successione a termini positivi, quindi possiamo applicare nuovamente il criterio del rapporto:

    \[\begin{aligned} 	\lim_{n\to+\infty} \dfrac{b_{n+1}}{b_n}&= 	\lim_{n\to+\infty} \dfrac{(n+2)!}{(n+1)^{n+1}} \cdot \dfrac{n^n}{(n+1)!}=\\ 	&=\lim_{n\to+\infty} \dfrac{(n+2)\cancel{(n+1)!}}{\cancel{n^n} \left(1+\dfrac{1}{n} \right)^n n(1+o(1))}\cdot \dfrac{\cancel{n^n}}{\cancel{(n+1)!}}=\\ 	&=\lim_{n\to+\infty} \dfrac{(n+2)}{\left(1+\dfrac{1}{n} \right)^n n(1+o(1))}=\\ 	&=\lim_{n\to+\infty} \dfrac{\cancel{n}(1+o(1))}{\left(1+\dfrac{1}{n} \right)^n \cancel{n}(1+o(1))}=\dfrac{1}{e}<1. \end{aligned}\]

Per il criterio del rapporto 8.1, si conclude che

    \[\lim_{n\to +\infty} \dfrac{(n+1)!}{n^n}=0.\]


 
 

Esercizio 14.18  (\bigstar\bigstar\bigstar\largewhitestar\largewhitestar). Calcolare

    \begin{equation*} 			\lim_{n \rightarrow +\infty} \dfrac{(2^n)^\alpha}{(n!)^\beta} 		\end{equation*}

per ogni \alpha,\,\beta>0.

Svolgimento.

Applichiamo nuovamente il criterio del rapporto alla successione \displaystyle b_n=\frac{(2^n)^\alpha}{(n!)^\beta}

    \[\begin{aligned} 	\lim_{n\to+\infty} \dfrac{b_{n+1}}{b_n} 	&=\lim_{n\to+\infty} \dfrac{2^{(n+1)\alpha}}{\left((n+1)! \right)^{\beta}} \dfrac{(n!)^{\beta}}{2^{n\alpha}}=\\ 	&=\lim_{n\to+\infty} \dfrac{2^{\alpha}}{(n+1)^{\beta}}=0 \quad\qquad \forall \alpha,\, \beta>0.\\ \end{aligned}\]

Per il criterio del rapporto 8.1, si conclude

    \[\lim_{n\to+\infty} \dfrac{2^{n\alpha}}{(n!)^{\beta}}=0 \quad \forall \alpha, \, \beta>0.\]


 
 

Esercizio 14.19  (\bigstar\bigstar\bigstar\largewhitestar\largewhitestar). Calcolare

    \begin{equation*} 			\lim_{n\to +\infty} \dfrac{\left(\sqrt{(n!)^2-13n}-n!\right)(n+1)!}{(n+1)^3\log \left(\frac{n+3}{n+2}\right)+(n+3)^{\frac{1}{n}}}. 		\end{equation*}

Svolgimento.

Cominciamo a studiare il numeratore della frazione. Raccogliendo n! nel primo fattore si ottiene

(229)   \begin{equation*} 	\sqrt{(n!)^2 - 13n} - n! 	= 	n! \left (\sqrt{ 1 - \frac{13 n}{(n!)^2} } - 1   \right ) 	\sim 	- 	n!  \frac{13 n}{2 (n!)^2} 	= 	\frac{13 n}{2\cdot n!}, \end{equation*}

dove per l’asintotica si è usato il principio di sostituzione stabilito dal teorema 10.9, il fatto che la successione \dfrac{13 n}{2 (n!)^2} è infinitesima e che, se a_n \to 0 con a_n \neq 0 definitivamente, allora

(230)   \begin{equation*} 	\sqrt{1 + a_n} - 1 	= 	\frac{\big( \sqrt{1 + a_n} - 1 \big) \big( \sqrt{1 + a_n} + 1 \big)}{\big( \sqrt{1 + a_n} + 1 \big)} 	= 	\frac{a_n}{\big( \sqrt{1 + a_n} + 1 \big)} 	\sim 	\frac{a_n}{2}. \end{equation*}

Da (229) si ottiene che il numeratore soddisfa

(231)   \begin{equation*} 	\big( \sqrt{(n!)^2 - 13n} - n! \big) (n+1)! 	\sim 	- \frac{13 n (n+1)!}{2 \cdot n!} 	= 	- 	\frac{13 n(n+1)}{2} 	\sim 	- \frac{13}{2}n^2. \end{equation*}

Riguardo il denominatore, invece, osserviamo che

(232)   \begin{equation*} 	(n+1)^3 \sim n^3 \end{equation*}

per l’esempio 10.2 e che

(233)   \begin{equation*} 	\log \left (\frac{n+3}{n+2} \right ) 	= 	\log \left (1 +\frac{1}{n+2} \right ) 	\sim 	\frac{1}{n+2} 	\sim 	\frac{1}{n}, \end{equation*}

dove nella prima asintotica si è usato il punto 4 dell’esempio 12.2, mentre nella seconda si è usato nuovamente l’esempio 10.2. Inoltre definitivamente si ha

(234)   \begin{equation*} 	1 \leq (n+3)^{\frac{1}{n}} \leq (2n)^{\frac{1}{n}}. \end{equation*}

Pertanto, per gli esempi 8.7 e 9.8 e per il teorema del confronto 5.8, segue che

(235)   \begin{equation*} 	\lim_{n \to + \infty} (n+3)^{\frac{1}{n}}  	= 	1. \end{equation*}

Da ciò, da (232) e (233) segue che

(236)   \begin{equation*} 	(n+1)^3  \log \left (\frac{n+3}{n+2} \right ) + (n+3)^{\frac{1}{n}} 	\sim 	\frac{n^3}{n}  	\sim 	n^2. \end{equation*}

Da (231) e (236) otteniamo quindi che

(237)   \begin{equation*} 	\frac{\big( \sqrt{(n!)^2 - 13n} - n! \big) (n+1)!}{(n+1)^3  \log \left (\dfrac{n+3}{n+2} \right ) + (n+3)^{\frac{1}{n}}} 	\sim 	- \frac{\frac{13}{2}n^2}{n^2} 	= 	-\frac{13}{2}. \end{equation*}

Da ciò segue che il limite cercato vale - \dfrac{13}{2}.


 
 

Esercizio 14.20  (\bigstar\bigstar\bigstar\largewhitestar\largewhitestar). Calcolare

    \begin{equation*} 		\lim_{n\to+\infty} \dfrac{((n+1)!)^{\frac{1}{n^3}}-((n-1)!)^{\frac{1}{n^3}}}{\log(n^4+n)\tan\left( \dfrac{1}{2n^3}\right)}. 	\end{equation*}

Svolgimento.

Studiamo separatamente il numeratore e il denominatore.

    \[\quad\]

  • Numeratore. Riguardo il numeratore, osserviamo che per ogni n \in \mathbb{N} si ha

    (238)   \begin{equation*} \begin{split} 			((n+1)!)^{\frac{1}{n^3}} - ((n-1)!)^{\frac{1}{n^3}} 			&= 			(n+1)^{\frac{1}{n^3}} n^{\frac{1}{n^3}}((n-1)!)^{\frac{1}{n^3}} - ((n-1)!)^{\frac{1}{n^3}} \\ 			&= 			((n-1)!)^{\frac{1}{n^3}}\Big( \big(n(n+1)\big)^{\frac{1}{n^3}} - 1\Big). \end{split} 		\end{equation*}

    Applicando al primo fattore la stima dall’alto fornita dalla proposizione 9.2, si ottiene

    (239)   \begin{equation*} 			((n-1)!)^{\frac{1}{n^3}} 			\leq 			\left ( \frac{n-1}{e} \right )^{\frac{n-1}{n^3}} 			\big(e (n-1)\big)^{\frac{1}{n^3}} 		\end{equation*}

    Dato che \dfrac{n-1}{n^3} \leq \dfrac{1}{n-1}, si ha per n\geq 2

    (240)   \begin{equation*} 			1 \leq (n-1)^{\frac{n-1}{n^3}} \leq (n-1)^{\frac{1}{n-1}} \to 1, 		\end{equation*}

    dove l’ultimo limite è fornito dall’esempio 9.8. Quindi (n-1)^{\frac{n-1}{n^3}} \to 1 per il teorema del confronto ??. Similmente si ottiene che e^{\frac{n-1}{n^3}} \to 1 e che \big(e (n-1)\big)^{\frac{1}{n^3}}\to 1 e, sostituendo in (239), di nuovo il teorema del confronto implica

    (241)   \begin{equation*} 			\lim_{n \to + \infty} ((n-1)!)^{\frac{1}{n^3}} 			= 			1. 		\end{equation*}

    Per il fattore \big(n(n+1)\big)^{\frac{1}{n^3}} - 1, scrivendo

    (242)   \begin{equation*} 			\big(n(n+1)\big)^{\frac{1}{n^3}} - 1 			= 			e^{\frac{1}{n^3} \log(n^2+n)} - 1 			\sim 			\frac{\log (n^2+n)}{n^3} 			= 			\frac{\log(n^2)+ \log\left (1+ \frac{1}{n} \right )}{n^3} 			\sim \frac{2 \log n}{n^3} 		\end{equation*}

    dove nella prima asintotica si è usato il fatto che la successione \dfrac{1}{n^3} \log(n^2+n) è infinitesima per le proprietà dei logaritmi, il limite notevole 4 e l’esempio 9.7. Da (241) e (242) si ottiene

    (243)   \begin{equation*} 			((n+1)!)^{\frac{1}{n^3}} - ((n-1)!)^{\frac{1}{n^3}} 			\sim 			\frac{2 \log n}{n^3} 		\end{equation*}

  •  

  • Denominatore. Per il fattore \log(n^4+n) scriviamo

    (244)   \begin{equation*} 			\log(n^4+n) 			= 			\log \left ( n^4 \left (1+ \frac{1}{n^3} \right ) \right ) 			= 			\log (n^4) + \log \left (1+ \frac{1}{n^3} \right ) 			\sim 			4\log n + \frac{1}{n^3} 			\sim 			4\log n, 		\end{equation*}

    dove nella prima asintotica si è usato il limite notevole (209), mentre nella seconda si è sfruttato il fatto che \log n è divergente, mentre \dfrac{1}{n^3} è infinitesima.

    Per il fattore \tan \left ( \dfrac{1}{2n^3} \right ) si può ottenere invece

    (245)   \begin{equation*} 			\tan \left ( \dfrac{1}{2n^3} \right ) 			= 			\frac{\sin \left ( \dfrac{1}{2n^3} \right )}{\cos \left ( \dfrac{1}{2n^3} \right )} 			\sim 			{\frac{1}{2n^3}}, 		\end{equation*}

    dove l’asintotica deriva dal limite notevole (207) e dal fatto che \cos \left ( \dfrac{1}{2n^3} \right ) \to 1.

Unendo (243), (244) e (245), per il teorema 5.8 si ha

(246)   \begin{equation*} 		\dfrac{((n+1)!)^{\frac{1}{n^3}} - ((n-1)!)^{\frac{1}{n^3}}}{\log(n^4+n) \tan \left ( \dfrac{1}{2n^3} \right )} 		\sim 		\frac{\dfrac{2 \log n}{n^3}}{4 \log n \cdot \dfrac{1}{2n^3}} 		= 		1. 	\end{equation*}


 

Esercizi proposti

Introduzione.

In questa sezione proponiamo ulteriori esercizi di natura simile ai precedenti, di cui forniamo le soluzioni solo per alcuni di essi, oppure dei semplici suggerimenti.

 
 

Esercizio 15.1  (\bigstar\bigstar\largewhitestar\largewhitestar\largewhitestar). Determinare i punti di accumulazione dell’insieme

(247)   \begin{equation*} 			A = \left\{ \frac{1}{n}+\frac{1}{n^2} \colon n \in \mathbb{N} \right\}. 		\end{equation*}

Svolgimento.

L’insieme A è costituito dai termini della successione a_n=\frac{1}{n}+\frac{1}{n^2}. La successione è strettamente decrescente e proviamo che da ciò segue che l’unico punto di accumulazione di A è il limite della successione, pari a 0. Esso è infatti di accumulazione per A in virtù della proposizione 3.18, in quanto la successione a_n in A \setminus \{0\} converge a 0. Sia ora x \in \mathbb{R}\setminus \{0\} e proviamo che esso non è di accumulazione per A. A tal fine, distinguiamo alcuni casi.

    \[\quad\]

  • x<0. Poiché A \subset (0,+\infty) l’intorno \left (\frac{3}{2}x, \frac{x}{2} \right ) non contiene alcun punto di A, quindi x è esterno ad A.
  •  

  • x\in (0,+\infty) \cap A. Si ha x=\frac{1}{n}+\frac{1}{n^2} per qualche n \in \mathbb{N}. Poiché la successione degli a_k è decrescente, l’intorno \left (a_{n+1},a_{n-1}\right ) di x non contiene alcun punto di A al di fuori di x.
  •  

  • x\in (0,+\infty) \setminus A. Se x>a_1, allora per la monotonia di a_n, l’intorno (a_1,+\infty) di x non contiene alcun punto di A. Se invece x \in (a_{n+1},a_n), questo insieme è un intorno di x disgiunto da A.

 
 

Esercizio 15.2  (\bigstar\bigstar\largewhitestar\largewhitestar\largewhitestar). Determinare se la successione definita per ricorrenza da

(248)   \begin{equation*} 			\begin{cases} 				a_0  = 1  						&   \\[8pt] 				a_{n}  = \dfrac{2}{a_{n-1}} 	& n \geq 1 			\end{cases} 		\end{equation*}

è convergente, divergente o non ha limite.

Suggerimento.

Calcolare i primi termini della successione e dedurne il carattere.

 
 

Esercizio 15.3  (\bigstar\bigstar\largewhitestar\largewhitestar\largewhitestar). Calcolare il limite della successione definita per ricorrenza da

(249)   \begin{equation*} 			\begin{cases} 				a_0  = a\geq -1  						&   \\[8pt] 				a_{n}  = \sqrt{\dfrac{1+a_{n-1}}{2} }	& n \geq 1. 			\end{cases} 		\end{equation*}

Suggerimento.

Osservare che, se a_n<1, allora a_n < a_{n+1}<1, mentre a_n>1 implica 1<a_{n+1}<a_n. Da ciò dedurre che a_n ha limite finito e calcolarlo determinando i punti fissi di f(x)= \sqrt{\frac{1+x}{2}}.

 
 

Esercizio 15.4  (\bigstar\bigstar\largewhitestar\largewhitestar\largewhitestar). Dimostrare che la successione i cui primi termini sono

(250)   \begin{equation*} 			a_0=\dfrac{1}{2}, 			\qquad 			a_1= \dfrac{1}{2+ \dfrac{1}{2}}, 			\qquad 			a_2= \dfrac{1}{2 + \dfrac{1}{2+ \dfrac{1}{2}}}, 			\dots 		\end{equation*}

è convergente e calcolarne il limite

Svolgimento.

Osserviamo innanzitutto che a_n>0 per ogni n \in \mathbb{N} e quindi in particolare la successione è ben definita. Sia f(x)=\dfrac{1}{2+x} e studiamo la disuguaglianza

(251)   \begin{equation*} f(x)\leq x \iff \frac{1}{2+x} \leq x \iff x^2+2x-1\leq0 \iff x\leq -1-\sqrt{2} \vee x \leq -1+\sqrt{2}. \end{equation*}

Dunque l’unico punto fisso positivo di f è x_0 \coloneqq \sqrt{2}-1. Proviamo per induzione che

(252)   \begin{equation*} a_{2n} > x_0, \quad a_{2n+1}< x_0. \end{equation*}

Infatti a_0=\dfrac{1}{2}>x_0. Se a_n>x_0 allora

(253)   \begin{equation*} a_{n+1}=f(a_n)< f(x_0) =x_0 \end{equation*}

in quanto f è decrescente. Analogamente si mostra che a_n<x_0 implica a_{n+1}>x_0. Ciò mostra che la successione oscilla intorno a x_0.

Il seguito è simile a quanto riportato nell’esempio 13.9.


 
 

Esercizio 15.5  (\bigstar\largewhitestar\largewhitestar\largewhitestar\largewhitestar). Se a_n è una successione reale convergente a un numero reale \ell \neq 0, dimostrare che definitivamente si ha

(254)   \begin{equation*} 			|a_n| > \frac{|\ell|}{2}. 		\end{equation*}

Suggerimento.

Usare il teorema della permanenza del segno su |a_n|-\dfrac{|\ell|}{2}.

 
 

Esercizio 15.6  (\bigstar\largewhitestar\largewhitestar\largewhitestar\largewhitestar). Mostrare che la successione a_n definita da

(255)   \begin{equation*} 			a_n= \frac{2n - 7}{3n+2} 		\end{equation*}

è crescente, limitata e ha limite.

Svolgimento.

Si ha

(256)   \begin{equation*} a_{n+1}-a_n = \frac{2(n+1)-7}{3(n+1)+2} - \frac{2n-7}{3n+2} = \frac{2(3n+2)-3(2n-7)}{(3n+5)(3n+2)} = \frac{25}{(3n+5)(3n+2)}>0 \qquad \forall n \in \mathbb{N}, \end{equation*}

quindi la successione è crescente. Osserviamo inoltre che essa è limitata dall’alto, ad esempio da 1:

(257)   \begin{equation*} a_n < 1 \iff \frac{2n-7}{3n+2} < 1 \iff 3n + 2 - 2n+7>0 \iff n+9>0, \end{equation*}

che è vera per ogni n \in \mathbb{N}. Essendo crescente e limitata, a_n converge per il teorema 5.1. Con un ragionamento analogo si può vedere che a_n<x per ogni n se e solo se x\geq \dfrac{2}{3} e quindi provare in maniera alternativa

(258)   \begin{equation*} \sup_{n \in \mathbb{N}} a_n = \lim_{n \to +\infty} a_n = \frac{2}{3}. \end{equation*}


 
 

Esercizio 15.7  (\bigstar\bigstar\bigstar\largewhitestar\largewhitestar). Confrontare i seguenti ordini di infinito, ovvero stabilire, tra le seguenti successioni, se ciascuna è un o-piccolo dell’altra oppure se sono asintotiche, eventualmente a meno di un coefficiente:

    \[\quad\]

  1. \displaystyle a_n=n^n  ,\qquad  b_n= (3n)^{1000}   , \qquad   c_n= (1000)^{3n};
  2.  

  3. \displaystyle a_n=n^{n+1}  ,\qquad  b_n=   n^n, \qquad   c_n=(n+2)!;
  4.  

  5. \displaystyle a_n=(10^n)^2  ,\qquad  b_n= \sqrt{10^{n^2}}  , \qquad   c_n=10000^{\sqrt{n}};
  6.  

  7. \displaystyle a_n=\log(n^n +1)  ,\qquad  b_n=n \sqrt{n}   , \qquad   c_n=\frac{n}{(\log_n e)^2};
  8.  

  9. \displaystyle a_n=\sqrt{n!+1}  ,\qquad  b_n=n^{\frac{n}{2}}   , \qquad   c_n=\left (\frac{n}{2}\right )^n;
  10.  

  11. \displaystyle a_n= (n^2)!  ,\qquad  b_n=   (n!)^2, \qquad   c_n=n^{2n};
  12.  

  13. \displaystyle a_n=2^{2^n}  ,\qquad  b_n=n^{n^2}   , \qquad   c_n=(n!)^n;
  14.  

  15. \displaystyle a_n=n^{\log n}  ,\qquad  b_n= (\log n)^n  , \qquad   c_n=n^{\sqrt{n}};
  16.  

  17. \displaystyle a_n=(n^n)^{n!}  ,\qquad  b_n= (n!)^{n^n}  , \qquad   c_n=2^{(n!)^2}.

 
 

Esercizio 15.8  (\bigstar\bigstar\largewhitestar\largewhitestar\largewhitestar). Calcolare

(259)   \begin{equation*} 			\lim_{n \to + \infty} 			\left ( \frac{1}{\sqrt{n}} + \frac{1}{\sqrt{n+1}} + \dots + \frac{1}{\sqrt{2n}} 			\right ) 			= 			\lim_{n \to + \infty} \left ( 			\sum_{k=0}^n \frac{1}{\sqrt{n+k}} 			\right ). 		\end{equation*}

Suggerimento.

Osservare che si ha

(260)   \begin{equation*} \frac{1}{\sqrt{n}} + \frac{1}{\sqrt{n+1}} + \cdots + \frac{1}{\sqrt{2n}} \geq \frac{1}{\sqrt{2n}} +\frac{1}{\sqrt{2n}} + \dots + \frac{1}{\sqrt{2n}} = \frac{n}{\sqrt{2n}} \qquad \forall n \in \mathbb{N}. \end{equation*}


 
 

Esercizio 15.9  (\bigstar\bigstar\largewhitestar\largewhitestar\largewhitestar). Calcolare

(261)   \begin{equation*} 			\lim_{n \to + \infty} 			\left ( 1 + \frac{1}{n^n} \right)^{n!} 		\end{equation*}

Suggerimento.

Si scriva

(262)   \begin{equation*} \lim_{n \to +\infty} \left (1+\frac{1}{n^n} \right )^{n!} = e^{n! \log\left (1+\frac{1}{n^n} \right )} \end{equation*}

e si usi il fatto che \log\left (1+\dfrac{1}{n^n} \right ) \sim \dfrac{1}{n^n}.


 
 

Esercizio 15.10  (\bigstar\bigstar\largewhitestar\largewhitestar\largewhitestar). Calcolare

(263)   \begin{equation*} 			\lim_{n \to + \infty} 			\frac{\log (n!)}{n \log n}. 		\end{equation*}

Suggerimento.

Definendo a_n= \log(n!) e b_n= n \log n, si ha

(264)   \begin{equation*} \begin{split} \frac{a_{n+1}-a_n}{b_{n+1}-b_n} &= \frac{\log(n+1)}{(n+1)\log(n+1) - n \log n} \\ &= \frac{\log(n+1)}{\log\left ((n+1) \left (1+\dfrac{1}{n}\right)^n  \right )} \\ &= \frac{\log(n+1)}{\log(n+1) + \log\left (\left (1+\dfrac{1}{n}\right)^n  \right )}, \end{split} \end{equation*}

che ha limite 1. Il teorema 11.5 implica allora

(265)   \begin{equation*} 1= \lim_{n \to +\infty} \frac{a_n}{b_n} = \lim_{n \to +\infty} \frac{\log(n!)}{n \log n}. \end{equation*}


 
 

Esercizio 15.11  (\bigstar\bigstar\largewhitestar\largewhitestar\largewhitestar). Calcolare

(266)   \begin{equation*} 			\lim_{n \to + \infty} 			\frac{1 + \dfrac{1}{2} + \dots + \dfrac{1}{n}}{\log n} 			= 			\lim_{n \to + \infty} 			\frac{\sum_{k=1}^n \dfrac{1}{k}}{\log n}. 		\end{equation*}

Suggerimento.

Usare il teorema 11.5.

 
 

Esercizio 15.12  (\bigstar\bigstar\largewhitestar\largewhitestar\largewhitestar). Per p < 1, mostrare che

(267)   \begin{equation*} 			\lim_{n \to + \infty} \sum_{k=1}^n \frac{1}{k^p} 			= 			+\infty 		\end{equation*}

e, più in generale, che

(268)   \begin{equation*} 			\lim_{n \to + \infty} 			\frac{\sum_{k=1}^n \frac{1}{k^p}}{n^{1-p}} 			< 			+\infty. 		\end{equation*}

Suggerimento.

Usare il teorema 11.5 per mostrare il secondo punto, dal quale segue necessariamente il primo.

 
 

Schemi riassuntivi

Introduzione.

Inseriamo infine una sezione di schemi riassuntivi, molto utili per destreggiarsi tra i vari risultati.

Definizioni e risultati fondamentali.

Successioni

Una funzione a \colon \mathbb{N} \to \mathbb{R} è una successione e si indica con \{a_n\}_{n \in \mathbb{N}}, o con a_n.

Definizione 2.1
Data una successione a_n e una funzione k \colon n \in \mathbb{N} \to n_k \in \mathbb{N} strettamente crescente, \{a_{n_k}\}_{k \in \mathbb{N}} è una sottosuccessione o un’estratta di a_n. Definizione 2.18

La successione a_n verifica la proprietà \mathcal{P} definitivamente se \exists N \in \mathbb{N} \colon \mathcal{P}(a_n) è vera per ogni n \geq N.

Definizione 2.15

    \[\lim_{n \to +\infty} a_n = \ell \in \mathbb{R} \iff \forall \varepsilon > 0 \quad \exists N \in \mathbb{N} \colon a_n \in (\ell - \varepsilon, \ell + \varepsilon) \quad \forall n \geq N\]

Definizione 3.2, Proposizione 3.4

    \[                 \lim_{n \to +\infty} a_n = +\infty \iff \forall M \in \mathbb{R} \quad \exists N \in \mathbb{N} \colon a_n > M \quad \forall n \geq N                 \]

Definizione 3.2, Proposizione 3.5

    \[                 a_n \text{ è \textbf{limitata superiormente}} \iff \exists M \in \mathbb{R} \colon a_n \leq M \quad \forall n \in \mathbb{N}                 \]

    \[                 a_n \text{ è \textbf{limitata inferiormente}} \iff \exists M \in \mathbb{R} \colon a_n \geq M \quad \forall n \in \mathbb{N}                 \]

    \[                 a_n \text{ è \textbf{limitata}} \iff \exists M \geq 0 \colon |a_n| \leq M \quad \forall n \in \mathbb{N}                 \]

Definizione 2.6
\displaystyle 		a_n \text{ è \textbf{(strettamente) crescente}} \iff a_{n+1} (>)\geq a_n \quad \forall n \in \mathbb{N} \displaystyle 		a_n \text{ è \textbf{(strettamente) decrescente}} \iff a_{n+1} (<)\leq a_n \quad \forall n \in \mathbb{N} Definizione 2.8

(269)   \begin{equation*} \begin{split}                $a_n \to \ell \in \mathbb{R} \quad \Longrightarrow \quad a_n$  è limitata\\          		$a_n \to +\infty \quad \Longrightarrow \quad a_n$  è limitata inferiormente\\ 		$a_n \to -\infty \quad \Longrightarrow \quad a_n$  è limitata superiormente \end{split}  \end{equation*}

Proposizione 3.19, Proposizione 3.21
Teorema di Bolzano-Weierstrass: Una successione limitata ha un’estratta convergente. Teorema 5.13
Operazioni con i limiti finiti: Se a_n \to a e b_n \to b, con a, b \in \mathbb{R}, allora:

    \[\begin{aligned}     &|a_n| \to |a| \\     &a_n + b_n \to a + b \\     &a_n \cdot b_n \to a \cdot b \\     &\left( \frac{a_n}{b_n} \right) \to \left( \frac{a}{b} \right) \quad \text{se } b \neq 0 \end{aligned}\]

Teorema 4.1
Operazioni con i limiti infiniti: se a_n \to +\infty,\,\, b_n>0 e limitata, allora \displaystyle 		\frac{a_n}{b_n} \to + \infty e \displaystyle \frac{b_n}{a_n} \to 0. Se invece a_n >0 e a_n \to 0, allora \displaystyle \dfrac{1}{a_n} \to + \infty. Proposizione 4.4
Teorema di permanenza del segno: a_n \to \ell >0, allora a_n >0 definitivamente. Teorema 5.3
Estratte monotone: ogni successione ha un’estratta monotona. Teorema 5.2
Criterio del rapporto: se a_n >0 e \frac{a_{n+1}}{a_n} \to \ell, allora \begin{cases} 			a_n \to 0			& \text{se } \ell <1\\ 			a_n \to + \infty	& \text{se } \ell >1 		\end{cases} Teorema 8.1
Criterio della radice: se a_n >0 e \sqrt[n]{a_n} \to \ell, allora \begin{cases} 			a_n \to 0			& \text{se } \ell <1\\ 			a_n \to + \infty	& \text{se } \ell >1 		\end{cases} Teorema 8.4
Teorema di Stolz-Cesaro: se b_n >0, strettamente crescente e illimitata, allora \displaystyle 		\lim_{n \to + \infty} \frac{a_{n+1}-a_n}{b_{n+1}-b_n} 		= 		\ell 		\quad 		\Longrightarrow 		\quad 		\lim_{n \to + \infty} 		\frac{a_n}{b_n} 		= 		\ell Teorema 11.5


Limiti notevoli e asintotiche.

\displaystyle \lim_{n \to + \infty} a^n 		\begin{cases} 			=+\infty			& \text{se } a >1\\ 			= 1 				& \text{se } a =1\\ 			= 0					& \text{se } a \in (-1,1)\\ 			\text{non esiste}	& \text{se } a \leq -1. 		\end{cases}

Esempio 5.11
\displaystyle x_n \to + \infty 		\quad 		\Longrightarrow 		\quad 		\lim_{n \to + \infty} a^{x_n} 		\begin{cases} 			=+\infty			& \text{se } a >1\\ 			= 1 				& \text{se } a =1\\ 			= 0					& \text{se } a \in (0,1). 		\end{cases} Osservazione 5.12
\displaystyle a >0 \quad \Longrightarrow \quad \lim_{n \to + \infty}\sqrt[n]{a} = 1 Esempio 8.7
\displaystyle \lim_{n \to + \infty} \sqrt[n]{n} 		= 		1. Esempio 9.8
\displaystyle \lim_{n \to + \infty} \left( 1+\frac{1}{n} \right )^n 		= 		e Teorema 7.1 (110)
\displaystyle \lim_{n \to +\infty}\left (1+\frac{x}{n}\right )^n=e^x 		\qquad 		\forall x \in \mathbb{R}. Proposizione 7.7
\displaystyle \lim_{n \to + \infty} \sum_{k=0}^n \frac{1}{k!} = e. Teorema 7.4
\displaystyle  		\lim_{n \to + \infty} \frac{n^n}{n!}=+\infty. Esempio 9.1
\displaystyle  		\frac{n^n}{e^{n-1}} \leq n! \leq \frac{n^n}{e^n} e n 		\quad 		\forall n \in \mathbb{N}, 		\qquad 		n! \sim \frac{n^n}{e^n} \sqrt{2\pi n} Proposizione 9.2 (??)
\displaystyle  		\lim_{n \to + \infty} \frac{n!}{a^n}=+\infty 		\qquad 		\forall a \neq 0. Esempio 9.4
\displaystyle x_n \to + \infty 		\quad 		\Longrightarrow 		\quad 		\lim_{n \to +\infty} \frac{a^{x_n}}{x_n^\beta} 		= 		+\infty 		\qquad 		\forall a >1,\,\, 		\forall \beta \in \mathbb{R}. Esempio 9.6
\displaystyle x_n \to + \infty 		\quad 		\Rightarrow 		\quad 		\lim_{n \to +\infty} \frac{\log_a x_n}{x_n^\beta} 		= 		0 		\qquad 		\forall a \in (0,1) \cup (1,+\infty),\,\, 		\forall \beta >0. Esempio 9.7
\displaystyle a_n \to 0 \,\,\, \text{e} \,\,\, a_n \neq 0 \text{ definitivamente:}

\displaystyle 			\sin a_n \sim a_n, 			\quad 			\cos a_n - 1 \sim \frac{a_n^2}{2}, 			\quad 			e^{a_n} - 1 \sim a_n, 			\quad 			\log(1+a_n) \sim a_n

Esempio 12.2


Forme indeterminate.

+\infty - \infty Esempio 4.5
\displaystyle \frac{\infty}{\infty} Esempi 10.11, 10.12 e 10.13
\displaystyle \frac{0}{0} Esempi 14.9 e 14.11
\displaystyle 0 \cdot \infty Esempi 14.8 e 14.10
\displaystyle 1^{\infty} Definizione 7.2, Esempio 7.6
\displaystyle {\infty}^0 Esempi 9.8, 9.10 e 9.11
\displaystyle 0^0 Esempio 9.12


 

Potenza a esponente reale

Leggi...

In questa sezione definiamo la potenza esponente reale e presentiamo alcune sue proprietà.

Il lettore conosce già il significato della potenza a esponente razionale a^q, con a>0 e q \in \mathbb{Q}: infatti, se q=\frac{m}{n} con n \neq 0, allora si ha

(270)   \begin{equation*} 	a^q 	= 	\sqrt[n]{a^m}. \end{equation*}

Osserviamo però che non è chiarissimo il significato di a^x quando x \in \mathbb{R} \setminus \mathbb{Q}, ovvero quando x è un numero irrazionale.

L’idea principale per dare tale definizione è che la funzione che associa a ogni numero razionale q la potenza a^q è crescente se a>1, quindi è ragionevole aspettarsi che, qualunque significato si voglia dare a a^x, questo debba soddisfare la seguente disuguaglianza

(271)   \begin{equation*} 	a^q < a^x < a^r 	\qquad 	\forall q,r \in \mathbb{Q} \text{ tali che } 	q < x < r. \end{equation*}

Poiché i valori a^q e a^r diventano arbitrariamente vicini all’avvicinarsi di q,r e poiché ogni numero reale x può essere ottenuto come limite di una successione q_n di numeri razionali, si pone la seguente definizione.

Definizione A.1 (potenza a esponente reale). Sia a \in [0,+\infty), sia x \in \mathbb{R} e sia \{q_n\}_{n \in \mathbb{N}} \in \mathbb{Q} una successione di numeri razionali tali che q_n \to x.Si definisce la potenza a esponente reale a^x come il limite delle potenze a^{q_n}, ossia

(272)   \begin{equation*} 			a^x 			\coloneqq 			\lim_{n \to + \infty}a^{q_n}. 		\end{equation*}

    \[\quad\]

Osservazione A.2. (esistenza del limite). Il limite in (272) esiste sempre in quanto la successione a^{q_n} è di Cauchy. Infatti, fissiamo \varepsilon>0 e determiniamo N \in \mathbb{N} tale che

(273)   \begin{equation*} 		1-\varepsilon < a^{-\frac{1}{N}}, a^{\frac{1}{N}} < 1-\varepsilon, \end{equation*}

che esiste poiché \lim_{N \to +\infty} a^{\frac{1}{N}}= \lim_{N \to +\infty} a^{-\frac{1}{N}}=1 per l’esempio 8.7. Poiché q_n \to x per il teorema 6.2, la successione q_n è di Cauchy quindi esiste \bar{n} \in \mathbb{N} tale che

(274)   \begin{equation*} 		- \frac{1}{N} < q_n - q_m < \frac{1}{N} 		\qquad \forall n,m \geq \bar{n}  \end{equation*}

e quindi, per la monotonia della funzione esponenziale sui razionali, si ha

(275)   \begin{equation*} 		|1 - a^{q_m- q_n}| < \varepsilon 		\quad \forall n,m \geq \bar{n}.  	\end{equation*}

Poiché la successione a^{q_n} è limitata (da a^M con M>x tale che q_n < M per ogni n \geq \bar{n}), si ha

(276)   \begin{equation*} 		0 \leq |a^{q_n} - a^{q_m}| 		\leq 		a^{q_n}|1 - a^{q_m- q_n}| 		\leq 		a^M \varepsilon 		\qquad 		\forall n,m \geq \bar{n}. 	\end{equation*}

Dall’arbitrarietà di \varepsilon segue quindi che a^{q_n} è di Cauchy e quindi ha limite per il teorema 6.2.

Osservazione A.3. (indipendenza di a^x dalla successione scelta). Analogamente, si può provare che la definizione è ben posta, cioè il valore a^x non dipende dalla successione crescente q_n scelta: se r_n è un’altra successione crescente di numeri razionali che converge a x, si ha

(277)   \begin{equation*} 		\lim_{n\to + \infty} a^{q_n} 		= 		\lim_{n\to + \infty} a^{r_n}. 	\end{equation*}

Infatti, fissiamo \varepsilon>0 e determiniamo N \in \mathbb{N} tale che

(278)   \begin{equation*} 		1-\varepsilon < a^{-\frac{1}{N}}, a^{\frac{1}{N}} < 1-\varepsilon. 	\end{equation*}

Dato che q_n \to x e r_n \to x, esiste \bar{n} \in \mathbb{N} tale che

(279)   \begin{equation*} 		- \frac{1}{N} < |q_n - r_n| < \frac{1}{N} 		\qquad \forall n \geq \bar{n}. 	\end{equation*}

Con le stesse stime di sopra, si mostra che i limiti a cui a^{q_n} e a^{r_n} convergono sono gli stessi.

La potenza a esponente reale eredita le proprietà delle potenze a esponente razionale, come stabilito dalla seguente proposizione.

Proposizione A.4 (proprietà delle potenze a esponente reale). Sia a \in (0,1) \cup (1,+\infty), e siano x,y \in \mathbb{R}. Valgono le seguenti proprietà

    \[\quad\]

  1. Se x=\dfrac{m}{k} \in \mathbb{Q}, allora la potenza a esponente reale a^x coincide con l’ordinaria potenza a esponente razionale a^{\frac{m}{k}}= \sqrt[k]{a^m}.
  2.  

  3. La funzione x \mapsto a^x è strettamente crescente se a>1 e strettamente decrescente se a \in (0,1).
  4.  

  5. a^{x+y} = a^x \cdot a^y.
  6.  

  7. a^{xy} = (a^x)^y.
  8.  

  9. Se x_n \to x, allora a^{x_n} \to a^x

    \[\quad\]

Dimostrazione. Mostriamo separatamente le diverse parti.

    \[\quad\]

  1. Se x=\frac{m}{k} con m \in \mathbb{Z} e k \in \mathbb{N}, allora si può scegliere come successione q_n a valori in \mathbb{Q}, la successione costantemente pari a x. Quindi

    (280)   \begin{equation*} 			a^x  			= 			\lim_{n\to +\infty} a^{\frac{m}{k}} 			= 			\sqrt[k]{a^m}. 		\end{equation*}

  2.  

  3. Dimostriamo la proprietà solo per a>1 in quanto l’altro caso è analogo. Se x < y e q_n,r_n sono successioni a valori razionali tali che q_n \to x e r_n \to y, allora per la proposizione 5.4 vale q_n < r_n definitivamente. Inoltre è possibile determinare due numeri razionali s,t \in \mathbb{Q} tale che

    (281)   \begin{equation*} 			q_n < s <  t < r_n 			\qquad 			\text{definitivamente}. 		\end{equation*}

    Per tale diseguaglianza, per la monotonia della potenza a esponente razionale e per la proposizione 5.5, si ha

    (282)   \begin{equation*} 			a^x= \lim_{n \to +\infty} a^{q_n} \leq a^s 			< 			a^t 			\leq 			\lim_{n \to +\infty} a^{r_n} 			= 			a^y. 		\end{equation*}

  4.  

  5. Siano q_n \to x e r_n \to y successioni a valori razionali. Allora q_n+r_n è una successione a valori razionali e, per il teorema 4.1, vale q_n+r_n \to x+y. Pertanto si ha

    (283)   \begin{equation*} 			a^{x+y} 			= 			\lim_{n \to + \infty} 			a^{q_n + r_n} 			= 			\lim_{n \to + \infty} a^{q_n} \cdot a^{r_n} 			= 			\left ( \lim_{n \to + \infty} a^{q_n} \right ) 			\cdot 			\left ( \lim_{n \to + \infty} a^{r_n} \right ) 			= 			a^x \cdot a^y. 		\end{equation*}

  6.  

  7. La dimostrazione è analoga al punto precedente.
  8.  

  9. Supponiamo a>1 (l’altro caso è analogo). L’asserto è dato dalla definizione se x_n è a valori in \mathbb{Q}. Se x_n è una successione a valori in \mathbb{R}, per ogni n\in\mathbb{N} esiste q_n\in\mathbb{Q} tale che

    (284)   \begin{equation*} 			|x_n-q_n|<\frac{1}{n} 		\end{equation*}

    allora per 5.8 converge a x e quindi, sempre dalla definizione A.1. Da (284) si ha

        \begin{equation*} 			-\frac{1}{n}<x_n-q_n<\frac{1}{n}\iff -\frac{1}{n}+q_n<x_n<\frac{1}{n}+q_n. 		\end{equation*}

    Poiché a>1 possiamo applicare la funzione esponenziale e ottenere

        \begin{equation*} 		a^{-\frac{1}{n}+q_n}<a^{x_n}<a^{\frac{1}{n}+q_n}\iff a^{-\frac{1}{n}}\cdot a^{q_n}<a^{x_n}<a^{\frac{1}{n}}\cdot a^{q_n}\qquad\forall n\in\mathbb{N}. 		\end{equation*}

    Per il teorema del confronto possiamo concludere che a^{x_n}\to a^x.


 
 

Riferimenti bibliografici

[1] Bertsch, M. & Dal Passo, R. & Giacomelli, L., Analisi Matematica,
McGraw-Hill (2011).

[2] De Marco, G., Analisi Uno, primo corso di analisi matematica Zanichelli (1996).

[3] Giusti, G., Analisi Matematica 1, Bollati Boringhieri (2002).

[4] Qui Si Risolve, Funzioni elementari – Volume 1

[5] Qui Si Risolve, Funzioni elementari – Volume 2

[6] Qui Si Risolve, Teoria sulle funzioni

[7] Qui Si Risolve, Teoria sui limiti

[8] Qui Si Risolve, Il teorema ponte

[9] Qui Si Risolve, Le successioni definite per ricorrenza – Esercizi misti

 
 

Tutta la teoria di analisi matematica

Leggi...

  1. Teoria Insiemi
  2. Il metodo della diagonale di Cantor
  3. Logica elementare
  4. Densità dei numeri razionali nei numeri reali
  5. Insiemi Numerici \left(\mathbb{N},\, \mathbb{Z},\, \mathbb{Q}\right)
  6. Il principio di induzione
  7. Gli assiomi di Peano
  8. L’insieme dei numeri reali: costruzione e applicazioni
  9. Concetti Fondamentali della Retta Reale: Sintesi Teorica
  10. Costruzioni alternative di \mathbb{R}
  11. Binomio di Newton
  12. Spazi metrici, un’introduzione
  13. Disuguaglianza di Bernoulli
  14. Disuguaglianza triangolare
  15. Teoria sulle funzioni
  16. Funzioni elementari: algebriche, esponenziali e logaritmiche
  17. Funzioni elementari: trigonometriche e iperboliche
  18. Funzioni goniometriche: la guida essenziale
  19. Teorema di Bolzano-Weierstrass per le successioni
  20. Criterio del rapporto per le successioni
  21. Definizione e proprietà del numero di Nepero
  22. Limite di una successione monotona
  23. Successioni di Cauchy
  24. Il teorema ponte
  25. Teoria sui limiti
  26. Simboli di Landau
  27. Funzioni continue – Teoria
  28. Il teorema di Weierstrass
  29. Il teorema dei valori intermedi
  30. Il teorema della permanenza del segno
  31. Il teorema di Heine-Cantor
  32. Il teorema di esistenza degli zeri
  33. Il metodo di bisezione
  34. Teorema ponte versione per le funzioni continue
  35. Discontinuità di funzioni monotone
  36. Continuità della funzione inversa
  37. Teorema delle contrazioni o Teorema di punto fisso di Banach-Caccioppoli
  38. Teoria sulle derivate
  39. Calcolo delle derivate: la guida pratica
  40. Teoria sulle funzioni convesse
  41. Il teorema di Darboux
  42. I teoremi di de l’Hôpital
  43. Teorema di Fermat
  44. Teoremi di Rolle e Lagrange
  45. Il teorema di Cauchy
  46. Espansione di Taylor: teoria, esempi e applicazioni pratiche
  47. Polinomi di Taylor nei limiti: istruzioni per l’uso
  48. Integrali definiti e indefiniti
  49. Teorema fondamentale del calcolo integrale (approfondimento)
  50. Integrali ricorsivi
  51. Formule del trapezio, rettangolo e Cavalieri-Simpson
  52. Teoria sugli integrali impropri
  53. Funzioni integrali – Teoria
  54. Introduzione ai numeri complessi – Volume 1 (per un corso di ingegneria — versione semplificata)
  55. Introduzione ai numeri complessi – Volume 1 (per un corso di matematica o fisica)
  56. Serie numeriche: la guida completa
  57. Successioni di funzioni – Teoria
  58. Teoremi sulle successioni di funzioni
    1. 58a. Criterio di Cauchy per la convergenza uniforme
    2. 58b. Limite uniforme di funzioni continue
    3. 58c. Passaggio al limite sotto il segno di integrale
    4. 58d. Limite uniforme di funzioni derivabili
    5. 58e. Piccolo teorema del Dini
    6. 58f. Procedura diagonale e teorema di Ascoli-Arzela
  59. Serie di funzioni – Teoria
  60. Serie di potenze – Teoria
  61. Serie di Fourier – Teoria e applicazioni
  62. Integrali multipli — Parte 1 (teoria)
  63. Integrali multipli — Parte 2 (teoria e esercizi misti)
  64. Regola della Catena — Teoria ed esempi.
  65. Jacobiano associato al cambiamento di coordinate sferiche
  66. Guida ai Massimi e Minimi: Tecniche e Teoria nelle Funzioni Multivariabili
  67. Operatore di Laplace o Laplaciano
  68. Teoria equazioni differenziali
  69. Equazione di Eulero
  70. Teoria ed esercizi sulla funzione Gamma di Eulero
  71. Teoria ed esercizi sulla funzione Beta
  72. Approfondimento numeri complessi
  73. Diverse formulazioni dell’assioma di completezza
  74. Numeri di Delannoy centrali
  75. Esercizi avanzati analisi

 
 

Tutte le cartelle di Analisi Matematica

Leggi...

  1. Prerequisiti di Analisi
    1. Ripasso algebra biennio liceo
    2. Ripasso geometria analitica
    3. Ripasso goniometria e trigonometria
    4. Errori tipici da evitare
    5. Insiemi numerici N,Z,Q,R
    6. Funzioni elementari
    7. Logica elementare
    8. Insiemi
  2. Successioni
    1. Teoria sulle Successioni
    2. Estremo superiore e inferiore
    3. Limiti base
    4. Forme indeterminate
    5. Limiti notevoli
    6. Esercizi misti Successioni
    7. Successioni per ricorrenza
  3. Funzioni
    1. Teoria sulle funzioni
    2. Verifica del limite in funzioni
    3. Limite base in funzioni
    4. Forme indeterminate in funzioni
    5. Limiti notevoli in funzioni
    6. Calcolo asintoti
    7. Studio di funzione senza derivate
    8. Dominio di una funzione
    9. Esercizi misti Funzioni
    10. Esercizi misti sui Limiti
  4. Funzioni continue-lipschitziane-holderiane
    1. Teoria sulle Funzioni continue-lipschitziane-holderiane
    2. Continuità delle funzioni
    3. Continuità uniforme
    4. Teorema degli zeri
    5. Esercizi sul teorema di Weierstrass senza l’uso delle derivate
  5. Calcolo differenziale
    1. Derivate
    2. Calcolo delle derivate
    3. Retta tangente nel calcolo differenziale
    4. Punti di non derivabilità nel calcolo differenziale
    5. Esercizi sul teorema di Weierstrass con l’uso delle derivate
    6. Studio di funzione completo nel calcolo differenziale
    7. Esercizi teorici nel calcolo differenziale
    8. Metodo di bisezione
    9. Metodo di Newton
  6. Teoremi del calcolo differenziale
    1. Teoria sui Teoremi del calcolo differenziale
    2. Teorema di Rolle
    3. Teorema di Lagrange
    4. Teorema di Cauchy
    5. Teorema di De L’Hôpital
  7. Calcolo integrale
    1. Integrale di Riemann
    2. Integrali immediati
    3. Integrale di funzione composta
    4. Integrali per sostituzione
    5. Integrali per parti
    6. Integrali di funzione razionale
    7. Calcolo delle aree
    8. Metodo dei rettangoli e dei trapezi
    9. Esercizi Misti Integrali Indefiniti
    10. Esercizi Misti Integrali Definiti
  8. Integrali impropri
    1. Teoria Integrali impropri
    2. Carattere di un integrale improprio
    3. Calcolo di un integrale improprio
  9. Espansione di Taylor
    1. Teoria Espansione di Taylor
    2. Limiti di funzione con Taylor
    3. Limiti di successione con Taylor
    4. Stime del resto
  10. Funzioni integrali (Approfondimento)
    1. Teoria Funzioni integrali (Approfondimento)
    2. Studio di funzione integrale
    3. Limiti con Taylor e De L’Hôpital
    4. Derivazione di integrali parametrici (Tecnica di Feynmann)
  11. Numeri Complessi
    1. Teoria Numeri complessi
    2. Espressioni con i numeri complessi
    3. Radice di un numero complesso
    4. Equazioni con i numeri complessi
    5. Disequazioni con i numeri complessi
    6. Esercizi misti Numeri complessi
  12. Serie numeriche
    1. Teoria Serie numeriche
    2. Esercizi Serie a termini positivi
    3. Esercizi Serie a termini di segno variabile
    4. Esercizi Serie geometriche e telescopiche
  13. Successioni di funzioni
    1. Teoria Successioni di funzioni
    2. Esercizi Successioni di funzioni
  14. Serie di funzioni
    1. Teoria Serie di funzioni
    2. Esercizi Serie di funzioni
  15. Serie di potenze
    1. Teoria Serie di potenze
    2. Esercizi Serie di potenze
  16. Serie di Fourier
    1. Teoria Serie di Fourier
    2. Esercizi Serie di Fourier
  17. Trasformata di Fourier
    1. Teoria Trasformata di Fourier
    2. Esercizi Trasformata di Fourier
  18. Funzioni di più variabili
    1. Teoria Funzioni di più variabili
    2. Massimi e minimi liberi e vincolati
    3. Limiti in due variabili
    4. Integrali doppi
    5. Integrali tripli
    6. Integrali di linea di prima specie
    7. Integrali di linea di seconda specie
    8. Forme differenziali e campi vettoriali
    9. Teorema di Gauss-Green
    10. Integrali di superficie
    11. Flusso di un campo vettoriale
    12. Teorema di Stokes
    13. Teorema della divergenza
    14. Campi solenoidali
    15. Teorema del Dini
  19. Equazioni differenziali lineari e non lineari
    1. Teoria equazioni differenziali lineari e non lineari
    2. Equazioni differenziali lineari e non lineari del primo ordine omogenee
  20. Equazioni differenziali lineari
    1. Del primo ordine non omogenee
    2. Di ordine superiore al primo,a coefficienti costanti,omogenee
    3. Di ordine superiore al primo,a coefficienti costanti,non omogenee
    4. Di Eulero,di Bernoulli,di Clairaut,di Lagrange e di Abel
    5. Non omogenee avente per omogenea associata un’equazione di Eulero
    6. Sistemi di EDO
  21. Equazioni differenziali non lineari
    1. A variabili separabiliO
    2. A secondo membro omogeneo
    3. Del tipo y’=y(ax+by+c)
    4. Del tipo y’=y(ax+by+c)/(a’x+b’y+c’)
    5. Equazioni differenziali esatte
    6. Mancanti delle variabili x e y
    7. Cenni sullo studio di un’assegnata equazione differenziale non lineare
    8. Di Riccati
    9. Cambi di variabile: simmetrie di Lie
  22. Analisi complessa
    1. Fondamenti
    2. Funzioni olomorfe
    3. Integrale di Cauchy e applicazioni
    4. Teorema della curva di Jordan e teorema fondamentale dell’Algebra
    5. Teorema di inversione di Lagrange
    6. Teorema dei Residui
    7. Funzioni meromorfe
    8. Prodotti infiniti e prodotti di Weierstrass
    9. Continuazione analitica e topologia
    10. Teoremi di rigidità di funzioni olomorfe
    11. Trasformata di Mellin
  23. Equazioni alle derivate parziali
    1. Equazioni del primo ordine
    2. Equazioni del secondo ordine lineari
    3. Equazioni non-lineari
    4. Sistemi di PDE
  24. Funzioni speciali
    1. Funzione Gamma di Eulero
    2. Funzioni Beta,Digamma,Trigamma
    3. Integrali ellittici
    4. Funzioni di Bessel
    5. Funzione zeta di Riemann e funzioni L di Dirichlet
    6. Funzione polilogaritmo
    7. Funzioni ipergeometriche
  25. Analisi funzionale
    1. Misura e integrale di Lebesgue
    2. Spazi Lp,teoremi di completezza e compattezza
    3. Spazi di Hilbert,serie e trasformata di Fourier
    4. Teoria e pratica dei polinomi ortogonali
    5. Spazi di Sobolev
  26. Complementi
    1. Curiosità e approfondimenti
    2. Compiti di analisi
    3. Esercizi avanzati analisi
  27. Funzioni Convesse

 
 

Tutti gli esercizi di geometria

In questa sezione vengono raccolti molti altri esercizi che coprono tutti gli argomenti di geometria proposti all’interno del sito con lo scopo di offrire al lettore la possibilità di approfondire e rinforzare le proprie competenze inerenti a tali argomenti.

Strutture algebriche.





 
 

Risorse didattiche aggiuntive per approfondire la matematica

Leggi...

  • Math Stack Exchange – Parte della rete Stack Exchange, questo sito è un forum di domande e risposte specificamente dedicato alla matematica. È una delle piattaforme più popolari per discutere e risolvere problemi matematici di vario livello, dall’elementare all’avanzato.
  • Art of Problem Solving (AoPS) – Questo sito è molto noto tra gli studenti di matematica di livello avanzato e i partecipanti a competizioni matematiche. Offre forum, corsi online, e risorse educative su una vasta gamma di argomenti.
  • MathOverflow – Questo sito è destinato a matematici professionisti e ricercatori. È una piattaforma per domande di ricerca avanzata in matematica. È strettamente legato a Math Stack Exchange ma è orientato a un pubblico con una formazione più avanzata.
  • PlanetMath – Una comunità collaborativa di matematici che crea e cura articoli enciclopedici e altre risorse di matematica. È simile a Wikipedia, ma focalizzata esclusivamente sulla matematica.
  • Wolfram MathWorld – Una delle risorse online più complete per la matematica. Contiene migliaia di articoli su argomenti di matematica, creati e curati da esperti. Sebbene non sia un forum, è una risorsa eccellente per la teoria matematica.
  • The Math Forum – Un sito storico che offre un’ampia gamma di risorse, inclusi forum di discussione, articoli e risorse educative. Sebbene alcune parti del sito siano state integrate con altri servizi, come NCTM, rimane una risorsa preziosa per la comunità educativa.
  • Stack Overflow (sezione matematica) – Sebbene Stack Overflow sia principalmente noto per la programmazione, ci sono anche discussioni rilevanti di matematica applicata, specialmente nel contesto della scienza dei dati, statistica, e algoritmi.
  • Reddit (r/Math) – Un subreddit popolare dove si possono trovare discussioni su una vasta gamma di argomenti matematici. È meno formale rispetto ai siti di domande e risposte come Math Stack Exchange, ma ha una comunità attiva e molte discussioni interessanti.
  • Brilliant.org – Offre corsi interattivi e problemi di matematica e scienza. È particolarmente utile per chi vuole allenare le proprie capacità di problem solving in matematica.
  • Khan Academy – Una risorsa educativa globale con lezioni video, esercizi interattivi e articoli su una vasta gamma di argomenti di matematica, dalla scuola elementare all’università.






Document









Document